NCC ELECTRONIC FETAL MONITORING CERTIFICATION EXAM VERSION B 2023 BRAND NEW VERSION 100+ QUESTIONS AND CORRECT ANSWERS|AGRADE

Which of the following factors can have a negative effect on uterine blood flow?
a. Hypertension
b. Epidural
c. Hemorrhage
d. Diabetes
e. All of the above
e. All of the above

How does the fetus compensate for decreased maternal circulating volume?
a. Increases cardiac output by increasing stroke volume.
b. Increases cardiac output by increasing it’s heart rate.
c. Increases cardiac output by increasing fetal movement.
b. Increases cardiac output by increasing it’s heart rate.

Stimulating the vagus nerve typically produces:
a. A decrease in the heart rate
b. An increase in the heart rate
c. An increase in stroke volume
d. No change
a. A decrease in the heart rate

What initially causes a chemoreceptor response?
a. Epidurals
b. Supine maternal position
c. Increased CO2 levels
d. Decreased O2 levels
e. A & C
f. A & B
g. C & D
g. C & D

The vagus nerve begins maturation 26 to 28 weeks. Its dominance results in what effect to the FHR baseline?
a. Increases baseline
b. Decreases baseline
b. Decreases baseline

T/F: Oxygen exchange in the placenta takes place in the intervillous space.
True

T/F: The parasympathetic nervous system is a cardioaccelerator.
False

T/F: Baroreceptors are stretch receptors which respond to increases or decreases in blood pressure.
True

T/F: There are two electronic fetal monitoring methods of obtaining the fetal heart rate: the ultrasound transducer and the fetal spiral electrode.
True

T/F: Variability can be determined with the fetoscope.
False

T/F: Because the ultrasound transducer and toco transducer are sealed units, they can be dipped in warm water to make cleaning easier.
False

T/F: The most common artifact with the ultrasound transducer system for fetal heart rate is increased variability.
True

T/F: All fetal monitors contain a logic system designed to reject artifact.
True

T/F: The monitor should always be tested before starting a tracing, either external or internal mode and labeled a test.
True

T/F: The paper speed on the fetal monitor should always be set at 1cm/min.
False

T/F: Both internal and external monitoring methods are equally accurate means of obtaining the fetal heart rate and contraction patterns.
False

T/F: The external toco is usually placed over the uterine fundus to pick up contractions.
True

T/F: The external toco gives measurable uterine pressure.
False

T/F: The fetal spiral electrode can be placed when vaginal bleeding of unknown origin is present.
False

T/F: The ultrasound transducer is usually placed on the side of the uterus over the baby’s back, as the fetal heart is heard best there.
True

T/F: The spiral electrode is used to more accurately determine the frequency, duration, and intensity of uterine contractions.
False

T/F: The heart rate from a well-applied fetal spiral electrode can only be fetal, not maternal.
False

T/F: The intrauterine catheter is used to pick up the fetal heart rate.
False

T/F: The internal spiral electrode may pick up the maternal heart rate if the baby has died.
True

T/F: Fetal arrhythmias can be seen on both internal and external monitor tracings.
True

T/F: Variability and periodic changes can be detected with both internal and external monitoring.
True

T/F: Variable decelerations are a result of cord compression.
True

T/F: The presence of FHR accelerations in the intrapartum and antepartum periods is a sign of adequate fetal oxygenation.
True

T/F: Variable decelerations are a vagal response.
True

T/F: Late decelerations have a gradual decrease in FHR (onset to nadir 30 seconds) and are delayed in timing with the nadir of the deceleration occurring after the peak of the contraction.
True

T/F: The fetal heart rate baseline can be determined during periods of marked variability.
False

T/F: Anything that affects maternal blood flow (cardiac output) can affect the blood flow through the placenta.
True

T/F: Variable decelerations are the most frequently seen fetal heart rate deceleration pattern in labor.
True

T/F: Minimal variability is always an indicator of hypoxia and a Cesarean section is indicated.
False

What is your first intervention in management of a patient experiencing variable decelerations?
a. Immediate delivery
b. Change maternal position
c. No treatment indicated
d. Oxygen
e. Stop oxytocin infusion
b. Change maternal position

Etiology of a baseline FHR of 165bpm occurring for the last hour can be:

  1. Maternal supine hypotension
  2. Maternal fever
  3. Maternal dehydration
  4. Unknown
    a. 1 and 2
    b. 1, 2 and 3
    c. 2, 3 and 4
    c. 2, 3 and 4

What is the most probable cause of recurrent late decelerations?
a. Utero-placental insufficiency
b. Head compression
c. Cord compression
d. Maternal position change
a. Utero-placental insufficiency

The most prevalent risk factor associated with fetal death before the onset of labor is:
a. Low socioeconomic status
b. Fetal malpresentation
c. Uteroplacental insufficiency
d. Uterine anomalies
c. Uteroplacental insufficiency

Which of the following is NOT used for antepartum fetal surveillance?
a. Fetal movement counting
b. Antepartum fetal heart rate testing
c. Biophysical profile testing
d. Maternal HCG levels
d. Maternal HCG levels

Which of the following conditions is not an indication for antepartum fetal surveillance?
a. Gestational hypertension
b. Diabetes in pregnancy
c. Fetus in breech presentation
d. Decreased fetal movement
c. Fetus in breech presentation

Which of the following does not affect the degree of fetal activity?
a. Vibroacoustic stimulation
b. Smoking
c. Fetal position
d. Gestational age
a. Vibroacoustic stimulation

To be considered reactive, a nonstress test must have:
a. 4 fetal heart rate accelerations in a 20 minute window
b. 2 fetal heart rate accelerations in a 10 minute window
c. 4 fetal heart rate accelerations in a 40 minute window
d. 2 fetal heart rate accelerations in a 20 minute window
d. 2 fetal heart rate accelerations in a 20 minute window

If a nonstress test is nonreactive after 40 minutes, the next step should be:
a. Have the client go home and do fetal movement counts
b. Do a biophysical profile or contraction stress test
c. Repeat the nonstress test within a week
d. Admit the client for delivery
b. Do a biophysical profile or contraction stress test

All of the following are components of a biophysical profile except:
a. Contraction stress test
b. Assessment of fetal breathing
c. Amniotic fluid volume measurement
d. Fetal movement assessment
a. Contraction stress test

A modified biophysical profile includes a nonstress test and:
a. Contraction stress test
b. Ultrasound assessment of fetal movement
c. Ultrasound assessment of amniotic fluid volume
d. Fetal movement counts
c. Ultrasound assessment of amniotic fluid volume

For a contraction stress test to be interpretable, you must have a minimum of:
a. 5 contractions in a 10-minute window
b. 3 contractions in a 10-minute window
c. 4 contractions in a 10-minute window
d. 2 contractions in a 10-minute window
b. 3 contractions in a 10 minute window

A negative contraction stress test is one in which:
a. No contractions are seen
b. There are late decelerations with > 50% of the contractions seen
c. There are no fetal heart rate late decelerations with the contractions
d. There is one fetal heart rate deceleration seen
c. There are no fetal heart rate late decelerations with the contractions

According to AWHONN, the normal baseline Fetal Heart Rate (FHR) is
A. 90-150 bpm
B. 100-170 bpm
C. 110-160 bpm
D. 120-140 bpm
C. 110-160 bpm

What are the two most important characteristics of the FHR?
A. Rate and decelerations
B. Variability and accelerations
C. Variability and decelerations
D. Rate and variability
B. Variability and accelerations

You recognize that an FHR tracing has been showing a decrease in variability for the last 45 minutes. Your first intervention should be to
A. Encourage ambulation
B. Administer oxygen
C. Discontinue IV fluids
D. Increase Pitocin rate
B. Administer oxygen

Resuscitation measures improves the baby’s variability, but the FHR is still not reactive. You attempt fetal scalp stimulation (FSE) because you know that a well-oxygenated fetus will respond to FSE with a(n)
A. Acceleration
B. Deceleration
C. Fetal movement
D. Sleep pattern
A. Acceleration

You are evaluating a patient in the Prenatal Testing Department who has just completed a biophysical profile (BPP). You suspect that there could be chronic fetal asphyxia because the score is below
A. 10
B. 6
C. 8
B. 6

When using a fetal scalp electrode (FSE), you notice an abnormally low FHR on the monitor. You should first
A. Compare maternal pulse simultaneously with FHR
B. Remove FSE
C. Call the doctor immediately
D. Turn off the monitor
A. Compare maternal pulse simultaneously with FHR

T/F: Umbilical cord influences that can alter blood flow include true knots, hematomas, and number of umbilical vessels.
True

T/F: Low amplitude contractions are not an early sign of preterm labor.
False

T/F: Preterm contractions are usually painful.
False

T/F: Corticosteroid administration may cause an increase in FHR accelerations.
False

T/F: Corticosteroid administration may cause an increase in FHR.
True

T/F: Contractions cause an increase in uterine venous pressure and a decrease in uterine artery perfusion.
True

As a result of the intrinsic fetal response to oxygen deprivation, increased catecholamine levels cause the peripheral blood flow to decrease while the blood flow to vital organs increases. These flow changes along with increased catecholamine secretions have what effect on fetal blood pressure and fetal heart rate?
A. Increase BP and increase HR
B. Increase BP and decrease HR
C. Decrease BP and increase HR
D. Decrease BP and decrease HR
B. Increase BP and decrease HR

All of the following might indicate a pseudosinusoidal pattern as opposed to a sinusoidal pattern, except:
A. Recent administration of narcotics to mother
B. Accelerations in FHR
C. Moderate variability
D. Frequency of oscillations of two to five cycles/min
D. Frequency of oscillations of two to five cycles/min

All of the following are appropriate interventions for fetal tachycardia except:
A. Increase maternal IV fluid rate
B. Assess maternal vital signs
C. Perform SVE
D. Administer oxygen
C. Perform SVE

During a term antepartum NST (non-stress test), you notice several variable decelerations that decrease at least 15 bpm and last at least 15 sec long. Which of the following is the least likely explanation?
A. True knot
B. Gestational diabetes
C. Umbilical cord entanglement
D. Oligohydramnios
B. Gestational diabetes

All of the following are likely causes of prolonged decelerations except:
A. Uterine tachysystole
B. Prolapsed cord
C. Maternal hypotension
D. Maternal fever
D. Maternal fever

_ decelerations occur with less than 50% of contractions.
A. Recurrent
B. Intermittent
C. Repetitive
B. Intermittent

_ decelerations occur with greater than or equal to 50% of contractions.
A. Recurrent
B. Intermittent
C. Repetitive
A. Recurrent

All of the following could likely cause minimal variability in FHR except
A. Magnesium sulfate administration
B. Fetal sleep cycle
C. Narcotic administration
D. Ephedrine administration
D. Ephedrine administration

When an IUPC has been placed, Montevideo units must be _ or greater for adequate cervical change to occur.
A. 100
B. 200
C. 300
D. 400
B. 200

The __ increases the heart rate and strengthens myocardial contractions through the release of epinephrine and nonepinephrine.
A. Sympathetic nervous system
B. Parasympathetic nervous system
A. Sympathetic nervous system

The __, through stimulation of the vagus nerve, reduces FHR and maintains variability.
A. Sympathetic nervous system
B. Parasympathetic nervous system
B. Parasympathetic nervous system

What would be a suspected pH in a fetus whose FHTs included recurrent late decelerations during labor?
A. 7.10
B. 7.26
C. 7.32
D. 7.41
A. 7.10

What is the most common cause of sinusoidal patterns?
A. Prolapsed cord
B. Rh incompatibility
C. Recurrent late decelerations
D. Oligohydramnios
B. Rh incompatibility

Before _ weeks of gestation, an increase in FHR that peaks at least 10 bpm above the baseline and lasts at least 10 seconds is considered an acceleration.
A. 28
B. 30
C. 32
D. 36
C. 32

The expected response of the fetal heart rate to active fetal movement of a 31-week gestational age fetus is:
a. Suppression of normal short term variability for 15 seconds
b. Acceleration of at least 15 beats per minute for 15 seconds
c. Acceleration followed by a 15-second deceleration of the heart rate
d. Acceleration of at least 10 beats per minute for 10 seconds
d. Acceleration of at least 10 beats per minute for 10 seconds

The nurse notes a pattern of variable decelerations to 75 bpm on the fetal monitor. The initial nursing action is to:
a. Reposition the woman
b. Administer oxygen
c. Increase the intravenous fluid infusion
d. Stimulate the fetal scalp
a. Reposition the woman

The tocotransducer should be placed:
a. In the suprapubic area
b. In the fundal area
c. Over the xiphoid process
d. Within the uterus
b. In the fundal area

The nurse notes a pattern of decelerations on the fetal monitor that begins shortly after the contraction and returns to baseline just before the contraction is over. The correct nursing response is to:
a. Give the woman oxygen by facemask at 8-10 L/min
b. Position the woman on her opposite side
c. Increase the rate of the woman’s intravenous fluid
d. Continue to observe and record the normal pattern
d. Continue to observe and record the normal pattern

Determining the FHR baseline requires the nurse to approximate the mean FHR rounded to increments of 5 bpm during a _-minute window (excluding accelerations and decelerations).
A. 2
B. 5
C. 10
D. 20
C. 10

Uterine tachysystole is observed when there are
A. 5 or more contractions in 10 min
B. 6 or more contractions in 10 min
C. 10 or more contractions in 10 min
D. 7 or more contractions in 10 min
B. 6 or more contractions in 10 min

Which of the following interventions would best stimulate an acceleration in the FHR?
A. Provide juice to patient
B. Perform vaginal exam
C. Turn patient on left side
D. Vibroacoustic stimulation
B. Perform vaginal exam

Scalp stimulation

Assessment of the _ is an indirect measurement of fetal oxygenation.
A. Fetal heart rate
B. Fetal scalp sampling
C. Uterine activity
D. Direct Coombs
A. Fetal heart rate

T/F: Intrauterine pressure catheters (IUPCs) do not increase risk for infection when placed on patients with intact membranes.
False

Membranes must be ruptured for use; infection is a risk

What are abnormal fetal heart rate tracings predictive of?
A. Likelihood of spontaneous vaginal delivery
B. Newborn condition at time of delivery
C. Fetal acid-base abnormalities
D. Fetal intrauterine growth
C. Fetal acid-base abnormalities

Which of the following is not an intervention that should be implemented in a patient with uterine tachysystole?
A. Administer terbutaline
B. Increase IV fluid rate
C. Decrease or discontinue IV oxytocin
D. Prepare patient for cesarean section
D. Prepare patient for cesarean section

Which of the following is most effective in determining the strength of a patient’s contractions?
A. Patient report
B. Tocodynanamometer tracing
C. RN palpation
D. Sterile vaginal exam during a contraction
C. RN palpation

The FHR is controlled by the
A. Sympathetic nervous system
B. Sinoatrial node
C. Atrioventricular node
D. Parasympathetic nervous system
B. Sinoatrial node

How do baseline heart rates differ in premature fetuses?
A. They are often lower
B. They are often higher
C. They are less likely to have decelerations
D. They experience longer accelerations
B. They are often higher

T/F: If etiology of fetal tachycardia is secondary to extrauterine infection, FHR will return to normal as maternal fever resolves.
True

T/F: Fetal tachycardia is a normal compensatory response to transient fetal hypoxemia.
True

Fetal heart rate bradycardia is defined as
A. FHR <110bpm lasting 10 min or greater
B. FHR <110bpm lasting 2 min or greater
C. FHR <110bpm lasting 20 min or greater
A. FHR <110bpm lasting 10 min or greater

_ variability warrants cesarean section delivery.
A. Minimal
B. Moderate
C. Marked
D. Absent
D. Absent

At how many weeks gestation should FHR variability be normal in manner?
A. 24 weeks
B. 28 weeks
C. 32 weeks
D. 36 weeks
B. 28 weeks

A deceleration from 145bpm down to 100bpm lasting 12 minutes may be defined as a
A. Prolonged deceleration
B. Variable deceleration
C. Late deceleration
D. Baseline change
D. Baseline change

Reduced respiratory gas exchange from persistent decelerations may cause a rise in fetal PCO2, which leads first to , then .
A. Respiratory alkalosis; metabolic acidosis
B. Respiratory acidosis; metabolic acidosis
C. Respiratory alkalosis; metabolic alkalosis
D. Respiratory acidosis; metabolic acidosis
B. Respiratory acidosis; metabolic acidosis

Decreased intervillious exchange of oxygenated blood resulting in fetal hypoxia is typically present in _.
A. Variable decelerations
B. Late decelerations
C. Early decelerations
D. Accelerations
B. Late decelerations

Place the following interventions for a sinusoidal FHR in the correct order:

  1. Prepare for cesarean delivery
  2. Place patient in lateral position
  3. Determine if pattern is related to narcotic analgesic administration
  4. Provide oxygen via face mask
    A. 4, 2, 3, 1
    B. 3, 1, 2, 4
    C. 4, 3, 2, 1
    D. 3, 2, 4, 1
    D. 3, 2, 4, 1

The _ is the source of all fetal oxygenation.
A. Placenta
B. Umbilical cord
C. Mother
D. Amniotic fluid
C. Mother

FHTs with recurrent variable decelerations, no accelerations, and minimal variability would be categorized as
A. Category I
B. Category II
C. Category III
B. Category II

FHTs with absent variability and bradycardia would be categorized as
A. Category I
B. Category II
C. Category III
C. Category III

FHTs with moderate variability, no accelerations, and early decelerations would be categorized as
A. Category I
B. Category II
C. Category III
A. Category I

FHTs with accelerations, no decelerations, and minimal variability would be categorized as
A. Category I
B. Category II
C. Category III
B. Category II

Sinusoidal pattern is categorized as
A. Category I
B. Category II
C. Category III
B. Category III

FHTs with absent variability and no accelerations or decelerations would be categorized as
A. Category I
B. Category II
C. Category III
B. Category II

Absence of accelerations following fetal stimulation (i.e. scalp stimulation) is categorized as
A. Category I
B. Category II
C. Category III
B. Category II

FHTs with minimal variability, absent accelerations, and a 3-minute prolonged deceleration would be categorized as
A. Category I
B. Category II
C. Category III
B. Category II

FHTs with minimal variability and a baseline of 95bpm would be categorized as
A. Category I
B. Category II
C. Category III
B. Category II

FHTs with a baseline of 170bpm, moderate variability, and no accelerations or decelerations would be categorized as
A. Category I
B. Category II
C. Category III
B. Category II

FHTs with a baseline 135bpm, moderate variability, accelerations, and one late deceleration would be categorized as
A. Category I
B. Category II
C. Category III
B. Category II

A prolonged acceleration lasts greater than _ minutes and less than _ minutes.
A. 2; 10
B. 2; 20
C. 10; 20
D. 10; 20
A. 2; 10

_ FHR patterns are those associated with uterine contractions.
A. Periodic
B. Episodic
C. Recurrent
D. Irregular
A. Periodic

_ FHR patterns are those that are not associated with uterine contractions.
A. Periodic
B. Episodic
C. Recurrent
D. Irregular
B. Episodic

Which of the following is not a likely cause of a sinusoidal FHR pattern?
A. Chronic fetal bleeding
B. Fetal hypoxia or anemia
C. Triple screen positive for Trisomy 21
D. Fetal isoimmunization
C. Triple screen positive for Trisomy 21

Which of the following factors is not likely to cause uteroplacental insufficiency?
A. Late-term gestation
B. Preeclampsia
C. Gestational diabetes
D. Polyhydramnios
E. Maternal smoking or drug use
D. Polyhydramnios

Which of the following are considered determinants of fetal well-being? (Select all that apply).
A. Absence of decelerations in FHR
B. Palpation of fetal movement
C. Presence of accelerations in FHR
D. Moderate variability in FHR
E. Presence of early decelerations in second stage
C. Presence of accelerations in FHR
D. Moderate variability in FHR

When auscultation is used for fetal assessment during labor for a low-risk woman, the FHR should be auscultated in the first stage of labor every
A. 5 min
B. 15-30 min
C. 60 min
B. 15-30 min

For a low-risk woman in the second stage of labor, the FHR should be auscultated every
A. 5-15 min
B. 30 min
C. 60 min
A. 5-15 min

The normal FHR baseline
A. Decreases during labor
B. Fluctuates during labor
C. Increases during labor
B. Fluctuates during labor

Bradycardia in the second stage of labor following a previously normal tracing may be caused by fetal
A. Hypoxemia
B. Rotation
C. Vagal stimulation
C. Vagal stimulation

A likely cause of fetal tachycardia with moderate variability is
A. Fetal hypoxemia
B. Maternal fever
C. Vagal stimulation
B. Maternal fever

Reduction in FHR variability can result from
A. Fetal scalp stimulation
B. Medication administration
C. Vaginal examination
B. Medication administration

The primary goal in treatment for late decelerations is to
A. Correct cord compression
B. Improve maternal oxygenation
C. Maximize uteroplacental blood flow
C. Maximize uteroplacental blood flow

The most frequently observed type of FHR deceleration is
A. Early
B. Late
C. Variable
C. Variable

Amnioinfusion may be useful in alleviating recurrent decelerations that are
A. Early
B. Late
C. Variable
C. Variable

Findings indicative of progressive fetal hypoxemia are
A. Late decelerations, moderate variability, stable baseline rate
B. Prolonged decelerations recovering to baseline and moderate variability
C. Loss of variability and recurrent late or variable decelerations
C. Loss of variability and recurrent late or variable decelerations

Clinically significant fetal metabolic academia is indicated by an arterial cord gas pH of less than or equal to 7.10 and a base deficit of
A. 3
B. 6
C. 12
C. 12

Fetal bradycardia can result during
A. The sleep state
B. Umbilical vein compression
C. Vagal stimulation
C. Vagal stimulation

While caring for a 235-lb laboring woman who is HIV-seropositive, the external FHR tracing is difficult to obtain. An appropriate nursing action would be to
A. Apply a fetal scalp electrode
B. Auscultate for presence of FHR variability
C. Notify the attending midwife or physician
C. Notify the attending midwife or physician

FHR decelerations that are benign and do not require intervention are
A. Early
B. Late
C. Variable
A. Early

FHR decelerations that results from decreased uteroplacental blood flow are
A. Early
B. Late
C. Variable
B. Late

FHR decelerations that results from umbilical cord compression are
A. Early
B. Late
C. Variable
C. Variable

An FHR pattern associated with severe fetal anemia is
A. Lambda
B. Saltatory
C. Sinusoidal
C. Sinusoidal

A workup for maternal systemic lupus erythematosus would likely be ordered in the presence of
A. Complete fetal heart block
B. Premature ventricular contractions
C. Fetal supraventricular tachycardia
A. Complete fetal heart block

Which IV fluid is most appropriate for maternal administration for intrauterine resuscitation?
A. Lactated Ringer’s solution
B. D5L/R
C. Normal saline
C. Normal saline

The position that best promotes maternal-fetal exchange is
A. Left lateral
B. Right lateral
C. Either right or left lateral
C. Either right or left lateral

The most appropriate equipment for administration of maternal oxygen for intrauterine resuscitation at 10 L/min is a
A. Nasal cannula
B. Simple face mask
C. Nonrebreather face mask
C. Nonrebreather face mask

Accurate determination of baseline rate requires
A. At least 2 contiguous minutes of FHR in a 10-min window
B. Evaluation of the FHR over at least a 10-min window
C. Averaging the FHR over 30 min
B. Evaluation of the FHR over at least a 10-min window

An EFM tracing with absent variability and no decelerations would be classified as
A. Category I
B. Category II (indeterminate)
C. Category III
B. Category II (indeterminate)

An EFM tracing with absent variability and intermittent late decelerations would be classified as
A. Category I
B. Category II
C. Category III
B. Category II

Interpretation and classification of FHR patterns are based on predictability of fetal status
A. At birth
B. At the time the pattern is observed
C. Over the previous hour
B. At the time the pattern is observed

Amnioinfusion is an appropriate measure for
A. Thick, meconium-stained fluid
B. Oligohydramnios
C. Recurrent variable decelerations unresolved by position changes
C. Recurrent variable decelerations unresolved by position changes

Baroreceptors respond to changes in fetal
A. Blood pressure
B. Oxygen status
C. Acid-base status
A. Blood pressure

Fetal scalp stimulation is appropriate in the context of
A. Minimal variability
B. Prolonged deceleration
C. Bradycardia
A. Minimal variability

Maternal oxygen administration is appropriate in the context of
A. Recurrent variable decelerations/moderate variability
B. Intermittent late decelerations/minimal variability
C. Prolonged decelerations/moderate variability
B. Intermittent late decelerations/minimal variability

A preterm fetus
A. Is more susceptible to hypoxic insults during labor than the term fetus
B. Requires internal monitoring if oxytocin is used for labor induction or augmentation
C. Should be born via cesarean section unless there are maternal contraindications
A. Is more susceptible to hypoxic insults during labor than the term fetus

Oxygen is transferred from the mother to the fetus via the placenta through
A. Active transport
B. Passive diffusion
C. Facilitated diffusion
B. Passive diffusion

Resting tone and intensity of uterine contractions cannot be assessed by
A. External tocodynamometer
B. Manual palpation
C. Intrauterine pressure catheter
A. External tocodynamometer

The FHR characteristic most predictive of a well-oxygenated baby at the time observed is
A. Moderate variability
B. Stable baseline rate
C. Absence of decelerations
A. Moderate variability

In the context of hypoxemia, fetal blood flow is shifted to the
A. Brain
B. Liver
C. Lungs
A. Brain

Baroreceptor-mediated decelerations are
A. Early
B. Late
C. Variable
C. Variable

The primary goal in the treatment of variable decelerations is to
A. Correct umbilical cord compression
B. Improve maternal oxygenation
C. Maximize blood flow to the uterus
A. Correct umbilical cord compression

Umbilical artery gas results reflect the status of the
A. Mother
B. Fetus
C. Placenta
B. Fetus

An appropriate initial treatment for recurrent late decelerations with moderate variability during first stage labor is
A. Amnioinfusion
B. Maternal repositioning
C. Oxygen at 10L per nonrebreather face mask
B. Maternal repositioning

Most fetal dysrhythmias are not life-threatening, except for _, which may lead to fetal congestive heart failure.
Supraventricular tachycardia

Medications, prematurity, fetal sleep, fetal dysrhythmia, anesthetic agents, or cardiac anomalies may result in _ variability.
Decreased

In the presence of late or variable decelerations, two parameters that indicate adequate fetal oxygenation are _ and _.
Moderate variability; normal baseline rate

To correctly interpret a baseline FHR as tachycardic or bradycardic, the rate must persist for a minimum of _ min.
10

In assessing fetal well-being, the most important characteristic of the FHR is
A. Rate
B. Variability
C. Presence of accelerations
D. Absence of decelerations
B. Variability

Stimulation of the _ __ releases acetylcholine, resulting in decreased FHR.
Parasympathetic nervous system

The _ __ maintains transmission of beat-to-beat variability.
Parasympathetic nervous system

Stimulation of the _ __ releases catecholamines, resulting in increased FHR.
Sympathetic nervous system

Stimulation of _ results in abrupt decreases in FHR, CO, and BP.
Baroreceptors

Baroreceptors influence _ decelerations with moderate variability.
Variable

Following an ultrasound which revealed decreased amniotic fluid, a woman at term is admitted in early labor. It should be recognized that oligohydramnios often results in fetal heart rate decelerations that are
A. Late in onset or occur after the peak of the contraction
B. Synchronous with that of the contraction
C. Varied in depth and duration
C. Varied in depth and duration

In comparing early and late decelerations, a distinguishing factor between the two is
A. Onset time to the nadir of the deceleration
B. The number of decelerations that occur
C. Timing in relation to contractions
C. Timing in relation to contractions

The underlying cause of early decelerations is decreased
A. Baroceptor response
B. Increased peripheral resistance
C. Vagal reflex
C. Vagal reflex

Glucose is transferred across the placenta via _ _.
Facilitated diffusion

Oxygen, carbon dioxide, water, electrolytes, urea, uric acid, fatty acids, fat-soluble vitamins, narcotics barbiturates, anesthetics, and antibiotics are transferred across the placenta via _ _.
Simple (passive) diffusion

Amino acids, water-soluble vitamins, calcium, phosphorus, iron, and iodine are transferred across the placenta via _ _.
Active transport

Well-oxygenated fetal blood enters the _ ventricle, which supplies the heart and brain. Less-oxygenated blood enters the __ ventricle, which supplies the rest of the body.
Left; right

The _ _ allows relatively well-oxygenated blood to enter the fetal heart directly, bypassing the liver.
Ductus venosus

The _ _ allows blood to bypass the lungs, flowing from the pulmonary artery to the aorta.
Ductus arteriosus

The _ _ is the shunt that bypasses the fetal lungs, moving blood from the right atrium to the left atrium.
Foramen ovale

Fetal blood has a _ hemoglobin concentration compared to adults.
A. Higher
B. Lower
A. Higher

Greater oxygen-carrying capacity

Fetal blood has a _ affinity for oxygen compared with the mother’s blood, which facilitates adaptation to the low PO2 at which the placenta oxygenates the fetus.
A. Higher
B. Lower
A. Higher

The fetus has a _ cardiac output and heart rate than the adult, resulting in rapid circulation.
A. Higher
B. Lower
A. Higher

As the FHR increases, the myocardium consumes _ oxygen.
A. More
B. Less
A. More

The fetal spiral electrode measures the
A. Peaks of the Doppler waveforms
B. R to R intervals of the fetal heart
C. ST segments of the fetal ECG
B. R to R intervals of the fetal heart

A woman who is admitted for an induction of labor with oxytocin is questioning the need for continuous electronic fetal monitoring. The appropriate response to this woman is
A. Hospital policy requires that all patients have continuous electronic fetal monitoring.
B. I would like to answer your questions about continuous monitoring and give you some information about why continuous monitoring is recommended.
C. Nurses have more training with continuous electronic fetal monitoring than intermittent auscultation so it is safer for you and your baby to use this method.
B. I would like to answer your questions about continuous monitoring and give you some information about why continuous monitoring is recommended.

The greater affinity that fetal hemoglobin has for oxygen allows for
A. Easier release of oxygen to the tissues
B. Greater binding of oxygen
C. Stimulation of erythropoietin release
B. Greater binding of oxygen

The process by which oxygen and carbon dioxide pass from a region of higher concentration to one of a lower concentration is called
A. Active transport
B. Simple diffusion
C. Facilitated diffusion
B. Simple diffusion

In the healthy fetus, blood flows from the right atrium to the left atrium through the
A. Ductus arteriosus
B. Ductus venosus
C. Foramen ovale
C. Foramen ovale

In the healthy fetus, the umbilical cord enters the fetal abdomen and bypasses the liver through the
A. Ductus arteriosus
B. Ductus venosus
C. Foramen ovale
B. Ductus venosus

Which statement best describes the relationship between maternal and fetal hemoglobin levels?
A. Fetal hemoglobin is higher than maternal hemoglobin
B. Maternal hemoglobin is higher than fetal hemoglobin
C. Maternal and fetal hemoglobin are the same
A. Fetal hemoglobin is higher than maternal hemoglobin

Baroreceptors respond mainly to changes in
A. Blood pressure
B. Hormonal changes
C. Oxygen and carbon dioxide levels
A. Blood pressure

Stimulation of the sympathetic nervous system causes the fetal heart rate to
A. Decrease
B. Increase
C. Remain the same
B. Increase

Which statement describes normal uterine activity?
A. Frequency of 1-1/2 to 2 minutes
B. Intensity of 90 mmHg early in labor
C. Resting tone less than 20-25 mmHg
C. Resting tone less than 20-25 mmHg

Greater than 5 contractions in 10 minutes averaged over 30 minutes indicates
A. Excessive uterine activity
B. Hyperstimulation
C. Tachysystole
C. Tachysystole

A 36 week gestation patient is brought to triage by squad after an MVA on her back. She is not bleeding and denies pain. She is not short of breath, but c/o dizziness and nausea since they put her on the gurney. The most likely cause is
A. Abruptio placenta
B. Preterm labor
C. Supine hypotension
C. Supine hypotension

One compensatory mechanism that helps maintain oxygen availability to the fetus during maternal exercise is
A. A decrease in maternal hematocrit
B. Transient increase in uterine blood flow
C. An increase in uterine oxygen uptake
C. An increase in uterine oxygen uptake

When the hydrogen ion content in the blood rises, the pH
A. Lowers
B. Neutralizes
C. Rises
A. Lowers

A woman receives terbutaline for an external version. You may expect what on the fetal heart tracing?
A. Decrease in variability
B. Increase in baseline
C. No change
B. Increase in baseline

What affect does magnesium sulfate have on the fetal heart rate?
A. Decreases variability
B. Increases variability
C. No change
A. Decreases variability

NICHD guidelines apply only to
A. Intrapartum patients
B. Internal monitoring of fetal heart rate
C. Interpreting tracings of good quality
C. Interpreting tracings of good quality

According to NICHD terminology, variability can be accurately accessed
A. Visually, by determining the number of R to R intervals in a one minute period
B. Visually, by determining the amplitude of the FHR change in bpm from the baseline
C. Only when a fetal spiral electrode is in place
B. Visually, by determining the amplitude of the FHR change in bpm from the baseline

When looking at the fetal heart rate, the most important characteristic to determine the absence of metabolic acidemia is
A. Absence of late decelerations
B. Baseline rate within normal range
C. Presence of moderate variability
C. Presence of moderate variability

Sinusoidal pattern can be documented when
A. Cycles are 4-6 beats per minute in frequency
B. The pattern lasts 20 minutes or longer
C. There is moderate or minimal variability
B. The pattern lasts 20 minutes or longer

Vagal stimulation would be manifested as what type of fetal heart rate pattern?
A. Acceleration
B. Early deceleration
C. Tachycardia
B. Early deceleration

Which fetal monitoring pattern is characteristic of cephalopelvic disproportion, especially when seen at the onset of labor?
A. Early deceleration
B. Late deceleration
C. Variable deceleration
A. Early deceleration

A risk of amnioinfusion is
A. Prolonged labor
B. Uterine overdistension
C. Water intoxication
B. Uterine overdistension

A fetal heart rate pattern that the NICHD has identified as predictive of current or impending fetal asphyxia so severe that the fetus is at risk of neurologic damage or death is
A. Baseline tachycardia with absent variability
B. Minimal baseline variability with recurrent late decelerations
C. Recurrent late or variable decelerations with absent variability
C. Recurrent late or variable decelerations with absent variability

A fetal heart rate pattern that can occur when there is a prolapsed cord is
A. Marked variability
B. Prolonged decelerations
C. Tachycardia
B. Prolonged decelerations

The patient is in early labor with pitocin at 8 mu/min, and FHR is Category I. In the next 15 minutes, there are 18 uterine contractions. Recommended management is to
A. Address contraction frequency by reducing pitocin dose
B. Continue to increase pitocin as long as FHR is Category I
C. Turn the patient on her side and initiate an IV fluid bolus
C. Turn the patient on her side and initiate an IV fluid bolus

A woman at 38 weeks gestation is in labor. The labor has been uneventful, and the fetal heart tracings have been normal. Spontaneous rupture of membranes occurs; fetal heart rate drops to 90 beats per minute for four minutes and then resumes a normal pattern. The most likely etiology for this fetal heart rate change is
A. Abnormal fetal presentation
B. Impaired placental circulation
C. Possible cord compression
C. Possible cord compression

A 42 week gestation woman has been diagnosed with oligohydramnios. Based on this, a FHR change that can be expected is
A. Late deceleration
B. Minimal variability
C. Variable deceleration
C. Variable deceleration

Mono-mono zygotic twins are prone to what type of deceleration during labor?
A. Early
B. Late
C. Variable
C. Variable

During labor, the recommended fetal heart rate assessment interval for auscultation is every
A. 15-30 minutes in the active phase of the first stage and every 5-15 minutes in second stage
B. 15 minutes no matter what stage of labor
C. 60 minutes in the active phase of the first stage and every 30 minutes in second stage
A. 15-30 minutes in the active phase of the first stage and every 5-15 minutes in second stage

What fetal heart rate characteristics can be determined with auscultation?
A. Baseline
B. Early decelerations
C. Variability
A. Baseline

When auscultating the fetal heart rate, the provider/nurse should simultaneously assess the maternal
A. Blood pressure
B. Pain level
C. Radial pulse
C. Radial pulse

A woman has 10 fetal movements in one hour. This is considered what kind of movement?
A. Decreased
B. Excessive
C. Normal
C. Normal

If the pH is low, what other blood gas parameter is used to determine if the acidosis is respiratory or metabolic?
A. HCO3
B. PCO2
C. PO2
B. PCO2

The following cord blood gasses are consistent with: pH 7.10, pCO2 70, pO2 25, base excess -10
A. Metabolic acidosis
B. Mixed acidosis
C. Respiratory acidosis
C. Respiratory acidosis

A fetus at 36 weeks receives a biophysical profile (BBP) score of 6. The amniotic fluid was scored as normal. The expected management is
A. Immediate delivery
B. Repeat the test in 24 hours
C. Schedule the next test in one week
B. Repeat the test in 24 hours

A woman with gestational diabetes is 38 weeks gestation. Her Biophysical Profile score is 4. This indicates need for
A. Follow up in one week
B. Induction of labor
C. Emergent cesarean section
B. Induction of labor

A 35 week gestation fetus is having an NST. The fetal heart rate baseline is 130 bpm. The nurse is using vibroacoustic stimulation to reduce the length of time needed to obtain the NST. Fetal well-being requires
A. 1 acceleration to 145 bpm
B. 2 accelerations to 140 bpm
C. 2 accelerations to at least 145 bpm
C. 2 accelerations to at least 145 bpm

A woman desires a natural childbirth. The nurse puts pressure on her to get an epidural. This is an example of going against which ethical principle?
A. Autonomy
B. Beneficence
C. Justice
A. Autonomy

One characteristic of a high reliability perinatal unit is
A. Alarms can only be called by unit leaders
B. Reliance on memorization of protocols
C. The organization creates a safety-oriented culture
C. The organization creates a safety-oriented culture

One example of evidence-based practice related to fetal monitoring is
A. Diagnosis of uterine rupture with an IUPC
B. Intermittent auscultation for the low-risk patient
C. Using electronic fetal monitoring to prevent cerebral palsy
B. Intermittent auscultation for the low-risk patient

Supporting the parents decision to choose no extraordinary measures on their baby who is about to deliver at 24 3/7 weeks gestation despite the nurses personal opinion is an example of
A. Autonomy
B. Beneficence
C. Non-maleficence
A. Autonomy

_ primarily function(s) to regulate respiratory activity and control circulation by responding to changes in arterial PO2, PCO2, and acid-base balance.
A. Baroreceptors
B. Chemoreceptors
C. Cardioregulatory center
B. Chemoreceptors

_ are protective stretch receptors located in the aortic arch and the carotid sinuses at the bifurcation of the external and internal carotid arteries.
A. Baroreceptors
B. Chemoreceptors
C. Arteries
A. Baroreceptors

_ respond to increases in fetal arterial blood pressure by detecting the amount of stretch and sending impulses via the vagus nerve to the midbrain, decreasing FHR, CO, and BP.
A. Baroreceptors
B. Chemoreceptors
C. Veins
A. Baroreceptors

As a contraction beings, partial umbilical cord compression causes occlusion of the low-pressure vein and decreased return of blood to the fetal heart, resulting in decreased CO, hypotension, and a compensatory FHR _.
A. Acceleration
B. Early deceleration
C. Late deceleration
D. Variable deceleration
A. Acceleration

With complete umbilical cord occlusion, the two umbilical arteries also become occluded, resulting in sudden fetal hypertension, stimulation of the baroreceptors, and a sudden _ in FHR.
A. Increase
B. Decrease
B. Decrease

Central are located in the medulla oblongata; peripheral are found in the carotid sinuses and aortic arch.
A. Baroreceptors
B. Chemoreceptors
B. Chemoreceptors

When a fetus is stressed, catecholamine release (epinephrine, norepinephrine) occurs from the medulla oblongata, shunting blood _ the brain, heart, and adrenal glands.
A. Toward
B. Away from
A. Toward

Because stroke volume in the fetus does not fluctuate significantly, fetal cardiac output is _ fetal heart rate.
A. Greater than
B. Less than
C. Approximately equal to
C. Approximately equal to

In _ sleep, the fetus may be observed to have infrequent or absent body movements, normal FHR baseline, minimal variability, and a nonreactive NST, but may respond to external stimuli. Such typically occurs by 28 to 32 weeks’ gestation.
A. REM (active sleep state)
B. Non-REM (quiet sleep state)
B. Non-REM (quiet sleep state)

Place the following steps for performing Leopold’s maneuver in the appropriate order.

  1. Assess location of fetal back
  2. Determine the descent of the presenting part
  3. Identify the presenting part
  4. Assess part of the fetus in the upper uterus
    A. 1, 3, 2, 4
    B. 2, 4, 1, 2
    C. 4, 1, 3, 2
    D. 4, 3, 2, 1
    C. 4, 1, 3, 2

When the internal mode of monitoring (FSE, IUPC) is used, the information obtained on the uterine activity panel on the fetal monitor should be validated by the clinician by
A. Asking the patient to report when she is feeling a contraction
B. Ultrasound imaging
C. Palpation
C. Palpation

T/F: Use of a fetoscope for intermittent auscultation of the fetal heart rate may be used to detect baseline, rhythm, changes from baseline, and presence of an irregular rhythm.
True

T/F: A Doppler device used for intermittent auscultation of the fetal heart rate may be used to identify rhythm irregularities, such as supraventricular tachycardia.
False

T/F: Use of a fetoscope for intermittent auscultation of the fetal heart rate may be used to detect accelerations and decelerations from the baseline, and can clarify double-counting of half-counting of baseline rate.
True

T/F: Auscultation may be used to detect baseline variability and discriminate FHR deceleration patterns.
False

_ represents increased sympathetic or decreased parasympathetic autonomic tone.
A. Bradycardia
B. Tachycardia
B. Tachycardia

T/F: In the context of moderate variability, late decelerations are considered neurogenic in origin and are typically amenable to intrauterine resuscitation techniques directed towards maximizing uterine blood flow.
True

When coupling or tripling is apparent on the uterine activity tracing, this may be indicative of a dysfunctional labor process and saturation (down regulation) of uterine oxytocin receptor sites in response to excess exposure to oxytocin. Which of the following interventions would be most appropriate?
A. Normal response; continue to increase oxytocin titration
B. Turn patient on side
C. Decrease or discontinue oxytocin infusion
C. Decrease or discontinue oxytocin infusion

The most common tachyarrhythmia in fetuses, supraventricular tachycardia, typically occurs at a rate of _ to _ bpm with minimal or absent variability.
A. 160-200
B. 200-240
C. 240-260
C. 240-260

In a patient with oxytocin-induced tachysystole with normal fetal heart tones, which of the following should be the nurse’s initial intervention?
A. Assist the patient to lateral position
B. Discontinue Pitocin
C. Administer IV fluid bolus
A. Assist the patient to lateral position

In a patient with oxytocin-induced tachysystole with indeterminate or abnormal fetal heart tones, which of the following should be the nurse’s initial intervention?
A. Assist the patient to lateral position
B. Discontinue Pitocin
C. Administer IV fluid bolus
B. Discontinue Pitocin

Fetal hypoxia and acidemia are demonstrated by pH < _ and base excess < _.
< 7.15; < -8

T/F: Amnioinfusion is an appropriate intervention to attempt to resolve patterns of moderate to severe late decelerations.
False

T/F: Amnioinfusion is an appropriate intervention to attempt to resolve patterns with absent variability.
False

T/F: Amnioinfusion may be an appropriate intervention for patients with oligohydramnios in the prevention of the development of variable decelerations.
False

Not for prevention

T/F: Meconium-stained amniotic fluid is an indication for amnioinfusion.
False

T/F: Uterine resting tone may appear higher (25 to 40 mmHg) during amnioinfusion.
True

_ denotes a decrease in oxygenation of the fetal tissues.
A. Hypercapnia
B. Hypoxia
C. Hypoxemia
B. Hypoxia

_ denotes a decrease in oxygen content of the fetal blood.
A. Hypercapnia
B. Hypoxia
C. Hypoxemia
C. Hypoxemia

_ denotes an increase in carbon dioxide in the fetal blood.
A. Hypercapnia
B. Hypoxia
C. Hypoxemia
A. Hypercapnia

_ denotes an increase in hydrogen ions in the fetal blood.
A. Acidosis
B. Acidemia
C. Hypercapnia
B. Acidemia

_ denotes an increase in hydrogen ions in the fetal tissues.
A. Acidosis
B. Acidemia
C. Hypercapnia
A. Acidosis

occurs when there is low bicarbonate (base excess) in the presence of normal pressure of carbon dioxide (PCO2) values.
A. Metabolic acidosis
B. Respiratory acidosis
C. Metabolic alkalosis
A. Metabolic acidosis

occurs when there is high PCO2 with normal bicarbonate levels.
A. Metabolic acidosis
B. Respiratory acidosis
C. Metabolic alkalosis
B. Respiratory acidosis

occurs when the HCO3 concentration is lower than normal.
A. Base deficit
B. Base excess
C. Metabolic acidosis
A. Base deficit

occurs when the HCO3 concentration is higher than normal.
A. Base deficit
B. Base excess
C. Metabolic acidosis
B. Base excess

_ is defined as the energy-consuming process of metabolism.
Anabolism

_ is defined as the energy-releasing process of metabolism.
Catabolism

T/F: It is an appropriate intervention to perform fetal scalp stimulation during a deceleration.
False

Normal oxygen saturation for the fetus in labor is % to %.
30% to 65%

The normal mean value range for arterial pH is
A. 7.0-7.20
B. 7.20-7.29
C. 7.29-7.39
B. 7.20-7.29

The normal mean value range for arterial PCO2 is
A. 22-24
B. 35-40
C. 49-56
C. 49-56

The normal mean value range for arterial bicarbonate (HCO3) is
A. 22-24
B. 35-40
C. 49-56
A. 22-24

The normal mean value range for arterial base deficit is
A. 2.7-8.3
B. -3.0-2.5
C. 4.2-12.9
A. 2.7-8.3

The normal mean value range for arterial PO2 is
A. 22-24
B. 15-24
C. 49-56
B. 15-24

pH 7.05
PO2 21
PCO2 72
HCO3 24
Base excess -12

A. Metabolic acidosis
B. Respiratory acidosis
C. Mixed acidosis
B. Respiratory acidosis

pH 7.0
PO2 18
PCO2 54
HCO3 20
Base deficit 14

A. Metabolic acidosis
B. Respiratory acidosis
C. Mixed acidosis
A. Metabolic acidosis

pH 7.02
PO2 17
PCO2 72
HCO3 19
Base deficit 16

A. Metabolic acidosis
B. Respiratory acidosis
C. Mixed acidosis
C. Mixed acidosis

Which is a correct description of daily fetal movement counting?
A. Counting should not be started by low-risk mothers until 38 weeks of gestation
B. Fetal movement counting is not reliable because the methods of counting vary
C. The mother counts the number of movements over a specified length of time
C. The mother counts the number of movements over a specified length of time

The BPP includes the assessment of FHR along with what other four components?
A. Amniotic fluid, fetal breathing, fetal movement, fetal tone
B. Amniotic fluid, fetal anomalies, fetal breathing, fetal movement
C. Estimated fetal weight, fetal breathing, fetal movement, placental grading
A. Amniotic fluid, fetal breathing, fetal movement, fetal tone

Which of the following might indicate a potential for chronic fetal hypoxemia?
A. Decreased amniotic fluid volume
B. Increased amniotic fluid volume
C. Intrauterine growth restriction
C. Intrauterine growth restriction

With the finding of a single umbilical artery, what would you expect to observe with Doppler flow studies?
A. Decreased blood perfusion from the fetus to the placenta
B. Decreased blood perfusion from the placenta to the fetus
C. Homeostatic dilation of the umbilical artery
A. Decreased blood perfusion from the fetus to the placenta

Two umbilical arteries flow from the fetus to the placenta

Primary benefits associated with the use of standardized terminology for fetal heart monitoring interpretation include
A. Enhanced communication among health care providers and promotion of patient safety
B. Increased nursing time at the bedside and enhanced patient satisfaction
C. Increased likelihood of correctly diagnosing fetal acidosis during labor
A. Enhanced communication among health care providers and promotion of patient safety

A patient presents with a small amount of thick dark blood clots who denies pain and whose abdomen is soft to the touch. Which component of oxygen transport to the fetus could potentially be compromised by this bleeding?
A. Affinity
B. Saturation
C. Delivery
C. Delivery

What is the most appropriate physiologic goal for a patient whose tracing reveals a sinusoidal baseline?
A. Maximize umbilical circulation
B. Maximize uteroplacental circulation
C. Reduce uterine activity
B. Maximize uteroplacental circulation

Compromised oxygenation – maximize uteroplacental circulation to promote perfusion and oxygenation

When recurrent late decelerations are occurring, the correct physiologic interpretation of this event is
A. Fetal myocardial acidosis is occurring
B. Fetal hypoxemia may be present
C. The fetus has oxygen reserves
B. Fetal hypoxemia may be present

Which intrinsic homeostatic response is the fetus demonstrating when abrupt variable decelerations are present?
A. Baroreceptor
B. Catecholamine
C. Sympathetic
A. Baroreceptor

An amnioinfusion is intended to relieve which extrinsic factor that compromises oxygen transport?
A. Excessive uterine compression
B. Structural abnormalities of the placenta
C. Umbilical cord compression
C. Umbilical cord compression

The nurse wants to document her conversation with the attending physician during an emergent situation. The best approach to documentation would be to
A. Continue providing care for the patient and write a late entry summarizing the conversation after the cesarean is completed
B. Enter an objective transcription of the conversation in the electronic record while colleagues are preparing the patient for surgery
C. Report the conversation to the charge nurse, who can make an entry in the medical record
A. Continue providing care for the patient and write a late entry summarizing the conversation after the cesarean is completed

An infant was delivered via cesarean. Umbilical cord blood gases were: pH 6.88, PCO2 114, PO2 10, bicarbonate 15, base excess (-) 20. The initial neonatal hemocrit was 20% and the hemoglobin was 8.

Which interpretation of these umbilical cord and initial neonatal blood results is correct?
A. Base buffers have been used to maintain oxygenation
B. The mother was probably hypoglycemic
C. The neonate is anemic
C. The neonate is anemic

An infant was delivered via cesarean. Umbilical cord blood gases were: pH 6.88, PCO2 114, PO2 10, bicarbonate 15, base excess (-) 20. The initial neonatal hemocrit was 20% and the hemoglobin was 8.

These umbilical cord blood gases indicate
A. Asphyxia related to umbilical and placental abnormalities
B. Hypoxia related to neurological damage
C. Mixed acidosis
C. Mixed acidosis

An infant was delivered via cesarean. Umbilical cord blood gases were: pH 6.88, PCO2 114, PO2 10, bicarbonate 15, base excess (-) 20. The initial neonatal hemocrit was 20% and the hemoglobin was 8.

Which phrase best describes acidemia?
A. A decrease of oxygen concentration in the blood
B. An increase in hydrogen ions in the blood
C. An increase of hydrogen ions in the tissues
B. An increase in hydrogen ions in the blood

Which compromise in fetal oxygenation could be a result of a post-date pregnancy?
A. Increased saturation capacity
B. Increased fetal oxygen affinity
C. Decreased placental perfusion
C. Decreased placental perfusion

What are the possible implications of an AFI of 3 cm for labor?
A. An amnioinfusion will be needed
B. Increased risk of uterine hyperstimulation
C. Potential umbilical cord compression
C. Potential umbilical cord compression

Which is a characteristic of a negative CST?
A. Absence of late decelerations
B. Recurrent late decelerations
C. Reactive FHR tracing
A. Absence of late decelerations

Which FHR tracing features must be assessed to distinguish arrhythmias from artifact?
A. Shape and regularity of the spikes
B. Spikes and variability
C. Spikes and baseline
A. Shape and regularity of the spikes

How might a fetal arrhythmia affect fetal oxygenation?
A. By increasing fetal oxygen affinity
B. By increasing sympathetic response
C. By reducing fetal perfusion
C. By reducing fetal perfusion

Which medication is used to treat fetal arrhythmias?
A. Digoxin
B. Labetolol
C. Nifedipine
A. Digoxin

Inotropic – promotes regular and effective cardiac contraction

Fetal hydrops may present on ultrasound as fetal scalp edema and increased abdominal fluid as a results of
A. An increase in gestational age
B. Congestive heart failure
C. Sustained oligohydramnios
B. Congestive heart failure

What might increase fetal oxygen consumption?
A. Hyperthermia
B. Umbilical cord compression
C. Uterine tachysystole
A. Hyperthermia

Increases metabolism and oxygen consumption

T/F: Adequacy of uterine resting tone measured by an IUPC should be validated by palpation.
True

Which assessment or intervention would be least appropriate in a patient whose FHR tracing revealed tachycardia and a prolonged deceleration?
A. Change maternal position to right lateral
B. Further assess fetal oxygenation with scalp stimulation
C. Perform a vaginal exam to assess fetal descent
B. Further assess fetal oxygenation with scalp stimulation

Only used with normal baseline rate and never during decels; not an intervention

Which of the following pieces of information would be of highest priority to relay to the neonatal team as they prepare for an emergency cesarean delivery?
A. FHR arrhythmia, meconium, length of labor
B. Gestational age, meconium, arrhythmia
C. Gravidity & parity, gestational age, maternal temperature
B. Gestational age, meconium, arrhythmia

What characteristics are necessary for a reactive NST in a G3P0 patient who is 25 6/7 weeks’ gestation?
A. Accelerations of 15bpm above the baseline lasting at least 15 seconds above the baseline
B. Accelerations of 10bpm above the baseline lasting at least 10 seconds above the baseline
C. Any acceleration greater than 10bpm above the baseline
B. Accelerations of 10bpm above the baseline lasting at least 10 seconds above the baseline

How long may a standard NST be extended in a term gestation if reactivity is not initially demonstrated?
A. From the initial 20 min to 40 min
B. From the initial 20 min to 60 min
C. From the initial 30 min to 60 min
A. From the initial 20 min to 40 min

Accounts for fetal sleep

Which is the most appropriate application of vibroacoustic stimulation (fetal acoustic stimulation) in a standard NST?
A. The device is placed at the top of the maternal fundus
B. The mother uses a marker button to document the fetal movement response
C. The stimulation is applied after a baseline is established
C. The stimulation is applied after a baseline is established

Which of the following characteristics are most common in the preterm fetus?
A. Increased baseline rate and prolonged accelerations
B. Increased baseline rate and variable decelerations
C. Prolonged accelerations and variable decelerations
B. Increased baseline rate and variable decelerations

As well as decreased variability

What typical characteristics of preterm uterine activity may be present in a patient experiencing preterm labor?
A. An irritable uterus with wandering hypertonus
B. Low-amplitude high-frequency contractions
C. Tetanic contractions
B. Low amplitude high-frequency contractions

Which medications used with preterm labor can affect the FHR characteristics?
A. Terbutaline and antibiotics
B. Betamethasone and terbutaline
C. Antibiotics and narcotics
B. Betamethasone and terbutaline

What characterizes a preterm fetal response to stress?
A. More frequently occurring late decelerations
B. More frequently occurring prolonged decelerations
C. More rapid deterioration from Category I to Category II or III
C. More rapid deterioration from Category I to Category II or III

More likely to be subjected to hypoxia

Clinical decision-making at the bedside should include:
A. Communication with the primary care provider only during a crisis
B. Integration of physiologic concepts with maternal-fetal assessment findings
C. Reliance primarily on technology over bedside assessments
B. Integration of physiologic concepts with maternal-fetal assessment findings

In the United States, the paper speed on the monitor is set at
A. 1 cm/min
B. 2 cm/min
C. 3 cm/min
C. 3 cm/min

A woman being monitored externally has a suspected fetal arrhythmia. The most appropriate action is to
A. Insert a spiral electrode and turn off the logic
B. Turn the logic on if an external monitor is in place
C. Use a Doppler to listen to the ventricular rate
A. Insert a spiral electrode and turn off the logic

The ultrasound transducer on the electronic fetal monitor measures the
A. Electrical signal of the fetal heart
B. Mechanical movements of the fetal heart reflected off of sound waves
C. R-to-R intervals of the fetal heart
B. Mechanical movements of the fetal heart reflected off of sound waves

The purpose of autocorrelation in external monitoring is to
A. Compare incoming waveforms for comparison
B. Decrease signal to noise levels
C. Distinguish fetal from maternal HR
A. Compare incoming waveforms for comparison

The area of maximum intensity of the FHR is usually the fetal
A. Back
B. Chest
C. Umbilicus
A. Back

Palpating the uterus is best performed by using the
A. Back of the hand
B. Fingertips
C. Palm
B. Fingertips

What is the most sensitive method of assessing uterine activity?
A. IUPC
B. Manual palpation
C. Maternal perception
A. IUPC

In comparison to maternal blood, the affinity of fetal blood for oxygen is
A. Higher
B. Lower
C. The same
A. Higher

The process that requires energy to accomplish the passage of substances within the intervillous space is
A. Active transport
B. Diffusion
C. Facilitated diffusion
A. Active transport

The umbilical vein carries
A. Carbon dioxide from the fetus back to the placenta
B. Deoxygenated blood from the fetus to the placenta
C. Oxygenated blood from the placenta to the fetus
C. Oxygenated blood from the placenta to the fetus

Fetal hypoxia is best described as a condition of
A. Decreased oxygen in the blood
B. Decreased oxygen in the tissue
C. Increased hydrogen ions in the blood
B. Decreased oxygen in the tissue

Stimulation of the parasympathetic nervous system causes the FHR to
A. Decrease
B. Increase
C. Remain the same
A. Decrease

Chemoreceptors respond mainly to changes in
A. Blood pressure
B. Hormonal levels
C. Oxygen and carbon dioxide levels
C. Oxygen and carbon dioxide levels

The fetus responds to a significant drop of PO2 by
A. Increasing O2 consumption
B. Reducing lactic acid production
C. Shifting blood to vital organs
C. Shifting blood to vital organs

Activation of fetal chemoreceptors results in which FHR change?
A. Decrease
B. Increase
C. No change
A. Decrease

Tachycardia is associated with increased
A. Parasympathetic tone
B. Sympathetic tone
C. Vagal response
B. Sympathetic tone

A patient receiving oxytocin has 17 contractions in 30 minutes. According to the NICHD guidelines, this is called
A. Hyperstimulation
B. Hypertonus
C. Tachysystole
C. Tachysystole

Maternal supine hypotension is caused mainly by compression of the
A. Inferior vena cava
B. Spiral arteries
C. Uterine vessels
A. Inferior vena cava

Which factor influences blood flow to the uterus?
A. Fetal arterial pressure
B. Intervillous space flow
C. Maternal arterial vasoconstriction
C. Maternal arterial vasoconstriction

Betamethasone given to the mother can transiently affect the FHR by
A. Decreasing variability
B. Increasing variability
C. Lowering the baseline
A. Decreasing variability

In a fetal heart rate tracing with marked variability, which of the following is likely the cause?
A. Recent ephedrine administration
B. Recent epidural placement
C. Fetal acidemia
A. Recent ephedrine administration

(T/F) The baseline can be established in a fetal heart rate tracing in which there is marked variability.
False

A fetal heart rate change that can be seen after administration of butorphanol (Stadol) is
A. Bradycardia
B. Marked variability
C. Sinusoidal-appearing
C. Sinusoidal-appearing

One fetal heart rate pattern that is associated with an abnormal acid-base status is
A. Minimal variability with no accelerations or decelerations
B. Recurrent variable decelerations with absent variability
C. Tachycardia with absent variability
B. Recurrent variable decelerations with absent variability

The NICHD definitions are applicable to
A. Antepartum only
B. Antepartum and intrapartum
C. Intrapartum only
B. Antepartum and intrapartum

A wandering FHR baseline may be indicative of
A. Fetal seizure activity
B. Impending fetal death
C. Maternal medication administration
B. Impending fetal death

The FHR pattern that is likely to be seen with maternal hypothermia is
A. Bradycardia
B. Marked variability
C. Tachycardia
A. Bradycardia

Baseline FHR variability is determined in what amount of time, excluding accelerations and decelerations?
A. 10 min
B. 20 min
C. 30 min
A. 10 min

One possible cause of a sinusoidal FHR pattern from fetal hypoxemia is
A. Fetal anemia
B. Fetal thumb sucking
C. Maternal administration of a narcotic
A. Fetal anemia

Which of the following tachyarrhythmias can result in fetal hydrops?
A. Persistent supraventricular tachycardia
B. Premature atrial contractions
C. Sinus tachycardia
A. Persistent supraventricular tachycardia

A preterm fetus with persistent supraventricular tachycardia that is not hydropic is best treated with maternal administration of
A. Digoxin
B. Phenobarbital
C. Terbutaline
A. Digoxin

The maternal medical condition that predisposes the fetus to a heart block is
A. Lupus erythematosus
B. Marfan’s disease
C. Mitral valve prolapse
A. Lupus erythematosus

The initial response in treating a primigravida being induced for preeclampsia who has a seizure is
A. Administer terbutaline to slow down uterine activity
B. Initiate magnesium sulfate
C. Perform an immediate cesarean delivery
B. Initiate magnesium sulfate

Which FHR sounds are counted with a stethoscope and a fetoscope?
A. Atrial
B. Atrial and ventricular
C. Ventricular
C. Ventricular

When using auscultation to determine FHR baseline, the FHR should be counted after the contractions for
A. 5-10 sec
B. 15-30 sec
C. 30-60 sec
C. 30-60 sec

Which of the following characteristics can be determined using FHR auscultation?
A. Baseline
B. Type of decelerations
C. Variability
A. Baseline

One advantage of using a fetoscope is that it can
A. Allow more rapid detection of a baseline change
B. More accurately assess the FHR variability
C. Verify the presence of an irregular rhythm
C. Verify the presence of an irregular rhythm

According to ACOG, intermittent auscultation is appropriate for
A. All pregnancies
B. Neither complicated nor uncomplicated pregnancies
C. Uncomplicated pregnancies
C. Uncomplicated pregnancies

A woman who is 34 weeks’ gestation is counting fetal movements each day. Today she counted eight fetal movements in a two-hour period. Based on her kick counts, this woman should
A. Continue counting for one more hour
B. Discontinue counting until tomorrow
C. Notify her provider for further evaluation
C. Notify her provider for further evaluation

A BPP score of 6 is considered
A. Abnormal
B. Normal
C. Equivocal
C. Equivocal

A modified BPP reveals the following: reactive NST with moderate variability; AFI of 7. This test would be interpreted as
A. Abnormal
B. Equivocal
C. Normal
C. Normal

As fetal hypoxia (asphyxia) worsens, the last component of the BPP to disappear is fetal
A. Breathing
B. Movement
C. Tone
C. Tone

The legal term that describes a failure to meet the required standard of care is
A. Breach of duty
B. Negligence
C. Proximate cause
A. Breach of duty

According to ACOG, in a patient without complications, the FHR tracing during the first stage of labor should be reviewed approximately every
A. 5 min
B. 15 min
C. 30 min
C. 30 min

According to ACOG, in a patient without complications, the FHR tracing during the second stage of labor should be reviewed approximately every
A. 5 min
B. 15 min
C. 30 min
B. 15 min

Since the widespread use of EFM, the rate of cerebral palsy has
A. Decreased
B. Increased
C. Remained the same
C. Remained the same

Regarding the reliability of EFM, there is
A. Good interobserver reliability
B. Good intraobserver reliability
C. Poor interobserver and intraobserver reliability
C. Poor interobserver and intraobserver reliability

The objective of intrapartum FHR monitoring is to assess for fetal
A. Acidemia
B. Oxygenation
C. Well-being
B. Oxygenation

At 32 weeks and beyond, FHR accelerations are defined as increases in FHR that must be at least
A. 10 bpm above the baseline and the accel must last at least 10 sec
B. 15 bpm above the baseline and the accel must last at least 10 sec
C. 15 bpm above the baseline and the accel must last at least 15 sec
C. 15 bpm above the baseline and the accel must last at least 15 sec

Use of the terms “beat-to-beat” variability and “long-term” variability is not recommended by the NICHD because in clinical practice
A. They are visually determined as a unit
B. Both signify an intact cerebral cortex
C. Clinical management is unchanged
A. They are visually determined as a unit

Variable decelerations in FHR are defined as visually apparent, abrupt decreases in FHR. The time from onset to nadir is less than
A. 15 sec
B. 30 sec
C. 2 min
B. 30 sec

According to NICHD definitions of FHR variability, which of the following is accurate?
A. Range 1-5 bpm = absent variability
B. Range 6-25 bpm = average variability
C. Range visually detectable but </=5 bpm = minimal variability
C. Range visually detectable but </=5 bpm = minimal variability

Late decelerations of the FHR are associated most specifically with
A. Transient fetal tissue metabolic acidosis during a contraction
B. Transient fetal hypoxemia during a contraction
C. Transient fetal asphyxia during a contraction
B. Transient fetal hypoxemia during a contraction

Assessment of FHR variability
A. Requires a fetal scalp electrode
B. Includes quantification of beat-to-beat changes
C. Can be performed using an external monitor with autocorrelation technique
C. Can be performed using an external monitor with autocorrelation technique

Decelerations that occur with at least 50% of contractions in a 20-minute window are defined as
A. Repetitive
B. Recurrent
C. Nonreassuring
B. Recurrent

Which setting is most appropriate for fetal vibroacoustic stimulation?
A. 38 weeks, active labor, FHR baseline 140 bpm, minimal variability for 60 min, no accels, no decels
B. 40 weeks, active labor, FHR baseline 150 bpm, moderate variability, prolonged decel to 60 bpm lasting 8 min
C. 34 weeks, frequent contractions without cervical change, FHR baseline 180 bpm, moderate variability, late decels
Which setting is most appropriate for fetal vibroacoustic stimulation?
A. 38 weeks, active labor, FHR baseline 140 bpm, minimal variability for 60 min, no accels, no decels

A complete description of an FHR tracing requires a qualitative and quantitative description of all of the following except
A. Baseline rate
B. Beat-to-beat variability
C. Changes or trends in the FHR patterns over time
B. Beat-to-beat variability

A Category I FHR tracing requires which of the following?
A. Baseline rate 110-160 bpm
B. Accelerations
C. No late, variable, or early decelerations
A. Baseline rate 110-160 bpm

Which of the following would be classified as a Category III FHR tracing?
A. Baseline 180 bpm, minimal variability, no accels, recurrent late decels
B. Baseline rate 140 bpm, absent variability, recurrent late decels
C. Baseline 105 bpm, minimal variability, recurrent variable decels
B. Baseline rate 140 bpm, absent variability, recurrent late decels

Normal frequency of contractions is
A. </= 5 in 10 min averaged over 20 min
B. </= 6 in 10 min averaged over 30 min
C. </= 5 in 10 min averaged over 30 min
C. </= 5 in 10 min averaged over 30 min

Which of the following most closely approximates normal umbilical artery pH at term?
A. 7.0-7.1
B. 7.1-7.2
C. 7.2-7.3
C. 7.2-7.3

The “overshoot” FHR pattern is highly predictive of
A. Fetal hypoxia
B. Preexisting fetal neurological injury
C. None of the above
C. None of the above

A key point regarding the occurrence of tachysystole is that
A. It requires FHR decelerations to be significant
B. It can occur in spontaneous or stimulated labor
C. It should be documented as hyperstimulation if oxytocin is being used
B. It can occur in spontaneous or stimulated labor

A Category II tracing
A. Predicts abnormal fetal acid-base status
B. Excludes abnormal fetal acid-base status
C. Is not predictive of abnormal fetal acid-base status
C. Is not predictive of abnormal fetal acid-base status

Plans of the health care team with a patient with a sinusoidal FHR pattern may include
A. Administration of an NST
B. Administration of tocolytics
C. Kleinhauer-Betke lab test
C. Kleinhauer-Betke lab test

Stimulation of the fetal vagus nerve will
A. Increase FHR
B. Decrease FHR
C. Initially increase, then decrease FHR
B. Decrease FHR

Which of the following is not true when assessing preterm fetuses?
A. FHR baseline may be in upper range of normal (150-160 bpm)
B. They may have fewer accels, and if <35 weeks, may be 10×10
C. Variability may be in lower range for moderate (6-10 bpm)
B. They may have fewer accels, and if <35 weeks, may be 10×10

One of the side effects of terbutaline as a tocolytic is
A. Fetal bradycardia
B. Increased oxygen consumption
C. Marked variability
B. Increased oxygen consumption

Common problems seen during monitoring of postterm fetuses include all of the following except
A. Baseline may be 100-110bpm
B. Increased variables
C. Polyhydramnios
C. Polyhydramnios

Which of the following is not commonly affected by corticosteroids?
A. Doppler flow studies
B. FHR baseline
C. Frequency of FHR accelerations
A. Doppler flow studies

A fetus of a diabetic mother may commonly develop
A. Polyhydramnios
B. Supraventricular tachycardia
C. Third-degree heart block
A. Polyhydramnios

The fetus of a mother with preeclampsia is at high risk for developing
A. Intrauterine growth restriction (IUGR)
B. Macrosomia
C. Polyhydramnios
A. Intrauterine growth restriction (IUGR)

Maternal infection most often results in
A. Fetal tachycardia
B. Moderate variability
C. Placenta previa
A. Fetal tachycardia

Uncontrolled maternal hypertension is often a causative factor in
A. Postmaturity
B. Preterm contractions
C. Placental abruption
C. Placental abruption

EFM is an excellent _ test for determining the presence or absence of neurological injury.
A. Definitive
B. Diagnostic
C. Screening
C. Screening

The primary assumption when using EFM is that _ produces physiological changes that can be detected by changes in the FHR.
A. Disruption of oxygen
B. Labor induction
C. Parity
A. Disruption of oxygen

The primary difference between intermittent auscultation (IA) and EFM is
A. EFM is primarily a visual assessment
B. IA is associated with worse outcomes
C. IA is less labor-intensive than EFM
A. EFM is primarily a visual assessment

The tocodynamometer is completely unreliable for
A. Contraction duration
B. Contraction frequency
C. Contraction intensity
C. Contraction intensity

High resting tone may occur with an IUPC because of all of the following except
A. Extraovular placement
B. Maternal BMI
C. Multiple gestations
B. Maternal BMI

All of the following are disadvantages of the ultrasound transducer for monitoring the FHR except
A. It is noninvasive
B. It may half or double the actual FHR
C. It restricts patient movement
A. It is noninvasive

The internal scalp electrode works by
A. Detecting heart motion
B. Measuring the R-to-R interval of the fetal ECG
C. Sensing the opening and closing of the fetal heart valves
B. Measuring the R-to-R interval of the fetal ECG

When educating the patient about the ultrasound transducer, the nurse should include the following:
A. Contraction strength can alter the ability to monitor the FHR
B. If baby moves, the tracing may “drop” or become erratic
C. The ultrasound detects electrical activity of the fetal heart
B. If baby moves, the tracing may “drop” or become erratic

When educating the patient about the IUPC, the nurse should include the following:
A. Can’t get out of bed
B. Does not accurately measure contraction strength
C. Measures the FHR more accurately
A. Can’t get out of bed

Intrinsic factors are those that are
A. External to the fetus
B. Internal to the fetus
C. Related to the mother
B. Internal to the fetus

The parasympathetic branch of the autonomic nervous system
A. Decreases FHR
B. Increases contractility of the fetal heart
C. Increases FHR
A. Decreases FHR

Which of the following is the primary neurotransmitter of the sympathetic branch of the autonomic nervous system?
A. Acetylcholine
B. Dopamine
C. Norepinephrine
C. Norepinephrine

Which of the following is the primary neurotransmitter of the parasympathetic branch of the autonomic nervous system?
A. Acetylcholine
B. Dopamine
C. Norepinephrine
A. Acetylcholine

Which of the following is responsible for variations in the FHR and fetal behavioral states?
A. Cerebellum
B. Cerebral cortex
C. Medulla oblongata
B. Cerebral cortex

Extrinsic factors affecting the FHR include
A. Fetal baroreceptors
B. Fetal chemoreceptors
C. Maternal oxygen transport
C. Maternal oxygen transport

When the umbilical vessels traverse the membranes to the placenta without any cord protection, this is called
A. Placenta previa
B. Succenturiate lobe (SL)
C. Velamentous insertion
C. Velamentous insertion

Which of the following is the primary factor in uteroplacental blood flow?
A.. Fetal heart rate
B. Maternal cardiac output
C. Maternal oxygen consumption
B. Maternal cardiac output

Chemoreceptors respond to changes in
A. Blood pressure
B. Pulse
C. Oxygenation
C. Oxygenation

Which of the following occurs when the parasympathetic branch of the autonomic nervous system is stimulated?
A. Norepinephrine release
B. Slowed conduction to sinoatrial node
C. Increase in fetal heart rate
B. Slowed conduction to sinoatrial node

Which of the following is responsible for fetal muscle coordination?
A. Cerebellum
B. Cerebral cortex
C. Medulla oblongata
A. Cerebellum

When the placenta covers the cervical os, this is called
A. Placenta previa
B. Succenturiate lobe (SL)
C. Velamentous insertion
A. Placenta previa

An additional placental lobe is defined as
A. Placenta previa
B. Succenturiate lobe (SL)
C. Velamentous insertion
B. Succenturiate lobe (SL)

During periods of fetal tachycardia, FHR variability is usually diminished due to
A. The dominance of the parasympathetic nervous system
B. The dominance of the sympathetic nervous system
C. Stimulation of the fetal vagus nerve
A. The dominance of the parasympathetic nervous system

The presence of moderate FHR variability is most specifically an excellent predictor of
A. Absence of fetal metabolic acidosis
B. Fetal movement
C. Fetal oxygenation
A. Absence of fetal metabolic acidosis

All of the following can be used to describe episodic FHR changes except
A. Accelerations
B. Late decelerations
C. Variable decelerations
B. Late decelerations

Periodic accelerations can indicate all of the following except
A. Stimulation of fetal chemoreceptors
B. Tracing is a maternal tracing
C. Umbilical vein compression
A. Stimulation of fetal chemoreceptors

A transient decrease in cerebral blood flow (increased cerebral blood pressure) during a contraction may stimulate _ and may cause a(n) _
A. Baroreceptors; early deceleration
B. Baroreceptors; late deceleration
C. Chemoreceptors; early deceleration
A. Baroreceptors; early deceleration

The primary physiologic goal of interventions for late decelerations is to
A. Decrease maternal oxygen consumption
B. Maximize placental blood flow
C. Maximize umbilical circulation
B. Maximize placental blood flow

Which of the following is most responsible for producing FHR variability as the fetus grows?
A. Maturation of the parasympathetic nervous system
B. Maturation of the sympathetic nervous system
C. Release of maternal prostaglandins
A. Maturation of the parasympathetic nervous system

Using the NICHD terminology, tachysystole is defined as
A. More than 5 contractions in 10 minutes averaged over a 20-minute period
B. More than 5 contractions in 15 minutes averaged over a 30-minute period
C. More than 5 contractions in 10 minutes averaged over a 30-minute period
C. More than 5 contractions in 10 minutes averaged over a 30-minute period

When a patient reports that she is feeling contractions but the nurse does not note any on the toco tracing, what is the first intervention that should be performed?
A. Ask the provider to place an IUPC
B. Reposition the toco
C. Palpate the uterus
C. Palpate the uterus

Which of the following represents the normal range for FHR baseline?
A. 120-160 bpm
B. 110-160 bpm
B. 110-160 bpm

Which of the following is not a type of supraventricular dysrhythmia?
A. Premature atrial contraction (PAC)
B. Premature ventricular contraction (PVC)
C. Supraventricular tachycardia (SVT)
B. Premature ventricular contraction (PVC)

Which is the most common type of fetal dysrhythmia?
A. Premature atrial contraction (PAC)
B. Premature ventricular contraction (PVC)
C. Third-degree heart block
A. Premature atrial contraction (PAC)

What is the most common cause of third-degree heart block in the fetus?
A. Maternal HIV infection
B. Maternal lupus
C. Tocolysis
B. Maternal lupus

All of the following are traits of fetal supraventricular tachycardia (SVT), but which is most problematic?
A. Decreases diastolic filling time
B. Dramatically increases oxygen consumption
C. Often leads to ventricular tachycardia (VT)
C. Often leads to ventricular tachycardia (VT)

Which abnormal FHR pattern is most likely to lead to hydrops in the fetus?
A. Marked variability
B. A premature ventricular contraction (PVC)
C. Supraventricular tachycardia (SVT)
C. Supraventricular tachycardia (SVT)

Which of the following is an irregular FHR pattern associated with normal conduction and rate?
A. Premature atrial contractions (PACs)
B. Sinus arrhythmias
C. Sinus tachycardias
B. Sinus arrhythmias

Which method of assessing FHR allows the clinician to hear the opening and closing of heart valves, which helps detect dysrhythmias?
A. A fetoscope
B. An internal scalp electrode (FSE)
C. An ultrasound transducer
A. A fetoscope

Which method of assessment allows the clinician to visually see cardiac wall movements?
A. A fetal echocardiogram
B. A fetal pulse oximetry reading
C. A nonstress test (NST)
A. A fetal echocardiogram

_ denotes the spontaneous, rhythmic depolarization of cardiac cells.
A. Automaticity
B. Excitability
C. Conductivity
A. Automaticity

_ denotes the readiness of cardiac cells to receive and respond to electrical stimuli.
A. Automaticity
B. Excitability
C. Conductivity
B. Excitability

_ denotes the ability of cardiac cells to conduct electrical impulses from one cell to another.
A. Automaticity
B. Excitability
C. Conductivity
C. Conductivity

Which of the following is the most common type of fetal tachyarrhythmia?
A. Atrial flutter
B. Supraventricular tachycardia (SVT)
C. Premature ventricular contraction (PVC)
B. Supraventricular tachycardia (SVT)

Which of the following is one example of a fetal tachyarrhythmia?
A. Second-degree heart block, Type I
B. Atrial fibrillation
C. Premature atrial contraction (PAC)
B. Atrial fibrillation

(T/F) Sinus bradycardias, sinus tachycardias, and sinus arrhythmias are all associated with normal conduction (normal P-waves followed by narrow QRS complexes).
True

(T/F) An internal scalp electrode will detect the actual fetal ECG.
True

(T/F) An internal scalp electrode can solely diagnose a fetal dysrhythmia.
False

_ are patterns of abnormal FHR associated with variability in R-to-R intervals, but with normal P-waves preceding normal QRS complexes.
A. Arrhythmias
B. Complete heart blocks
C. Dysrhythmias
A. Arrhythmias

_ are abnormal FHR rhythms associated with disordered impulse formation, conduction, or both.
A. Arrhythmias
B. Supraventricular tachycardias
C. Dysrhythmias
C. Dysrhythmias

Which of the following fetal dysrhythmias may be related to maternal hyperthyroidism?
A. Sinus tachycardia
B. Premature atrial contractions (PACs)
C. Third-degree heart block
B. Premature atrial contractions (PACs)

With _ premature ventricular contractions (PVCs), the baseline and variability are obscured.
A. Idioventricular
B. Bigeminal
C. Trigeminal
B. Bigeminal

With _ premature ventricular contractions (PVCs), the upward spikes will be slightly longer than the downward spikes.
A. Idioventricular
B. Bigeminal
C. Trigeminal
C. Trigeminal

Which of the following dysrhythmias may progress to atrial fibrillation or atrial flutter?
A. Premature atrial contractions (PACs)
B. Supraventricular tachycardia (SVT)
C. Sinus tachycardia
B. Supraventricular tachycardia (SVT)

Which of the following is not commonly caused by administration of indomethacin?
A. Decreased fetal urine (decreased amniotic fluid index [AFI])
B. Decreased FHR baseline
C. Increased variable decelerations
B. Decreased FHR baseline

Which of the following is not commonly caused by terbutaline administration?
A. Increased FHR baseline
B. Decreased FHR late decelerations
C. Increased maternal HR
B. Decreased FHR late decelerations

Which of the following is not commonly caused by nifedipine administration?
A. Maternal hypotension
B. Decreased uterine blood flow
C. Increased FHR accelerations
C. Increased FHR accelerations

Which of the following would likely be affected by betamethasone administration?
A. Fetal echocardiogram
B. Biophysical profile (BPP) score
C. Contraction stress test (CST)
B. Biophysical profile (BPP) score

Fetal breathing decreased with betamethasone administration

Which of the following is a common EFM finding when monitoring a preterm fetus?
A. Decreased baseline
B. Variable decelerations
C. Increased FHR variability
B. Variable decelerations

Which of the following is most strongly correlated with placental abruption?
A. Uncontrolled gestational diabetes
B. Multiple gestation
C. Uncontrolled hypertension
C. Uncontrolled hypertension

Which of the following is not typically associated with a postterm pregnancy?
A. Meconium-stained amniotic fluid
B. Presence of late decelerations in the fetal heart rate
C. Polyhydramnios
C. Polyhydramnios

Which of the following is the most appropriate method of monitoring a patient who is a gestational diabetic?
A. Daily NSTs
B. Twice-weekly BPPs
C. Weekly contraction stress tests
B. Twice-weekly BPPs

Which of the following is not commonly caused by magnesium sulfate?
A. Increased FHR baseline
B. Decreased FHR variability
C. Decreased FHR accelerations
A. Increased FHR baseline

Which of the following FHR changes would not typically be seen in a mother with chorioamnionitis?
A. Increased baseline
B. Increased variability
C. Decreased accelerations
B. Increased variability

Which of the following is false regarding electronic fetal monitoring of twins?
A. Both twins must be monitored, as opposed to monitoring one twin at a time
B. Twins rarely have accelerations and decelerations simultaneously, and tracings should appear distinctly different
C. Twins must be identified and monitored as A or B throughout the entire antenatal and intrapartum periods
B. Twins rarely have accelerations and decelerations simultaneously, and tracings should appear distinctly different

pH 6.9, PO2 15, PCO2 55, HCO3 18, BE -22
The nurse reviews the arterial gas results and concludes that the fetus had _ acidosis. With results such as these, you would expect a _ resuscitation.
A. Metabolic; lengthy
B. Metabolic; short
C. Respiratory; lengthy
A. Metabolic; lengthy

Which of the following umbilical artery cord gases would most likely result in a fetus who had a Category I strip, then had a cord prolapse and was delivered within 3 minutes?
A. pH 7.17, PO2 22, PCO2 70, HCO3 24, BE -5
B. pH 7.25, PO2 23, PCO2 46, HCO3 22, BE -8
C. pH 7.02, PO2 18, PCO2 56, HCO3 15, BE -18
A. pH 7.17, PO2 22, PCO2 70, HCO3 24, BE -5

Cord blood gases from one of the umbilical arteries best represent
A. The status of fetal acid-base balance
B. The status of maternal oxygenation
C. The status of placental function
A. The status of fetal acid-base balance

A modified biophysical profile (mBPP) is considered normal if
A. NST is reactive and mother reports at least 10 fetal movements in 2 hours
B. NST is reactive and the amniotic fluid index (AFI) is greater than 5 cm
C. There are no decelerations following contractions
B. NST is reactive and the amniotic fluid index (AFI) is greater than 5 cm

Which of the following is not a component of a BPP?
A. Amniotic fluid index (AFI)
B. Fetal breathing
C. Nonstress test (NST)
A. Amniotic fluid index (AFI)

(Should be volume, not index)

A contraction stress test (CST) is done. During the testing, there were three contractions in 10 minutes, no decels, the baseline was 145, and there was moderate variability. This is interpreted as
A. Negative
B. Positive
C. Suspicious
A. Negative

Which of the following is most likely to result in absent end diastolic flow during umbilical artery velocimetry?
A. Preeclampsia
B. Preterm labor
C. Previous cesarean delivery
A. Preeclampsia

Which of the following best describes a condition in which there is decreased oxygen in the tissues?
A. Acidosis
B. Hypoxemia
C. Hypoxia
C. Hypoxia

What is the maximum amount of time allotted for a nonstress test?
A. 20 minutes
B. 40 minutes
C. 60 minutes
B. 40 minutes

A nonstress test (NST) is nonreactive after 40 minutes. The nurse should
A. Call the physician to plan for a possible induction
B. Call the physician to arrange for a BPP or CST
C. Perform vibroacoustic stimulation and monitor patient for 20 additional minutes
B. Call the physician to arrange for a BPP or CST

(T/F) Contractions during a contraction stress test (CST) may be spontaneous or induced with oxytocin or nipple stimulation.
True

A contraction stress test (CST) is performed. Late decelerations were noted in three out of the five contractions in 10 minutes. This is interpreted as
A. Positive
B. Negative
C. Suspicious
A. Positive

A contraction stress test (CST) is performed. Late decelerations were noted in two out of the five contractions in 10 minutes. This is interpreted as
A. Positive
B. Negative
C. Suspicious
C. Suspicious

A contraction stress test (CST) is performed. Two variable decelerations were seen on the FHR tracing and there were four contractions in 10 minutes. This is interpreted as
A. Positive
B. Negative
C. Suspicious
B. Negative

A contraction stress test (CST) is performed. No decelerations were noted with the two contractions that occurred over 10 minutes. This is interpreted as
A. Positive
B. Negative
C. Unsatisfactory
C. Unsatisfactory

A patient has a reactive NST and negative CST. Select the appropriate followup.
A. Prepare patient for delivery
B. Repeat in 24 hours
C. Repeat in one week
C. Repeat in one week

A patient has a nonreactive NST and negative CST. Select the appropriate followup.
A. Prepare patient for delivery
B. Repeat in 24 hours
C. Repeat in one week
B. Repeat in 24 hours

For a patient with a suspicious or unsatisfactory CST, select the appropriate followup.
A. Prepare patient for delivery
B. Repeat in 24 hours
C. Repeat in one week
B. Repeat in 24 hours

In a patient whose CST reveals late decelerations with three out of the four induced contractions, which of the following would be the least appropriate plan for treatment?
A. Obtain physician order for BPP
B. Prepare for possible induction of labor
C. Repeat CST in 24 hours
C. Repeat CST in 24 hours

For a patient at 38 weeks’ gestation with a BPP score of 6, select the most appropriate course of action.
A. Repeat in one week
B. Consider induction of labor
C. Prepare for cesarean delivery
B. Consider induction of labor

For a patient at 35 weeks’ gestation with a BPP score of 4, select the most appropriate course of action.
A. Repeat in 24 hours
B. Obtain physician order for CST
C. Prepare for probable induction of labor
C. Prepare for probable induction of labor

(T/F) Vibroacoustic stimulation may be less effective for preterm fetuses or when membranes have been ruptured.
True

_ cord blood sampling is predictive of uteroplacental function.
A. Arterial
B. Venous
C. Maternal
B. Venous

(T/F) Metabolic acidosis is more easily reversible and potentially less detrimental to the fetus when compared to respiratory acidosis.
False

(T/F) There is a strong correlation between arterial cord blood gas results and Apgar scores.
False

Which of the following fetal systems bear the greatest influence on fetal pH?
A. Heart and lungs
B. Lungs and kidneys
C. Sympathetic and parasympathetic nervous systems
B. Lungs and kidneys

Your patient is a 41-year-old diabetic primigravida. You have just performed a vaginal exam: 6/90/-1. The fetus has been having late decels with absent to minimal variability and no accels for one hour, despite interventions to improve the tracing. The provider arrives on the unit and states that she will remain on the floor, and writes orders to start oxytocin. You inform the physician that you are not comfortable starting oxytocin based on maternal history, SVE, and FHR tracing. The provider insists that you should start oxytocin. What is your response?
A. Tell the physician that if she will remain on the unit, you will start the oxytocin.
B. Tell the physician that she can start the oxytocin herself.
C. Tell the physician that you decline to start the oxytocin, and notify the physician that you are activating your chain of command.
C. Tell the physician that you decline to start the oxytocin, and notify the physician that you are activating your chain of command.

All of the following are components of liability except
A. Breach of duty
B. Chain of command
C. Damages/loss
B. Chain of command

If the EFM pattern you see does not fit any of the definitions for the NICHD, you should
A. Choose the term closest to the pattern you see
B. Describe the tracing in detail
C. Do not chart the tracing, as there is no correlating terminology to match it
B. Describe the tracing in detail

When using SBAR communication, the “R” involves
A. Making a recommendation
B. Reporting pertinent lab results
C. Reviewing the patient’s medical history
A. Making a recommendation

Which of the following is considered using the chain of command?
A. Asking the patient to talk with the provider directly
B. Notifying the charge nurse
C. Writing a variance report
B. Notifying the charge nurse

Elements of a malpractice claim include all of the following except
A. Breach of duty
B. Deposition
C. Injury or loss
B. Deposition

Oxygen is transferred from mom to fetus via the placenta through?
Passive (Simple) Diffusion

Intervillous space perfusion is dependent on?
Adequate Uterine Blood Flow

Maternal Fetal Exchange is best promoted by which maternal position?
Either Rt or Lt Lateral

What is transfer down the concentration gradient from higher to lower called?
Diffusion

The most likely physical rationale for recurrent late decels after epidural is?
Maternal Sympathetic Blockade

Which FHR pattern would be anticipated when monitoring mono-mono twins?
Variable Decels

Fetus can survive in an environment w/ a PO2 equal to adult venous blood d/t?
increased O2 carrying capacity

Variable decels are mediated primarily by?
baroreceptors

The sympathetic branch of the ANS influences FHR to?
increase

the average difference in baseline FHR b/w 30 & 40 weeks is?
10bpm
usually 5-6; 10 is closest

Fetal blood is most highly oxygenated in the?
Ductus Venosous

An abrupt rise in fetal bp can stimulate?
variable decels

During an acute episode of fetal hypoxemia, fetal blood flow is redistributed primarily to the?
brain

Over the course of pregnancy, the FHR baseline?
decreases

FHR variability is dependent upon?
cerebral oxygen and intact CNS

chemoreceptors respond mainly to?
hypoxemia

pH: 7.22
pCO2: 50
HCO3: 24
BE: -3
normal acid-base status
(if pH is normal, answer is normal)

Fetal respiratory academia is indicated by a pH of 7.04 and a PCO2 of?

60

pH: 6.98
PCO2: 49
HCO3: 16
BE:-16
metabolic acidemia

pH: 7.04
pCO2: 80
HCO3: 22
BE: -4
respiratory acidemia

Fetal metabolic acidemia is indicated by an umbilical artery cord blood gas pH of 6.94 & BE of?

-12

Decels that occur with <50% ctx:
intermittent

An acceleration of FHR that is elicited during fetal scalp stim indicates a fetal pH of at least?
7.19

FHR characteristic most predictive of a vigorous baby at birth is?
variability

EFM tracing w/ absent variability and recurrent late decels would be categorized as?
abnormal (cat 3)

likely cause of fetal tachy w/ moderate variability in a term fetus?
maternal fever

FHR pattern likely to develop w/severe fetal anemia?
sinusoidal

marked variability is mediated by?
adrenergic activity

during 1st stage of labor for women w/ no risk factors, efm should be reviewed every?
30 min.

during 1st stage of labor for women w/complications, EFM should be reviewed every?
15 min.

during 2nd stage labor for women requiring oxytocin, EFM should be reviewed every?
5 min.

in any 10 minute sequence of FHR tracing, minimum baseline duration must be at least?
2 min.

if baseline FHR is indeterminate, refer to prior?
10 min. window

Baseline FHR variability is classified as?
absent, minimal, moderate, marked

primary goal in treatment of late decels?
maximize uteroplacentel blood flow

Rationale for low dose oxytocin protocol?
half-life of oxytocin

Initiate treatment for recurrent late decels w/ moderate variability during 1st stage?
maternal repositioning

FHR pattern likely to develop w/ severe fetal anemia?
sinusoidal

most consistent clinical sign of uterine rupture during tolac?
recurrent decels of bradycardia

loss of FHR variability can result from?
medication administration

IV stadol may result in?
transient sinusoidal (pseudosinusoidal)

When using a doppler to determine FHR, autocorrelation:
compares successive reflective US waveforms at many points

Sustained SVT increases fetal risk for:
CHF (hydrops)

toco detects changes in?
the shape of the uterine wall

fetoscope works by detecting?
sound of opening/closing of heart valves

short r-to-r intervals in fetal egg is indicative of?
tachycardia

most common fetal arrhythmia?
PAC

Fetal hydrops may develop w?
paroxysmal atrial tachy

complete heart block increases fetal risk for?
neonatal pacemaker

reactive NST in term fetus requires?
2 access 15×15 w/I 20 min.

in the BPP the chronic marker is?
AFV

most commonly cited source of adverse outcome during labor?
communication failure

Which of the following factors can have a negative effect on uterine blood flow?
a. Hypertension
b. Epidural
c. Hemorrhage
d. Diabetes
e. All of the above
e. All of the above

How does the fetus compensate for decreased maternal circulating volume?
a. Increases cardiac output by increasing stroke volume.
b. Increases cardiac output by increasing it’s heart rate.
c. Increases cardiac output by increasing fetal movement.
b. Increases cardiac output by increasing it’s heart rate.

Stimulating the vagus nerve typically produces:
a. A decrease in the heart rate
b. An increase in the heart rate
c. An increase in stroke volume
d. No change
a. A decrease in the heart rate

What initially causes a chemoreceptor response?
a. Epidurals
b. Supine maternal position
c. Increased CO2 levels
d. Decreased O2 levels
e. A & C
f. A & B
g. C & D
g. C & D

The vagus nerve begins maturation 26 to 28 weeks. Its dominance results in what effect to the FHR baseline?
a. Increases baseline
b. Decreases baseline
b. Decreases baseline

T/F: Oxygen exchange in the placenta takes place in the intervillous space.
True

T/F: The parasympathetic nervous system is a cardioaccelerator.
False

T/F: Baroreceptors are stretch receptors which respond to increases or decreases in blood pressure.
True

T/F: There are two electronic fetal monitoring methods of obtaining the fetal heart rate: the ultrasound transducer and the fetal spiral electrode.
True

T/F: Variability can be determined with the fetoscope.
False

T/F: Because the ultrasound transducer and toco transducer are sealed units, they can be dipped in warm water to make cleaning easier.
False

T/F: The most common artifact with the ultrasound transducer system for fetal heart rate is increased variability.
True

T/F: All fetal monitors contain a logic system designed to reject artifact.
True

T/F: The monitor should always be tested before starting a tracing, either external or internal mode and labeled a test.
True

T/F: In the U.S. the paper speed on the fetal monitor is set at 3cm/min.
True

T/F: Both internal and external monitoring methods are equally accurate means of obtaining the fetal heart rate and contraction patterns.
False

T/F: The external toco is usually placed over the uterine fundus to pick up contractions.
True

T/F: The external toco gives measurable uterine pressure.
False

T/F: The fetal spiral electrode can be placed when vaginal bleeding of unknown origin is present.
False

T/F: The ultrasound transducer is usually placed on the side of the uterus over the baby’s back, as the fetal heart is heard best there.
True

T/F: The spiral electrode is used to more accurately determine the frequency, duration, and intensity of uterine contractions.
False

T/F: The heart rate from a well-applied fetal spiral electrode can only be fetal, not maternal.
False

T/F: The intrauterine catheter is used to pick up the fetal heart rate.
False

T/F: The internal spiral electrode may pick up the maternal heart rate if the baby has died.
True

T/F: Fetal arrhythmias can be seen on both internal and external monitor tracings.
True

T/F: Variability and periodic changes can be detected with both internal and external monitoring.
True

T/F: Variable decelerations are a result of cord compression.
True

T/F: The presence of FHR accelerations in the intrapartum and antepartum periods is a sign of adequate fetal oxygenation at the time that it is observed
True

T/F: Variable decelerations are a vagal response.
True

T/F: Late decelerations have a gradual decrease in FHR (onset to nadir 30 seconds) and are delayed in timing with the nadir of the deceleration occurring after the peak of the contraction.
True

T/F: The fetal heart rate baseline can be determined during periods of marked variability.
False

T/F: Anything that affects maternal blood flow (cardiac output) can affect the blood flow through the placenta.
True

T/F: Variable decelerations are the most frequently seen fetal heart rate deceleration pattern in labor.
True

T/F: Minimal variability is always an indicator of hypoxia and a Cesarean section is indicated.
False

What is your first intervention in management of a patient experiencing variable decelerations?
a. Immediate delivery
b. Change maternal position
c. No treatment indicated
d. Oxygen
e. Stop oxytocin infusion
b. Change maternal position

Etiology of a baseline FHR of 165bpm occurring for the last hour can be:

  1. Maternal supine hypotension
  2. Maternal fever
  3. Maternal dehydration
  4. Unknown
    a. 1 and 2
    b. 1, 2 and 3
    c. 2, 3 and 4
    c. 2, 3 and 4

What is the most probable cause of recurrent late decelerations?
a. Utero-placental insufficiency
b. Head compression
c. Cord compression
d. Maternal position change
a. Utero-placental insufficiency

The most prevalent risk factor associated with fetal death before the onset of labor is:
a. Low socioeconomic status
b. Fetal malpresentation
c. Uteroplacental insufficiency
d. Uterine anomalies
c. Uteroplacental insufficiency

Which of the following is NOT used for antepartum fetal surveillance?
a. Fetal movement counting
b. Antepartum fetal heart rate testing
c. Biophysical profile testing
d. Maternal HCG levels
d. Maternal HCG levels

Which of the following conditions is not an indication for antepartum fetal surveillance?
a. Gestational hypertension
b. Diabetes in pregnancy
c. Fetus in breech presentation
d. Decreased fetal movement
c. Fetus in breech presentation

Which of the following does not affect the degree of fetal activity?
a. Vibroacoustic stimulation
b. Smoking
c. Fetal position
d. Gestational age
a. Vibroacoustic stimulation

To be considered reactive, a nonstress test must have:
a. 4 fetal heart rate accelerations in a 20 minute window
b. 2 fetal heart rate accelerations in a 10 minute window
c. 4 fetal heart rate accelerations in a 40 minute window
d. 2 fetal heart rate accelerations in a 20 minute window
d. 2 fetal heart rate accelerations in a 20 minute window

If a nonstress test is nonreactive after 40 minutes, the next step should be:
a. Have the client go home and do fetal movement counts
b. Do a biophysical profile or contraction stress test
c. Repeat the nonstress test within a week
d. Admit the client for delivery
b. Do a biophysical profile or contraction stress test

All of the following are components of a biophysical profile except:
a. Contraction stress test
b. Assessment of fetal breathing
c. Amniotic fluid volume measurement
d. Fetal movement assessment
a. Contraction stress test

A modified biophysical profile includes a nonstress test and:
a. Contraction stress test
b. Ultrasound assessment of fetal movement
c. Ultrasound assessment of amniotic fluid volume
d. Fetal movement counts
c. Ultrasound assessment of amniotic fluid volume

For a contraction stress test to be interpretable, you must have a minimum of:
a. 5 contractions in a 10-minute window
b. 3 contractions in a 10-minute window
c. 4 contractions in a 10-minute window
d. 2 contractions in a 10-minute window
b. 3 contractions in a 10 minute window

A negative contraction stress test is one in which:
a. No contractions are seen
b. There are late decelerations with > 50% of the contractions seen
c. There are no fetal heart rate late decelerations with the contractions
d. There is one fetal heart rate deceleration seen
c. There are no fetal heart rate late decelerations with the contractions

According to AWHONN, the normal baseline Fetal Heart Rate (FHR) is
A. 90-150 bpm
B. 100-170 bpm
C. 110-160 bpm
D. 120-140 bpm
C. 110-160 bpm

What are the two most important characteristics of the FHR?
A. Rate and decelerations
B. Variability and accelerations
C. Variability and decelerations
D. Rate and variability
B. Variability and accelerations

You recognize that an FHR tracing has been showing a decrease in variability for the last 45 minutes. Your first intervention should be to
A. Encourage ambulation
B. Administer oxygen
C. Discontinue IV fluids
D. Increase Pitocin rate
B. Administer oxygen

Resuscitation measures improves the baby’s variability, but the FHR is still not reactive. You attempt fetal scalp stimulation (FSE) because you know that a well-oxygenated fetus will respond to FSE with a(n)
A. Acceleration
B. Deceleration
C. Fetal movement
D. Sleep pattern
A. Acceleration

You are evaluating a patient in the Prenatal Testing Department who has just completed a biophysical profile (BPP). You suspect that there could be chronic fetal asphyxia because the score is below
A. 10
B. 6
C. 8
B. 6

When using a fetal scalp electrode (FSE), you notice an abnormally low FHR on the monitor. You should first
A. Compare maternal pulse simultaneously with FHR
B. Remove FSE
C. Call the doctor immediately
D. Turn off the monitor
A. Compare maternal pulse simultaneously with FHR

T/F: Umbilical cord influences that can alter blood flow include true knots, hematomas, and number of umbilical vessels.
True

T/F: Low amplitude contractions are not an early sign of preterm labor.
False

T/F: Preterm contractions are usually painful.
False

T/F: Corticosteroid administration may cause an increase in FHR accelerations.
False

T/F: Corticosteroid administration may cause an increase in FHR.
True

T/F: Contractions cause an increase in uterine venous pressure and a decrease in uterine artery perfusion.
True

As a result of the intrinsic fetal response to oxygen deprivation, increased catecholamine levels cause the peripheral blood flow to decrease while the blood flow to vital organs increases. These flow changes along with increased catecholamine secretions have what effect on fetal blood pressure and fetal heart rate?
A. Increase BP and increase HR
B. Increase BP and decrease HR
C. Decrease BP and increase HR
D. Decrease BP and decrease HR
B. Increase BP and decrease HR

All of the following might indicate a pseudosinusoidal pattern as opposed to a sinusoidal pattern, except:
A. Recent administration of narcotics to mother
B. Accelerations in FHR
C. Moderate variability
D. Frequency of oscillations of two to five cycles/min
D. Frequency of oscillations of two to five cycles/min

All of the following are appropriate interventions for fetal tachycardia except:
A. Increase maternal IV fluid rate
B. Assess maternal vital signs
C. Perform SVE
D. Administer oxygen
C. Perform SVE

During a term antepartum NST (non-stress test), you notice several variable decelerations that decrease at least 15 bpm and last at least 15 sec long. Which of the following is the least likely explanation?
A. True knot
B. Gestational diabetes
C. Umbilical cord entanglement
D. Oligohydramnios
B. Gestational diabetes

All of the following are likely causes of prolonged decelerations except:
A. Uterine tachysystole
B. Prolapsed cord
C. Maternal hypotension
D. Maternal fever
D. Maternal fever

_ decelerations occur with less than 50% of contractions.
A. Recurrent
B. Intermittent
C. Repetitive
B. Intermittent

_ decelerations occur with greater than or equal to 50% of contractions.
A. Recurrent
B. Intermittent
C. Repetitive
A. Recurrent

All of the following could likely cause minimal variability in FHR except
A. Magnesium sulfate administration
B. Fetal sleep cycle
C. Narcotic administration
D. Ephedrine administration
D. Ephedrine administration

When an IUPC has been placed, Montevideo units must be _ or greater for adequate cervical change to occur.
A. 100
B. 200
C. 300
D. 400
B. 200

The __ increases the heart rate and strengthens myocardial contractions through the release of epinephrine and nonepinephrine.
A. Sympathetic nervous system
B. Parasympathetic nervous system
A. Sympathetic nervous system

The __, through stimulation of the vagus nerve, reduces FHR and maintains variability.
A. Sympathetic nervous system
B. Parasympathetic nervous system
B. Parasympathetic nervous system

What would be a suspected pH in a fetus whose FHTs included recurrent late decelerations during labor?
A. 7.10
B. 7.26
C. 7.32
D. 7.41
A. 7.10

What is the most common cause of sinusoidal patterns?
A. Prolapsed cord
B. Rh incompatibility
C. Recurrent late decelerations
D. Oligohydramnios
B. Rh incompatibility

Before _ weeks of gestation, an increase in FHR that peaks at least 10 bpm above the baseline and lasts at least 10 seconds is considered an acceleration.
A. 28
B. 30
C. 32
D. 36
C. 32

The expected response of the fetal heart rate to active fetal movement of a 31-week gestational age fetus is:
a. Suppression of normal short term variability for 15 seconds
b. Acceleration of at least 15 beats per minute for 15 seconds
c. Acceleration followed by a 15-second deceleration of the heart rate
d. Acceleration of at least 10 beats per minute for 10 seconds
d. Acceleration of at least 10 beats per minute for 10 seconds

The nurse notes a pattern of variable decelerations to 75 bpm on the fetal monitor. The initial nursing action is to:
a. Reposition the woman
b. Administer oxygen
c. Increase the intravenous fluid infusion
d. Stimulate the fetal scalp
a. Reposition the woman

The tocotransducer should be placed:
a. In the suprapubic area
b. In the fundal area
c. Over the xiphoid process
d. Within the uterus
b. In the fundal area

The nurse notes a pattern of decelerations on the fetal monitor that begins shortly after the contraction and returns to baseline just before the contraction is over. The correct nursing response is to:
a. Give the woman oxygen by facemask at 8-10 L/min
b. Position the woman on her opposite side
c. Increase the rate of the woman’s intravenous fluid
d. Continue to observe and record the normal pattern
d. Continue to observe and record the normal pattern

Determining the FHR baseline requires the nurse to approximate the mean FHR rounded to increments of 5 bpm during a _-minute window (excluding accelerations and decelerations).
A. 2
B. 5
C. 10
D. 20
C. 10

Uterine tachysystole is observed when there are
A. 5 or more contractions in 10 min
B. 6 or more contractions in 10 min
C. 10 or more contractions in 10 min
D. 7 or more contractions in 10 min
B. 6 or more contractions in 10 min

Which of the following interventions would best stimulate an acceleration in the FHR?
A. Provide juice to patient
B. Perform vaginal exam
C. Turn patient on left side
D. Vibroacoustic stimulation
B. Perform vaginal exam

Scalp stimulation

Assessment of the _ is an indirect measurement of fetal oxygenation.
A. Fetal heart rate
B. Fetal scalp sampling
C. Uterine activity
D. Direct Coombs
A. Fetal heart rate

T/F: Intrauterine pressure catheters (IUPCs) do not increase risk for infection when placed on patients with intact membranes.
False

Membranes must be ruptured for use; infection is a risk

What are abnormal fetal heart rate tracings predictive of?
A. Likelihood of spontaneous vaginal delivery
B. Newborn condition at time of delivery
C. Fetal acid-base abnormalities
D. Fetal intrauterine growth
C. Fetal acid-base abnormalities

Which of the following is not an intervention that should be implemented in a patient with uterine tachysystole?
A. Administer terbutaline
B. Increase IV fluid rate
C. Decrease or discontinue IV oxytocin
D. Prepare patient for cesarean section
D. Prepare patient for cesarean section

Which of the following is most effective in determining the strength of a patient’s contractions?
A. Patient report
B. Tocodynanamometer tracing
C. RN palpation
D. Sterile vaginal exam during a contraction
C. RN palpation

The FHR is controlled by the
A. Sympathetic nervous system
B. Sinoatrial node
C. Atrioventricular node
D. Parasympathetic nervous system
B. Sinoatrial node

How do baseline heart rates differ in premature fetuses?
A. They are often lower
B. They are often higher
C. They are less likely to have decelerations
D. They experience longer accelerations
B. They are often higher

T/F: If etiology of fetal tachycardia is secondary to extrauterine infection, FHR will return to normal as maternal fever resolves.
True

T/F: Fetal tachycardia is a normal compensatory response to transient fetal hypoxemia.
True

Fetal heart rate bradycardia is defined as
A. FHR <110bpm lasting 10 min or greater
B. FHR <110bpm lasting 2 min or greater
C. FHR <110bpm lasting 20 min or greater
A. FHR <110bpm lasting 10 min or greater

_ variability warrants cesarean section delivery.
A. Minimal
B. Moderate
C. Marked
D. Absent
D. Absent

At how many weeks gestation should FHR variability be normal in manner?
A. 24 weeks
B. 28 weeks
C. 32 weeks
D. 36 weeks
B. 28 weeks

A deceleration from 145bpm down to 100bpm lasting 12 minutes may be defined as a
A. Prolonged deceleration
B. Variable deceleration
C. Late deceleration
D. Baseline change
D. Baseline change

Reduced respiratory gas exchange from persistent decelerations may cause a rise in fetal PCO2, which leads first to , then .
A. Respiratory alkalosis; metabolic acidosis
B. Respiratory acidosis; metabolic acidosis
C. Respiratory alkalosis; metabolic alkalosis
D. Respiratory acidosis; metabolic acidosis
B. Respiratory acidosis; metabolic acidosis

Decreased intervillious exchange of oxygenated blood resulting in fetal hypoxia is typically present in _.
A. Variable decelerations
B. Late decelerations
C. Early decelerations
D. Accelerations
B. Late decelerations

Place the following interventions for a sinusoidal FHR in the correct order:

  1. Prepare for cesarean delivery
  2. Place patient in lateral position
  3. Determine if pattern is related to narcotic analgesic administration
  4. Provide oxygen via face mask
    A. 4, 2, 3, 1
    B. 3, 1, 2, 4
    C. 4, 3, 2, 1
    D. 3, 2, 4, 1
    D. 3, 2, 4, 1

The _ is the source of all fetal oxygenation.
A. Placenta
B. Umbilical cord
C. Mother
D. Amniotic fluid
C. Mother

FHTs with recurrent variable decelerations, no accelerations, and minimal variability would be categorized as
A. Category I
B. Category II
C. Category III
B. Category II

FHTs with absent variability and bradycardia would be categorized as
A. Category I
B. Category II
C. Category III
C. Category III

FHTs with moderate variability, no accelerations, and early decelerations would be categorized as
A. Category I
B. Category II
C. Category III
A. Category I

FHTs with accelerations, no decelerations, and minimal variability would be categorized as
A. Category I
B. Category II
C. Category III
B. Category II

Sinusoidal pattern is categorized as
A. Category I
B. Category II
C. Category III
B. Category III

FHTs with absent variability and no accelerations or decelerations would be categorized as
A. Category I
B. Category II
C. Category III
B. Category II

Absence of accelerations following fetal stimulation (i.e. scalp stimulation) is categorized as
A. Category I
B. Category II
C. Category III
B. Category II

FHTs with minimal variability, absent accelerations, and a 3-minute prolonged deceleration would be categorized as
A. Category I
B. Category II
C. Category III
B. Category II

FHTs with minimal variability and a baseline of 95bpm would be categorized as
A. Category I
B. Category II
C. Category III
B. Category II

FHTs with a baseline of 170bpm, moderate variability, and no accelerations or decelerations would be categorized as
A. Category I
B. Category II
C. Category III
B. Category II

FHTs with a baseline 135bpm, moderate variability, accelerations, and one late deceleration would be categorized as
A. Category I
B. Category II
C. Category III
B. Category II

A prolonged acceleration lasts greater than _ minutes and less than _ minutes.
A. 2; 10
B. 2; 20
C. 10; 20
D. 10; 20
A. 2; 10

_ FHR patterns are those associated with uterine contractions.
A. Periodic
B. Episodic
C. Recurrent
D. Irregular
A. Periodic

_ FHR patterns are those that are not associated with uterine contractions.
A. Periodic
B. Episodic
C. Recurrent
D. Irregular
B. Episodic

Which of the following is not a likely cause of a sinusoidal FHR pattern?
A. Chronic fetal bleeding
B. Fetal hypoxia or anemia
C. Triple screen positive for Trisomy 21
D. Fetal isoimmunization
C. Triple screen positive for Trisomy 21

Which of the following factors is not likely to cause uteroplacental insufficiency?
A. Late-term gestation
B. Preeclampsia
C. Gestational diabetes
D. Polyhydramnios
E. Maternal smoking or drug use
D. Polyhydramnios

Which of the following are considered determinants of fetal well-being? (Select all that apply).
A. Absence of decelerations in FHR
B. Palpation of fetal movement
C. Presence of accelerations in FHR
D. Moderate variability in FHR
E. Presence of early decelerations in second stage
C. Presence of accelerations in FHR
D. Moderate variability in FHR

When auscultation is used for fetal assessment during labor for a low-risk woman, the FHR should be auscultated in the first stage of labor every
A. 5 min
B. 15-30 min
C. 60 min
B. 15-30 min

For a low-risk woman in the second stage of labor, the FHR should be auscultated every
A. 5-15 min
B. 30 min
C. 60 min
A. 5-15 min

The normal FHR baseline
A. Decreases during labor
B. Fluctuates during labor
C. Increases during labor
B. Fluctuates during labor

Bradycardia in the second stage of labor following a previously normal tracing may be caused by fetal
A. Hypoxemia
B. Rotation
C. Vagal stimulation
C. Vagal stimulation

A likely cause of fetal tachycardia with moderate variability is
A. Fetal hypoxemia
B. Maternal fever
C. Vagal stimulation
B. Maternal fever

Reduction in FHR variability can result from
A. Fetal scalp stimulation
B. Medication administration
C. Vaginal examination
B. Medication administration

The primary goal in treatment for late decelerations is to
A. Correct cord compression
B. Improve maternal oxygenation
C. Maximize uteroplacental blood flow
C. Maximize uteroplacental blood flow

The most frequently observed type of FHR deceleration is
A. Early
B. Late
C. Variable
C. Variable

Amnioinfusion may be useful in alleviating recurrent decelerations that are
A. Early
B. Late
C. Variable
C. Variable

Findings indicative of progressive fetal hypoxemia are
A. Late decelerations, moderate variability, stable baseline rate
B. Prolonged decelerations recovering to baseline and moderate variability
C. Loss of variability and recurrent late or variable decelerations
C. Loss of variability and recurrent late or variable decelerations

Clinically significant fetal metabolic academia is indicated by an arterial cord gas pH of less than or equal to 7.10 and a base deficit of
A. 3
B. 6
C. 12
C. 12

Fetal bradycardia can result during
A. The sleep state
B. Umbilical vein compression
C. Vagal stimulation
C. Vagal stimulation

While caring for a 235-lb laboring woman who is HIV-seropositive, the external FHR tracing is difficult to obtain. An appropriate nursing action would be to
A. Apply a fetal scalp electrode
B. Auscultate for presence of FHR variability
C. Notify the attending midwife or physician
C. Notify the attending midwife or physician

FHR decelerations that are benign and do not require intervention are
A. Early
B. Late
C. Variable
A. Early

FHR decelerations that results from decreased uteroplacental blood flow are
A. Early
B. Late
C. Variable
B. Late

FHR decelerations that results from umbilical cord compression are
A. Early
B. Late
C. Variable
C. Variable

An FHR pattern associated with severe fetal anemia is
A. Lambda
B. Saltatory
C. Sinusoidal
C. Sinusoidal

A workup for maternal systemic lupus erythematosus would likely be ordered in the presence of
A. Complete fetal heart block
B. Premature ventricular contractions
C. Fetal supraventricular tachycardia
A. Complete fetal heart block

Which IV fluid is most appropriate for maternal administration for intrauterine resuscitation?
A. Lactated Ringer’s solution
B. D5L/R
C. Normal saline
C. Normal saline

The position that best promotes maternal-fetal exchange is
A. Left lateral
B. Right lateral
C. Either right or left lateral
C. Either right or left lateral

The most appropriate equipment for administration of maternal oxygen for intrauterine resuscitation at 10 L/min is a
A. Nasal cannula
B. Simple face mask
C. Nonrebreather face mask
C. Nonrebreather face mask

Accurate determination of baseline rate requires
A. At least 2 contiguous minutes of FHR in a 10-min window
B. Evaluation of the FHR over at least a 10-min window
C. Averaging the FHR over 30 min
B. Evaluation of the FHR over at least a 10-min window

An EFM tracing with absent variability and no decelerations would be classified as
A. Category I
B. Category II (indeterminate)
C. Category III
B. Category II (indeterminate)

An EFM tracing with absent variability and intermittent late decelerations would be classified as
A. Category I
B. Category II
C. Category III
B. Category II

Interpretation and classification of FHR patterns are based on predictability of fetal status
A. At birth
B. At the time the pattern is observed
C. Over the previous hour
B. At the time the pattern is observed

Amnioinfusion is an appropriate measure for
A. Thick, meconium-stained fluid
B. Oligohydramnios
C. Recurrent variable decelerations unresolved by position changes
C. Recurrent variable decelerations unresolved by position changes

Baroreceptors respond to changes in fetal
A. Blood pressure
B. Oxygen status
C. Acid-base status
A. Blood pressure

Fetal scalp stimulation is appropriate in the context of
A. Minimal variability
B. Prolonged deceleration
C. Bradycardia
A. Minimal variability

Maternal oxygen administration is appropriate in the context of
A. Recurrent variable decelerations/moderate variability
B. Intermittent late decelerations/minimal variability
C. Prolonged decelerations/moderate variability
B. Intermittent late decelerations/minimal variability

A preterm fetus
A. Is more susceptible to hypoxic insults during labor than the term fetus
B. Requires internal monitoring if oxytocin is used for labor induction or augmentation
C. Should be born via cesarean section unless there are maternal contraindications
A. Is more susceptible to hypoxic insults during labor than the term fetus

Oxygen is transferred from the mother to the fetus via the placenta through
A. Active transport
B. Passive diffusion
C. Facilitated diffusion
B. Passive diffusion

Resting tone and intensity of uterine contractions cannot be assessed by
A. External tocodynamometer
B. Manual palpation
C. Intrauterine pressure catheter
A. External tocodynamometer

The FHR characteristic most predictive of a well-oxygenated baby at the time observed is
A. Moderate variability
B. Stable baseline rate
C. Absence of decelerations
A. Moderate variability

In the context of hypoxemia, fetal blood flow is shifted to the
A. Brain
B. Liver
C. Lungs
A. Brain

Baroreceptor-mediated decelerations are
A. Early
B. Late
C. Variable
C. Variable

The primary goal in the treatment of variable decelerations is to
A. Correct umbilical cord compression
B. Improve maternal oxygenation
C. Maximize blood flow to the uterus
A. Correct umbilical cord compression

Umbilical artery gas results reflect the status of the
A. Mother
B. Fetus
C. Placenta
B. Fetus

An appropriate initial treatment for recurrent late decelerations with moderate variability during first stage labor is
A. Amnioinfusion
B. Maternal repositioning
C. Oxygen at 10L per nonrebreather face mask
B. Maternal repositioning

Most fetal dysrhythmias are not life-threatening, except for _, which may lead to fetal congestive heart failure.
Supraventricular tachycardia

Medications, prematurity, fetal sleep, fetal dysrhythmia, anesthetic agents, or cardiac anomalies may result in _ variability.
Decreased

In the presence of late or variable decelerations, two parameters that indicate adequate fetal oxygenation are _ and _.
Moderate variability; normal baseline rate

To correctly interpret a baseline FHR as tachycardic or bradycardic, the rate must persist for a minimum of _ min.
10

In assessing fetal well-being, the most important characteristic of the FHR is
A. Rate
B. Variability
C. Presence of accelerations
D. Absence of decelerations
B. Variability

Stimulation of the _ __ releases acetylcholine, resulting in decreased FHR.
Parasympathetic nervous system

The _ __ maintains transmission of beat-to-beat variability.
Parasympathetic nervous system

Stimulation of the _ __ releases catecholamines, resulting in increased FHR.
Sympathetic nervous system

Stimulation of _ results in abrupt decreases in FHR, CO, and BP.
Baroreceptors

Baroreceptors influence _ decelerations with moderate variability.
Variable

Following an ultrasound which revealed decreased amniotic fluid, a woman at term is admitted in early labor. It should be recognized that oligohydramnios often results in fetal heart rate decelerations that are
A. Late in onset or occur after the peak of the contraction
B. Synchronous with that of the contraction
C. Varied in depth and duration
C. Varied in depth and duration

In comparing early and late decelerations, a distinguishing factor between the two is
A. Onset time to the nadir of the deceleration
B. The number of decelerations that occur
C. Timing in relation to contractions
C. Timing in relation to contractions

The underlying cause of early decelerations is decreased
A. Baroceptor response
B. Increased peripheral resistance
C. Vagal reflex
C. Vagal reflex

Glucose is transferred across the placenta via _ _.
Facilitated diffusion

Oxygen, carbon dioxide, water, electrolytes, urea, uric acid, fatty acids, fat-soluble vitamins, narcotics barbiturates, anesthetics, and antibiotics are transferred across the placenta via _ _.
Simple (passive) diffusion

Amino acids, water-soluble vitamins, calcium, phosphorus, iron, and iodine are transferred across the placenta via _ _.
Active transport

Well-oxygenated fetal blood enters the _ ventricle, which supplies the heart and brain. Less-oxygenated blood enters the __ ventricle, which supplies the rest of the body.
Left; right

The _ _ allows relatively well-oxygenated blood to enter the fetal heart directly, bypassing the liver.
Ductus venosus

The _ _ allows blood to bypass the lungs, flowing from the pulmonary artery to the aorta.
Ductus arteriosus

The _ _ is the shunt that bypasses the fetal lungs, moving blood from the right atrium to the left atrium.
Foramen ovale

Fetal blood has a _ hemoglobin concentration compared to adults.
A. Higher
B. Lower
A. Higher

Greater oxygen-carrying capacity

Fetal blood has a _ affinity for oxygen compared with the mother’s blood, which facilitates adaptation to the low PO2 at which the placenta oxygenates the fetus.
A. Higher
B. Lower
A. Higher

The fetus has a _ cardiac output and heart rate than the adult, resulting in rapid circulation.
A. Higher
B. Lower
A. Higher

As the FHR increases, the myocardium consumes _ oxygen.
A. More
B. Less
A. More

The fetal spiral electrode measures the
A. Peaks of the Doppler waveforms
B. R to R intervals of the fetal heart
C. ST segments of the fetal ECG
B. R to R intervals of the fetal heart

A woman who is admitted for an induction of labor with oxytocin is questioning the need for continuous electronic fetal monitoring. The appropriate response to this woman is
A. Hospital policy requires that all patients have continuous electronic fetal monitoring.
B. I would like to answer your questions about continuous monitoring and give you some information about why continuous monitoring is recommended.
C. Nurses have more training with continuous electronic fetal monitoring than intermittent auscultation so it is safer for you and your baby to use this method.
B. I would like to answer your questions about continuous monitoring and give you some information about why continuous monitoring is recommended.

The greater affinity that fetal hemoglobin has for oxygen allows for
A. Easier release of oxygen to the tissues
B. Greater binding of oxygen
C. Stimulation of erythropoietin release
B. Greater binding of oxygen

The process by which oxygen and carbon dioxide pass from a region of higher concentration to one of a lower concentration is called
A. Active transport
B. Simple diffusion
C. Facilitated diffusion
B. Simple diffusion

In the healthy fetus, blood flows from the right atrium to the left atrium through the
A. Ductus arteriosus
B. Ductus venosus
C. Foramen ovale
C. Foramen ovale

In the healthy fetus, the umbilical cord enters the fetal abdomen and bypasses the liver through the
A. Ductus arteriosus
B. Ductus venosus
C. Foramen ovale
B. Ductus venosus

Which statement best describes the relationship between maternal and fetal hemoglobin levels?
A. Fetal hemoglobin is higher than maternal hemoglobin
B. Maternal hemoglobin is higher than fetal hemoglobin
C. Maternal and fetal hemoglobin are the same
A. Fetal hemoglobin is higher than maternal hemoglobin

Baroreceptors respond mainly to changes in
A. Blood pressure
B. Hormonal changes
C. Oxygen and carbon dioxide levels
A. Blood pressure

Stimulation of the sympathetic nervous system causes the fetal heart rate to
A. Decrease
B. Increase
C. Remain the same
B. Increase

Which statement describes normal uterine activity?
A. Frequency of 1-1/2 to 2 minutes
B. Intensity of 90 mmHg early in labor
C. Resting tone less than 20-25 mmHg
C. Resting tone less than 20-25 mmHg

Greater than 5 contractions in 10 minutes averaged over 30 minutes indicates
A. Excessive uterine activity
B. Hyperstimulation
C. Tachysystole
C. Tachysystole

A 36 week gestation patient is brought to triage by squad after an MVA on her back. She is not bleeding and denies pain. She is not short of breath, but c/o dizziness and nausea since they put her on the gurney. The most likely cause is
A. Abruptio placenta
B. Preterm labor
C. Supine hypotension
C. Supine hypotension

One compensatory mechanism that helps maintain oxygen availability to the fetus during maternal exercise is
A. A decrease in maternal hematocrit
B. Transient increase in uterine blood flow
C. An increase in uterine oxygen uptake
C. An increase in uterine oxygen uptake

When the hydrogen ion content in the blood rises, the pH
A. Lowers
B. Neutralizes
C. Rises
A. Lowers

A woman receives terbutaline for an external version. You may expect what on the fetal heart tracing?
A. Decrease in variability
B. Increase in baseline
C. No change
B. Increase in baseline

What affect does magnesium sulfate have on the fetal heart rate?
A. Decreases variability
B. Increases variability
C. No change
A. Decreases variability

NICHD guidelines apply only to
A. Intrapartum patients
B. Internal monitoring of fetal heart rate
C. Interpreting tracings of good quality
C. Interpreting tracings of good quality

According to NICHD terminology, variability can be accurately accessed
A. Visually, by determining the number of R to R intervals in a one minute period
B. Visually, by determining the amplitude of the FHR change in bpm from the baseline
C. Only when a fetal spiral electrode is in place
B. Visually, by determining the amplitude of the FHR change in bpm from the baseline

When looking at the fetal heart rate, the most important characteristic to determine the absence of metabolic acidemia is
A. Absence of late decelerations
B. Baseline rate within normal range
C. Presence of moderate variability
C. Presence of moderate variability

Sinusoidal pattern can be documented when
A. Cycles are 4-6 beats per minute in frequency
B. The pattern lasts 20 minutes or longer
C. There is moderate or minimal variability
B. The pattern lasts 20 minutes or longer

Vagal stimulation would be manifested as what type of fetal heart rate pattern?
A. Acceleration
B. Early deceleration
C. Tachycardia
B. Early deceleration

Which fetal monitoring pattern is characteristic of cephalopelvic disproportion, especially when seen at the onset of labor?
A. Early deceleration
B. Late deceleration
C. Variable deceleration
A. Early deceleration

A risk of amnioinfusion is
A. Prolonged labor
B. Uterine overdistension
C. Water intoxication
B. Uterine overdistension

A fetal heart rate pattern that the NICHD has identified as predictive of current or impending fetal asphyxia so severe that the fetus is at risk of neurologic damage or death is
A. Baseline tachycardia with absent variability
B. Minimal baseline variability with recurrent late decelerations
C. Recurrent late or variable decelerations with absent variability
C. Recurrent late or variable decelerations with absent variability

A fetal heart rate pattern that can occur when there is a prolapsed cord is
A. Marked variability
B. Prolonged decelerations
C. Tachycardia
B. Prolonged decelerations

The patient is in early labor with pitocin at 8 mu/min, and FHR is Category I. In the next 15 minutes, there are 18 uterine contractions. Recommended management is to
A. Address contraction frequency by reducing pitocin dose
B. Continue to increase pitocin as long as FHR is Category I
C. Turn the patient on her side and initiate an IV fluid bolus
C. Turn the patient on her side and initiate an IV fluid bolus

A woman at 38 weeks gestation is in labor. The labor has been uneventful, and the fetal heart tracings have been normal. Spontaneous rupture of membranes occurs; fetal heart rate drops to 90 beats per minute for four minutes and then resumes a normal pattern. The most likely etiology for this fetal heart rate change is
A. Abnormal fetal presentation
B. Impaired placental circulation
C. Possible cord compression
C. Possible cord compression

A 42 week gestation woman has been diagnosed with oligohydramnios. Based on this, a FHR change that can be expected is
A. Late deceleration
B. Minimal variability
C. Variable deceleration
C. Variable deceleration

Mono-mono zygotic twins are prone to what type of deceleration during labor?
A. Early
B. Late
C. Variable
C. Variable

During labor, the recommended fetal heart rate assessment interval for auscultation is every
A. 15-30 minutes in the active phase of the first stage and every 5-15 minutes in second stage
B. 15 minutes no matter what stage of labor
C. 60 minutes in the active phase of the first stage and every 30 minutes in second stage
A. 15-30 minutes in the active phase of the first stage and every 5-15 minutes in second stage

What fetal heart rate characteristics can be determined with auscultation?
A. Baseline
B. Early decelerations
C. Variability
A. Baseline

When auscultating the fetal heart rate, the provider/nurse should simultaneously assess the maternal
A. Blood pressure
B. Pain level
C. Radial pulse
C. Radial pulse

A woman has 10 fetal movements in one hour. This is considered what kind of movement?
A. Decreased
B. Excessive
C. Normal
C. Normal

If the pH is low, what other blood gas parameter is used to determine if the acidosis is respiratory or metabolic?
A. HCO3
B. PCO2
C. PO2
B. PCO2

The following cord blood gasses are consistent with: pH 7.10, pCO2 70, pO2 25, base excess -10
A. Metabolic acidosis
B. Mixed acidosis
C. Respiratory acidosis
C. Respiratory acidosis

A fetus at 36 weeks receives a biophysical profile (BBP) score of 6. The amniotic fluid was scored as normal. The expected management is
A. Immediate delivery
B. Repeat the test in 24 hours
C. Schedule the next test in one week
B. Repeat the test in 24 hours

A woman with gestational diabetes is 38 weeks gestation. Her Biophysical Profile score is 4. This indicates need for
A. Follow up in one week
B. Induction of labor
C. Emergent cesarean section
B. Induction of labor

A 35 week gestation fetus is having an NST. The fetal heart rate baseline is 130 bpm. The nurse is using vibroacoustic stimulation to reduce the length of time needed to obtain the NST. Fetal well-being requires
A. 1 acceleration to 145 bpm
B. 2 accelerations to 140 bpm
C. 2 accelerations to at least 145 bpm
C. 2 accelerations to at least 145 bpm

A woman desires a natural childbirth. The nurse puts pressure on her to get an epidural. This is an example of going against which ethical principle?
A. Autonomy
B. Beneficence
C. Justice
A. Autonomy

One characteristic of a high reliability perinatal unit is
A. Alarms can only be called by unit leaders
B. Reliance on memorization of protocols
C. The organization creates a safety-oriented culture
C. The organization creates a safety-oriented culture

One example of evidence-based practice related to fetal monitoring is
A. Diagnosis of uterine rupture with an IUPC
B. Intermittent auscultation for the low-risk patient
C. Using electronic fetal monitoring to prevent cerebral palsy
B. Intermittent auscultation for the low-risk patient

Supporting the parents decision to choose no extraordinary measures on their baby who is about to deliver at 24 3/7 weeks gestation despite the nurses personal opinion is an example of
A. Autonomy
B. Beneficence
C. Non-maleficence
A. Autonomy

_ primarily function(s) to regulate respiratory activity and control circulation by responding to changes in arterial PO2, PCO2, and acid-base balance.
A. Baroreceptors
B. Chemoreceptors
C. Cardioregulatory center
B. Chemoreceptors

_ are protective stretch receptors located in the aortic arch and the carotid sinuses at the bifurcation of the external and internal carotid arteries.
A. Baroreceptors
B. Chemoreceptors
C. Arteries
A. Baroreceptors

_ respond to increases in fetal arterial blood pressure by detecting the amount of stretch and sending impulses via the vagus nerve to the midbrain, decreasing FHR, CO, and BP.
A. Baroreceptors
B. Chemoreceptors
C. Veins
A. Baroreceptors

As a contraction beings, partial umbilical cord compression causes occlusion of the low-pressure vein and decreased return of blood to the fetal heart, resulting in decreased CO, hypotension, and a compensatory FHR _.
A. Acceleration
B. Early deceleration
C. Late deceleration
D. Variable deceleration
A. Acceleration

With complete umbilical cord occlusion, the two umbilical arteries also become occluded, resulting in sudden fetal hypertension, stimulation of the baroreceptors, and a sudden _ in FHR.
A. Increase
B. Decrease
B. Decrease

Central are located in the medulla oblongata; peripheral are found in the carotid sinuses and aortic arch.
A. Baroreceptors
B. Chemoreceptors
B. Chemoreceptors

When a fetus is stressed, catecholamine release (epinephrine, norepinephrine) occurs from the medulla oblongata, shunting blood _ the brain, heart, and adrenal glands.
A. Toward
B. Away from
A. Toward

Because stroke volume in the fetus does not fluctuate significantly, fetal cardiac output is _ fetal heart rate.
A. Greater than
B. Less than
C. Approximately equal to
C. Approximately equal to

In _ sleep, the fetus may be observed to have infrequent or absent body movements, normal FHR baseline, minimal variability, and a nonreactive NST, but may respond to external stimuli. Such typically occurs by 28 to 32 weeks’ gestation.
A. REM (active sleep state)
B. Non-REM (quiet sleep state)
B. Non-REM (quiet sleep state)

Place the following steps for performing Leopold’s maneuver in the appropriate order.

  1. Assess location of fetal back
  2. Determine the descent of the presenting part
  3. Identify the presenting part
  4. Assess part of the fetus in the upper uterus
    A. 1, 3, 2, 4
    B. 2, 4, 1, 2
    C. 4, 1, 3, 2
    D. 4, 3, 2, 1
    C. 4, 1, 3, 2

When the internal mode of monitoring (FSE, IUPC) is used, the information obtained on the uterine activity panel on the fetal monitor should be validated by the clinician by
A. Asking the patient to report when she is feeling a contraction
B. Ultrasound imaging
C. Palpation
C. Palpation

T/F: Use of a fetoscope for intermittent auscultation of the fetal heart rate may be used to detect baseline, rhythm, changes from baseline, and presence of an irregular rhythm.
True

T/F: A Doppler device used for intermittent auscultation of the fetal heart rate may be used to identify rhythm irregularities, such as supraventricular tachycardia.
False

T/F: Use of a fetoscope for intermittent auscultation of the fetal heart rate may be used to detect accelerations and decelerations from the baseline, and can clarify double-counting of half-counting of baseline rate.
True

T/F: Auscultation may be used to detect baseline variability and discriminate FHR deceleration patterns.
False

_ represents increased sympathetic or decreased parasympathetic autonomic tone.
A. Bradycardia
B. Tachycardia
B. Tachycardia

T/F: In the context of moderate variability, late decelerations are considered neurogenic in origin and are typically amenable to intrauterine resuscitation techniques directed towards maximizing uterine blood flow.
True

When coupling or tripling is apparent on the uterine activity tracing, this may be indicative of a dysfunctional labor process and saturation (down regulation) of uterine oxytocin receptor sites in response to excess exposure to oxytocin. Which of the following interventions would be most appropriate?
A. Normal response; continue to increase oxytocin titration
B. Turn patient on side
C. Decrease or discontinue oxytocin infusion
C. Decrease or discontinue oxytocin infusion

The most common tachyarrhythmia in fetuses, supraventricular tachycardia, typically occurs at a rate of _ to _ bpm with minimal or absent variability.
A. 160-200
B. 200-240
C. 240-260
C. 240-260

In a patient with oxytocin-induced tachysystole with normal fetal heart tones, which of the following should be the nurse’s initial intervention?
A. Assist the patient to lateral position
B. Discontinue Pitocin
C. Administer IV fluid bolus
A. Assist the patient to lateral position

In a patient with oxytocin-induced tachysystole with indeterminate or abnormal fetal heart tones, which of the following should be the nurse’s initial intervention?
A. Assist the patient to lateral position
B. Discontinue Pitocin
C. Administer IV fluid bolus
B. Discontinue Pitocin

Fetal hypoxia and acidemia are demonstrated by pH < _ and base excess < _.
< 7.15; < -8

T/F: Amnioinfusion is an appropriate intervention to attempt to resolve patterns of moderate to severe late decelerations.
False

T/F: Amnioinfusion is an appropriate intervention to attempt to resolve patterns with absent variability.
False

T/F: Amnioinfusion may be an appropriate intervention for patients with oligohydramnios in the prevention of the development of variable decelerations.
False

Not for prevention

T/F: Meconium-stained amniotic fluid is an indication for amnioinfusion.
False

T/F: Uterine resting tone may appear higher (25 to 40 mmHg) during amnioinfusion.
True

_ denotes a decrease in oxygenation of the fetal tissues.
A. Hypercapnia
B. Hypoxia
C. Hypoxemia
B. Hypoxia

_ denotes a decrease in oxygen content of the fetal blood.
A. Hypercapnia
B. Hypoxia
C. Hypoxemia
C. Hypoxemia

_ denotes an increase in carbon dioxide in the fetal blood.
A. Hypercapnia
B. Hypoxia
C. Hypoxemia
A. Hypercapnia

_ denotes an increase in hydrogen ions in the fetal blood.
A. Acidosis
B. Acidemia
C. Hypercapnia
B. Acidemia

_ denotes an increase in hydrogen ions in the fetal tissues.
A. Acidosis
B. Acidemia
C. Hypercapnia
A. Acidosis

occurs when there is low bicarbonate (base excess) in the presence of normal pressure of carbon dioxide (PCO2) values.
A. Metabolic acidosis
B. Respiratory acidosis
C. Metabolic alkalosis
A. Metabolic acidosis

occurs when there is high PCO2 with normal bicarbonate levels.
A. Metabolic acidosis
B. Respiratory acidosis
C. Metabolic alkalosis
B. Respiratory acidosis

occurs when the HCO3 concentration is lower than normal.
A. Base deficit
B. Base excess
C. Metabolic acidosis
A. Base deficit

occurs when the HCO3 concentration is higher than normal.
A. Base deficit
B. Base excess
C. Metabolic acidosis
B. Base excess

_ is defined as the energy-consuming process of metabolism.
Anabolism

_ is defined as the energy-releasing process of metabolism.
Catabolism

T/F: It is an appropriate intervention to perform fetal scalp stimulation during a deceleration.
False

Normal oxygen saturation for the fetus in labor is % to %.
30% to 65%

The normal mean value range for arterial pH is
A. 7.0-7.20
B. 7.20-7.29
C. 7.29-7.39
B. 7.20-7.29

The normal mean value range for arterial PCO2 is
A. 22-24
B. 35-40
C. 49-56
C. 49-56

The normal mean value range for arterial bicarbonate (HCO3) is
A. 22-24
B. 35-40
C. 49-56
A. 22-24

The normal mean value range for arterial base deficit is
A. 2.7-8.3
B. -3.0-2.5
C. 4.2-12.9
A. 2.7-8.3

The normal mean value range for arterial PO2 is
A. 22-24
B. 15-24
C. 49-56
B. 15-24

pH 7.05
PO2 21
PCO2 72
HCO3 24
Base excess -12

A. Metabolic acidosis
B. Respiratory acidosis
C. Mixed acidosis
B. Respiratory acidosis

pH 7.0
PO2 18
PCO2 54
HCO3 20
Base deficit 14

A. Metabolic acidosis
B. Respiratory acidosis
C. Mixed acidosis
A. Metabolic acidosis

pH 7.02
PO2 17
PCO2 72
HCO3 19
Base deficit 16

A. Metabolic acidosis
B. Respiratory acidosis
C. Mixed acidosis
C. Mixed acidosis

Which is a correct description of daily fetal movement counting?
A. Counting should not be started by low-risk mothers until 38 weeks of gestation
B. Fetal movement counting is not reliable because the methods of counting vary
C. The mother counts the number of movements over a specified length of time
C. The mother counts the number of movements over a specified length of time

The BPP includes the assessment of FHR along with what other four components?
A. Amniotic fluid, fetal breathing, fetal movement, fetal tone
B. Amniotic fluid, fetal anomalies, fetal breathing, fetal movement
C. Estimated fetal weight, fetal breathing, fetal movement, placental grading
A. Amniotic fluid, fetal breathing, fetal movement, fetal tone

Which of the following might indicate a potential for chronic fetal hypoxemia?
A. Decreased amniotic fluid volume
B. Increased amniotic fluid volume
C. Intrauterine growth restriction
C. Intrauterine growth restriction

With the finding of a single umbilical artery, what would you expect to observe with Doppler flow studies?
A. Decreased blood perfusion from the fetus to the placenta
B. Decreased blood perfusion from the placenta to the fetus
C. Homeostatic dilation of the umbilical artery
A. Decreased blood perfusion from the fetus to the placenta

Two umbilical arteries flow from the fetus to the placenta

Primary benefits associated with the use of standardized terminology for fetal heart monitoring interpretation include
A. Enhanced communication among health care providers and promotion of patient safety
B. Increased nursing time at the bedside and enhanced patient satisfaction
C. Increased likelihood of correctly diagnosing fetal acidosis during labor
A. Enhanced communication among health care providers and promotion of patient safety

A patient presents with a small amount of thick dark blood clots who denies pain and whose abdomen is soft to the touch. Which component of oxygen transport to the fetus could potentially be compromised by this bleeding?
A. Affinity
B. Saturation
C. Delivery
C. Delivery

What is the most appropriate physiologic goal for a patient whose tracing reveals a sinusoidal baseline?
A. Maximize umbilical circulation
B. Maximize uteroplacental circulation
C. Reduce uterine activity
B. Maximize uteroplacental circulation

Compromised oxygenation – maximize uteroplacental circulation to promote perfusion and oxygenation

When recurrent late decelerations are occurring, the correct physiologic interpretation of this event is
A. Fetal myocardial acidosis is occurring
B. Fetal hypoxemia may be present
C. The fetus has oxygen reserves
B. Fetal hypoxemia may be present

Which intrinsic homeostatic response is the fetus demonstrating when abrupt variable decelerations are present?
A. Baroreceptor
B. Catecholamine
C. Sympathetic
A. Baroreceptor

An amnioinfusion is intended to relieve which extrinsic factor that compromises oxygen transport?
A. Excessive uterine compression
B. Structural abnormalities of the placenta
C. Umbilical cord compression
C. Umbilical cord compression

The nurse wants to document her conversation with the attending physician during an emergent situation. The best approach to documentation would be to
A. Continue providing care for the patient and write a late entry summarizing the conversation after the cesarean is completed
B. Enter an objective transcription of the conversation in the electronic record while colleagues are preparing the patient for surgery
C. Report the conversation to the charge nurse, who can make an entry in the medical record
A. Continue providing care for the patient and write a late entry summarizing the conversation after the cesarean is completed

An infant was delivered via cesarean. Umbilical cord blood gases were: pH 6.88, PCO2 114, PO2 10, bicarbonate 15, base excess (-) 20. The initial neonatal hemocrit was 20% and the hemoglobin was 8.

Which interpretation of these umbilical cord and initial neonatal blood results is correct?
A. Base buffers have been used to maintain oxygenation
B. The mother was probably hypoglycemic
C. The neonate is anemic
C. The neonate is anemic

An infant was delivered via cesarean. Umbilical cord blood gases were: pH 6.88, PCO2 114, PO2 10, bicarbonate 15, base excess (-) 20. The initial neonatal hemocrit was 20% and the hemoglobin was 8.

These umbilical cord blood gases indicate
A. Asphyxia related to umbilical and placental abnormalities
B. Hypoxia related to neurological damage
C. Mixed acidosis
C. Mixed acidosis

An infant was delivered via cesarean. Umbilical cord blood gases were: pH 6.88, PCO2 114, PO2 10, bicarbonate 15, base excess (-) 20. The initial neonatal hemocrit was 20% and the hemoglobin was 8.

Which phrase best describes acidemia?
A. A decrease of oxygen concentration in the blood
B. An increase in hydrogen ions in the blood
C. An increase of hydrogen ions in the tissues
B. An increase in hydrogen ions in the blood

Which compromise in fetal oxygenation could be a result of a post-date pregnancy?
A. Increased saturation capacity
B. Increased fetal oxygen affinity
C. Decreased placental perfusion
C. Decreased placental perfusion

What are the possible implications of an AFI of 3 cm for labor?
A. An amnioinfusion will be needed
B. Increased risk of uterine hyperstimulation
C. Potential umbilical cord compression
C. Potential umbilical cord compression

Which is a characteristic of a negative CST?
A. Absence of late decelerations
B. Recurrent late decelerations
C. Reactive FHR tracing
A. Absence of late decelerations

Which FHR tracing features must be assessed to distinguish arrhythmias from artifact?
A. Shape and regularity of the spikes
B. Spikes and variability
C. Spikes and baseline
A. Shape and regularity of the spikes

How might a fetal arrhythmia affect fetal oxygenation?
A. By increasing fetal oxygen affinity
B. By increasing sympathetic response
C. By reducing fetal perfusion
C. By reducing fetal perfusion

Which medication is used to treat fetal arrhythmias?
A. Digoxin
B. Labetolol
C. Nifedipine
A. Digoxin

Inotropic – promotes regular and effective cardiac contraction

Fetal hydrops may present on ultrasound as fetal scalp edema and increased abdominal fluid as a results of
A. An increase in gestational age
B. Congestive heart failure
C. Sustained oligohydramnios
B. Congestive heart failure

What might increase fetal oxygen consumption?
A. Hyperthermia
B. Umbilical cord compression
C. Uterine tachysystole
A. Hyperthermia

Increases metabolism and oxygen consumption

T/F: Adequacy of uterine resting tone measured by an IUPC should be validated by palpation.
True

Which assessment or intervention would be least appropriate in a patient whose FHR tracing revealed tachycardia and a prolonged deceleration?
A. Change maternal position to right lateral
B. Further assess fetal oxygenation with scalp stimulation
C. Perform a vaginal exam to assess fetal descent
B. Further assess fetal oxygenation with scalp stimulation

Only used with normal baseline rate and never during decels; not an intervention

Which of the following pieces of information would be of highest priority to relay to the neonatal team as they prepare for an emergency cesarean delivery?
A. FHR arrhythmia, meconium, length of labor
B. Gestational age, meconium, arrhythmia
C. Gravidity & parity, gestational age, maternal temperature
B. Gestational age, meconium, arrhythmia

What characteristics are necessary for a reactive NST in a G3P0 patient who is 25 6/7 weeks’ gestation?
A. Accelerations of 15bpm above the baseline lasting at least 15 seconds above the baseline
B. Accelerations of 10bpm above the baseline lasting at least 10 seconds above the baseline
C. Any acceleration greater than 10bpm above the baseline
B. Accelerations of 10bpm above the baseline lasting at least 10 seconds above the baseline

How long may a standard NST be extended in a term gestation if reactivity is not initially demonstrated?
A. From the initial 20 min to 40 min
B. From the initial 20 min to 60 min
C. From the initial 30 min to 60 min
A. From the initial 20 min to 40 min

Accounts for fetal sleep

Which is the most appropriate application of vibroacoustic stimulation (fetal acoustic stimulation) in a standard NST?
A. The device is placed at the top of the maternal fundus
B. The mother uses a marker button to document the fetal movement response
C. The stimulation is applied after a baseline is established
C. The stimulation is applied after a baseline is established

Which of the following characteristics are most common in the preterm fetus?
A. Increased baseline rate and prolonged accelerations
B. Increased baseline rate and variable decelerations
C. Prolonged accelerations and variable decelerations
B. Increased baseline rate and variable decelerations

As well as decreased variability

What typical characteristics of preterm uterine activity may be present in a patient experiencing preterm labor?
A. An irritable uterus with wandering hypertonus
B. Low-amplitude high-frequency contractions
C. Tetanic contractions
B. Low amplitude high-frequency contractions

Which medications used with preterm labor can affect the FHR characteristics?
A. Terbutaline and antibiotics
B. Betamethasone and terbutaline
C. Antibiotics and narcotics
B. Betamethasone and terbutaline

What characterizes a preterm fetal response to stress?
A. More frequently occurring late decelerations
B. More frequently occurring prolonged decelerations
C. More rapid deterioration from Category I to Category II or III
C. More rapid deterioration from Category I to Category II or III

More likely to be subjected to hypoxia

Clinical decision-making at the bedside should include:
A. Communication with the primary care provider only during a crisis
B. Integration of physiologic concepts with maternal-fetal assessment findings
C. Reliance primarily on technology over bedside assessments
B. Integration of physiologic concepts with maternal-fetal assessment findings

In the United States, the paper speed on the monitor is set at
A. 1 cm/min
B. 2 cm/min
C. 3 cm/min
C. 3 cm/min

A woman being monitored externally has a suspected fetal arrhythmia. The most appropriate action is to
A. Insert a spiral electrode and turn off the logic
B. Turn the logic on if an external monitor is in place
C. Use a Doppler to listen to the ventricular rate
A. Insert a spiral electrode and turn off the logic

The ultrasound transducer on the electronic fetal monitor measures the
A. Electrical signal of the fetal heart
B. Mechanical movements of the fetal heart reflected off of sound waves
C. R-to-R intervals of the fetal heart
B. Mechanical movements of the fetal heart reflected off of sound waves

The purpose of autocorrelation in external monitoring is to
A. Compare incoming waveforms for comparison
B. Decrease signal to noise levels
C. Distinguish fetal from maternal HR
A. Compare incoming waveforms for comparison

The area of maximum intensity of the FHR is usually the fetal
A. Back
B. Chest
C. Umbilicus
A. Back

Palpating the uterus is best performed by using the
A. Back of the hand
B. Fingertips
C. Palm
B. Fingertips

What is the most sensitive method of assessing uterine activity?
A. IUPC
B. Manual palpation
C. Maternal perception
A. IUPC

In comparison to maternal blood, the affinity of fetal blood for oxygen is
A. Higher
B. Lower
C. The same
A. Higher

The process that requires energy to accomplish the passage of substances within the intervillous space is
A. Active transport
B. Diffusion
C. Facilitated diffusion
A. Active transport

The umbilical vein carries
A. Carbon dioxide from the fetus back to the placenta
B. Deoxygenated blood from the fetus to the placenta
C. Oxygenated blood from the placenta to the fetus
C. Oxygenated blood from the placenta to the fetus

Fetal hypoxia is best described as a condition of
A. Decreased oxygen in the blood
B. Decreased oxygen in the tissue
C. Increased hydrogen ions in the blood
B. Decreased oxygen in the tissue

Stimulation of the parasympathetic nervous system causes the FHR to
A. Decrease
B. Increase
C. Remain the same
A. Decrease

Chemoreceptors respond mainly to changes in
A. Blood pressure
B. Hormonal levels
C. Oxygen and carbon dioxide levels
C. Oxygen and carbon dioxide levels

The fetus responds to a significant drop of PO2 by
A. Increasing O2 consumption
B. Reducing lactic acid production
C. Shifting blood to vital organs
C. Shifting blood to vital organs

Activation of fetal chemoreceptors results in which FHR change?
A. Decrease
B. Increase
C. No change
A. Decrease

Tachycardia is associated with increased
A. Parasympathetic tone
B. Sympathetic tone
C. Vagal response
B. Sympathetic tone

A patient receiving oxytocin has 17 contractions in 30 minutes. According to the NICHD guidelines, this is called
A. Hyperstimulation
B. Hypertonus
C. Tachysystole
C. Tachysystole

Maternal supine hypotension is caused mainly by compression of the
A. Inferior vena cava
B. Spiral arteries
C. Uterine vessels
A. Inferior vena cava

Which factor influences blood flow to the uterus?
A. Fetal arterial pressure
B. Intervillous space flow
C. Maternal arterial vasoconstriction
C. Maternal arterial vasoconstriction

Betamethasone given to the mother can transiently affect the FHR by
A. Decreasing variability
B. Increasing variability
C. Lowering the baseline
A. Decreasing variability

In a fetal heart rate tracing with marked variability, which of the following is likely the cause?
A. Recent ephedrine administration
B. Recent epidural placement
C. Fetal acidemia
A. Recent ephedrine administration

(T/F) The baseline can be established in a fetal heart rate tracing in which there is marked variability.
False

A fetal heart rate change that can be seen after administration of butorphanol (Stadol) is
A. Bradycardia
B. Marked variability
C. Sinusoidal-appearing
C. Sinusoidal-appearing

One fetal heart rate pattern that is associated with an abnormal acid-base status is
A. Minimal variability with no accelerations or decelerations
B. Recurrent variable decelerations with absent variability
C. Tachycardia with absent variability
B. Recurrent variable decelerations with absent variability

The NICHD definitions are applicable to
A. Antepartum only
B. Antepartum and intrapartum
C. Intrapartum only
B. Antepartum and intrapartum

A wandering FHR baseline may be indicative of
A. Fetal seizure activity
B. Impending fetal death
C. Maternal medication administration
B. Impending fetal death

The FHR pattern that is likely to be seen with maternal hypothermia is
A. Bradycardia
B. Marked variability
C. Tachycardia
A. Bradycardia

Baseline FHR variability is determined in what amount of time, excluding accelerations and decelerations?
A. 10 min
B. 20 min
C. 30 min
A. 10 min

One possible cause of a sinusoidal FHR pattern from fetal hypoxemia is
A. Fetal anemia
B. Fetal thumb sucking
C. Maternal administration of a narcotic
A. Fetal anemia

Which of the following tachyarrhythmias can result in fetal hydrops?
A. Persistent supraventricular tachycardia
B. Premature atrial contractions
C. Sinus tachycardia
A. Persistent supraventricular tachycardia

A preterm fetus with persistent supraventricular tachycardia that is not hydropic is best treated with maternal administration of
A. Digoxin
B. Phenobarbital
C. Terbutaline
A. Digoxin

The maternal medical condition that predisposes the fetus to a heart block is
A. Lupus erythematosus
B. Marfan’s disease
C. Mitral valve prolapse
A. Lupus erythematosus

The initial response in treating a primigravida being induced for preeclampsia who has a seizure is
A. Administer terbutaline to slow down uterine activity
B. Initiate magnesium sulfate
C. Perform an immediate cesarean delivery
B. Initiate magnesium sulfate

Which FHR sounds are counted with a stethoscope and a fetoscope?
A. Atrial
B. Atrial and ventricular
C. Ventricular
C. Ventricular

When using auscultation to determine FHR baseline, the FHR should be counted after the contractions for
A. 5-10 sec
B. 15-30 sec
C. 30-60 sec
C. 30-60 sec

Which of the following characteristics can be determined using FHR auscultation?
A. Baseline
B. Type of decelerations
C. Variability
A. Baseline

One advantage of using a fetoscope is that it can
A. Allow more rapid detection of a baseline change
B. More accurately assess the FHR variability
C. Verify the presence of an irregular rhythm
C. Verify the presence of an irregular rhythm

According to ACOG, intermittent auscultation is appropriate for
A. All pregnancies
B. Neither complicated nor uncomplicated pregnancies
C. Uncomplicated pregnancies
C. Uncomplicated pregnancies

A woman who is 34 weeks’ gestation is counting fetal movements each day. Today she counted eight fetal movements in a two-hour period. Based on her kick counts, this woman should
A. Continue counting for one more hour
B. Discontinue counting until tomorrow
C. Notify her provider for further evaluation
C. Notify her provider for further evaluation

A BPP score of 6 is considered
A. Abnormal
B. Normal
C. Equivocal
C. Equivocal

A modified BPP reveals the following: reactive NST with moderate variability; AFI of 7. This test would be interpreted as
A. Abnormal
B. Equivocal
C. Normal
C. Normal

As fetal hypoxia (asphyxia) worsens, the last component of the BPP to disappear is fetal
A. Breathing
B. Movement
C. Tone
C. Tone

The legal term that describes a failure to meet the required standard of care is
A. Breach of duty
B. Negligence
C. Proximate cause
A. Breach of duty

According to ACOG, in a patient without complications, the FHR tracing during the first stage of labor should be reviewed approximately every
A. 5 min
B. 15 min
C. 30 min
C. 30 min

According to ACOG, in a patient without complications, the FHR tracing during the second stage of labor should be reviewed approximately every
A. 5 min
B. 15 min
C. 30 min
B. 15 min

Since the widespread use of EFM, the rate of cerebral palsy has
A. Decreased
B. Increased
C. Remained the same
C. Remained the same

Regarding the reliability of EFM, there is
A. Good interobserver reliability
B. Good intraobserver reliability
C. Poor interobserver and intraobserver reliability
C. Poor interobserver and intraobserver reliability

The objective of intrapartum FHR monitoring is to assess for fetal
A. Acidemia
B. Oxygenation
C. Well-being
B. Oxygenation

At 32 weeks and beyond, FHR accelerations are defined as increases in FHR that must be at least
A. 10 bpm above the baseline and the accel must last at least 10 sec
B. 15 bpm above the baseline and the accel must last at least 10 sec
C. 15 bpm above the baseline and the accel must last at least 15 sec
C. 15 bpm above the baseline and the accel must last at least 15 sec

Use of the terms “beat-to-beat” variability and “long-term” variability is not recommended by the NICHD because in clinical practice
A. They are visually determined as a unit
B. Both signify an intact cerebral cortex
C. Clinical management is unchanged
A. They are visually determined as a unit

Variable decelerations in FHR are defined as visually apparent, abrupt decreases in FHR. The time from onset to nadir is less than
A. 15 sec
B. 30 sec
C. 2 min
B. 30 sec

According to NICHD definitions of FHR variability, which of the following is accurate?
A. Range 1-5 bpm = absent variability
B. Range 6-25 bpm = average variability
C. Range visually detectable but </=5 bpm = minimal variability
C. Range visually detectable but </=5 bpm = minimal variability

Late decelerations of the FHR are associated most specifically with
A. Transient fetal tissue metabolic acidosis during a contraction
B. Transient fetal hypoxemia during a contraction
C. Transient fetal asphyxia during a contraction
B. Transient fetal hypoxemia during a contraction

Assessment of FHR variability
A. Requires a fetal scalp electrode
B. Includes quantification of beat-to-beat changes
C. Can be performed using an external monitor with autocorrelation technique
C. Can be performed using an external monitor with autocorrelation technique

Decelerations that occur with at least 50% of contractions in a 20-minute window are defined as
A. Repetitive
B. Recurrent
C. Nonreassuring
B. Recurrent

Which setting is most appropriate for fetal vibroacoustic stimulation?
A. 38 weeks, active labor, FHR baseline 140 bpm, minimal variability for 60 min, no accels, no decels
B. 40 weeks, active labor, FHR baseline 150 bpm, moderate variability, prolonged decel to 60 bpm lasting 8 min
C. 34 weeks, frequent contractions without cervical change, FHR baseline 180 bpm, moderate variability, late decels
Which setting is most appropriate for fetal vibroacoustic stimulation?
A. 38 weeks, active labor, FHR baseline 140 bpm, minimal variability for 60 min, no accels, no decels

A complete description of an FHR tracing requires a qualitative and quantitative description of all of the following except
A. Baseline rate
B. Beat-to-beat variability
C. Changes or trends in the FHR patterns over time
B. Beat-to-beat variability

A Category I FHR tracing requires which of the following?
A. Baseline rate 110-160 bpm
B. Accelerations
C. No late, variable, or early decelerations
A. Baseline rate 110-160 bpm

Which of the following would be classified as a Category III FHR tracing?
A. Baseline 180 bpm, minimal variability, no accels, recurrent late decels
B. Baseline rate 140 bpm, absent variability, recurrent late decels
C. Baseline 105 bpm, minimal variability, recurrent variable decels
B. Baseline rate 140 bpm, absent variability, recurrent late decels

Normal frequency of contractions is
A. </= 5 in 10 min averaged over 20 min
B. </= 6 in 10 min averaged over 30 min
C. </= 5 in 10 min averaged over 30 min
C. </= 5 in 10 min averaged over 30 min

Which of the following most closely approximates normal umbilical artery pH at term?
A. 7.0-7.1
B. 7.1-7.2
C. 7.2-7.3
C. 7.2-7.3

The “overshoot” FHR pattern is highly predictive of
A. Fetal hypoxia
B. Preexisting fetal neurological injury
C. None of the above
C. None of the above

A key point regarding the occurrence of tachysystole is that
A. It requires FHR decelerations to be significant
B. It can occur in spontaneous or stimulated labor
C. It should be documented as hyperstimulation if oxytocin is being used
B. It can occur in spontaneous or stimulated labor

A Category II tracing
A. Predicts abnormal fetal acid-base status
B. Excludes abnormal fetal acid-base status
C. Is not predictive of abnormal fetal acid-base status
C. Is not predictive of abnormal fetal acid-base status

Plans of the health care team with a patient with a sinusoidal FHR pattern may include
A. Administration of an NST
B. Administration of tocolytics
C. Kleinhauer-Betke lab test
C. Kleinhauer-Betke lab test

Stimulation of the fetal vagus nerve will
A. Increase FHR
B. Decrease FHR
C. Initially increase, then decrease FHR
B. Decrease FHR

Which of the following is not true when assessing preterm fetuses?
A. FHR baseline may be in upper range of normal (150-160 bpm)
B. They may have fewer accels, and if <35 weeks, may be 10×10
C. Variability may be in lower range for moderate (6-10 bpm)
B. They may have fewer accels, and if <35 weeks, may be 10×10

One of the side effects of terbutaline as a tocolytic is
A. Fetal bradycardia
B. Increased oxygen consumption
C. Marked variability
B. Increased oxygen consumption

Common problems seen during monitoring of postterm fetuses include all of the following except
A. Baseline may be 100-110bpm
B. Increased variables
C. Polyhydramnios
C. Polyhydramnios

Which of the following is not commonly affected by corticosteroids?
A. Doppler flow studies
B. FHR baseline
C. Frequency of FHR accelerations
A. Doppler flow studies

A fetus of a diabetic mother may commonly develop
A. Polyhydramnios
B. Supraventricular tachycardia
C. Third-degree heart block
A. Polyhydramnios

The fetus of a mother with preeclampsia is at high risk for developing
A. Intrauterine growth restriction (IUGR)
B. Macrosomia
C. Polyhydramnios
A. Intrauterine growth restriction (IUGR)

Maternal infection most often results in
A. Fetal tachycardia
B. Moderate variability
C. Placenta previa
A. Fetal tachycardia

Uncontrolled maternal hypertension is often a causative factor in
A. Postmaturity
B. Preterm contractions
C. Placental abruption
C. Placental abruption

EFM is an excellent _ test for determining the presence or absence of neurological injury.
A. Definitive
B. Diagnostic
C. Screening
C. Screening

The primary assumption when using EFM is that _ produces physiological changes that can be detected by changes in the FHR.
A. Disruption of oxygen
B. Labor induction
C. Parity
A. Disruption of oxygen

The primary difference between intermittent auscultation (IA) and EFM is
A. EFM is primarily a visual assessment
B. IA is associated with worse outcomes
C. IA is less labor-intensive than EFM
A. EFM is primarily a visual assessment

The tocodynamometer is completely unreliable for
A. Contraction duration
B. Contraction frequency
C. Contraction intensity
C. Contraction intensity

High resting tone may occur with an IUPC because of all of the following except
A. Extraovular placement
B. Maternal BMI
C. Multiple gestations
B. Maternal BMI

All of the following are disadvantages of the ultrasound transducer for monitoring the FHR except
A. It is noninvasive
B. It may half or double the actual FHR
C. It restricts patient movement
A. It is noninvasive

The internal scalp electrode works by
A. Detecting heart motion
B. Measuring the R-to-R interval of the fetal ECG
C. Sensing the opening and closing of the fetal heart valves
B. Measuring the R-to-R interval of the fetal ECG

When educating the patient about the ultrasound transducer, the nurse should include the following:
A. Contraction strength can alter the ability to monitor the FHR
B. If baby moves, the tracing may “drop” or become erratic
C. The ultrasound detects electrical activity of the fetal heart
B. If baby moves, the tracing may “drop” or become erratic

When educating the patient about the IUPC, the nurse should include the following:
A. Can’t get out of bed
B. Does not accurately measure contraction strength
C. Measures the FHR more accurately
A. Can’t get out of bed

Intrinsic factors are those that are
A. External to the fetus
B. Internal to the fetus
C. Related to the mother
B. Internal to the fetus

The parasympathetic branch of the autonomic nervous system
A. Decreases FHR
B. Increases contractility of the fetal heart
C. Increases FHR
A. Decreases FHR

Which of the following is the primary neurotransmitter of the sympathetic branch of the autonomic nervous system?
A. Acetylcholine
B. Dopamine
C. Norepinephrine
C. Norepinephrine

Which of the following is the primary neurotransmitter of the parasympathetic branch of the autonomic nervous system?
A. Acetylcholine
B. Dopamine
C. Norepinephrine
A. Acetylcholine

Which of the following is responsible for variations in the FHR and fetal behavioral states?
A. Cerebellum
B. Cerebral cortex
C. Medulla oblongata
B. Cerebral cortex

Extrinsic factors affecting the FHR include
A. Fetal baroreceptors
B. Fetal chemoreceptors
C. Maternal oxygen transport
C. Maternal oxygen transport

When the umbilical vessels traverse the membranes to the placenta without any cord protection, this is called
A. Placenta previa
B. Succenturiate lobe (SL)
C. Velamentous insertion
C. Velamentous insertion

Which of the following is the primary factor in uteroplacental blood flow?
A.. Fetal heart rate
B. Maternal cardiac output
C. Maternal oxygen consumption
B. Maternal cardiac output

Chemoreceptors respond to changes in
A. Blood pressure
B. Pulse
C. Oxygenation
C. Oxygenation

Which of the following occurs when the parasympathetic branch of the autonomic nervous system is stimulated?
A. Norepinephrine release
B. Slowed conduction to sinoatrial node
C. Increase in fetal heart rate
B. Slowed conduction to sinoatrial node

Which of the following is responsible for fetal muscle coordination?
A. Cerebellum
B. Cerebral cortex
C. Medulla oblongata
A. Cerebellum

When the placenta covers the cervical os, this is called
A. Placenta previa
B. Succenturiate lobe (SL)
C. Velamentous insertion
A. Placenta previa

An additional placental lobe is defined as
A. Placenta previa
B. Succenturiate lobe (SL)
C. Velamentous insertion
B. Succenturiate lobe (SL)

During periods of fetal tachycardia, FHR variability is usually diminished due to
A. The dominance of the parasympathetic nervous system
B. The dominance of the sympathetic nervous system
C. Stimulation of the fetal vagus nerve
A. The dominance of the parasympathetic nervous system

The presence of moderate FHR variability is most specifically an excellent predictor of
A. Absence of fetal metabolic acidosis
B. Fetal movement
C. Fetal oxygenation
A. Absence of fetal metabolic acidosis

All of the following can be used to describe episodic FHR changes except
A. Accelerations
B. Late decelerations
C. Variable decelerations
B. Late decelerations

Periodic accelerations can indicate all of the following except
A. Stimulation of fetal chemoreceptors
B. Tracing is a maternal tracing
C. Umbilical vein compression
A. Stimulation of fetal chemoreceptors

A transient decrease in cerebral blood flow (increased cerebral blood pressure) during a contraction may stimulate _ and may cause a(n) _
A. Baroreceptors; early deceleration
B. Baroreceptors; late deceleration
C. Chemoreceptors; early deceleration
A. Baroreceptors; early deceleration

The primary physiologic goal of interventions for late decelerations is to
A. Decrease maternal oxygen consumption
B. Maximize placental blood flow
C. Maximize umbilical circulation
B. Maximize placental blood flow

Which of the following is most responsible for producing FHR variability as the fetus grows?
A. Maturation of the parasympathetic nervous system
B. Maturation of the sympathetic nervous system
C. Release of maternal prostaglandins
A. Maturation of the parasympathetic nervous system

Using the NICHD terminology, tachysystole is defined as
A. More than 5 contractions in 10 minutes averaged over a 20-minute period
B. More than 5 contractions in 15 minutes averaged over a 30-minute period
C. More than 5 contractions in 10 minutes averaged over a 30-minute period
C. More than 5 contractions in 10 minutes averaged over a 30-minute period

When a patient reports that she is feeling contractions but the nurse does not note any on the toco tracing, what is the first intervention that should be performed?
A. Ask the provider to place an IUPC
B. Reposition the toco
C. Palpate the uterus
C. Palpate the uterus

Which of the following represents the normal range for FHR baseline?
A. 120-160 bpm
B. 110-160 bpm
B. 110-160 bpm

Which of the following is not a type of supraventricular dysrhythmia?
A. Premature atrial contraction (PAC)
B. Premature ventricular contraction (PVC)
C. Supraventricular tachycardia (SVT)
B. Premature ventricular contraction (PVC)

Which is the most common type of fetal dysrhythmia?
A. Premature atrial contraction (PAC)
B. Premature ventricular contraction (PVC)
C. Third-degree heart block
A. Premature atrial contraction (PAC)

What is the most common cause of third-degree heart block in the fetus?
A. Maternal HIV infection
B. Maternal lupus
C. Tocolysis
B. Maternal lupus

All of the following are traits of fetal supraventricular tachycardia (SVT), but which is most problematic?
A. Decreases diastolic filling time
B. Dramatically increases oxygen consumption
C. Often leads to ventricular tachycardia (VT)
C. Often leads to ventricular tachycardia (VT)

Which abnormal FHR pattern is most likely to lead to hydrops in the fetus?
A. Marked variability
B. A premature ventricular contraction (PVC)
C. Supraventricular tachycardia (SVT)
C. Supraventricular tachycardia (SVT)

Which of the following is an irregular FHR pattern associated with normal conduction and rate?
A. Premature atrial contractions (PACs)
B. Sinus arrhythmias
C. Sinus tachycardias
B. Sinus arrhythmias

Which method of assessing FHR allows the clinician to hear the opening and closing of heart valves, which helps detect dysrhythmias?
A. A fetoscope
B. An internal scalp electrode (FSE)
C. An ultrasound transducer
A. A fetoscope

Which method of assessment allows the clinician to visually see cardiac wall movements?
A. A fetal echocardiogram
B. A fetal pulse oximetry reading
C. A nonstress test (NST)
A. A fetal echocardiogram

_ denotes the spontaneous, rhythmic depolarization of cardiac cells.
A. Automaticity
B. Excitability
C. Conductivity
A. Automaticity

_ denotes the readiness of cardiac cells to receive and respond to electrical stimuli.
A. Automaticity
B. Excitability
C. Conductivity
B. Excitability

_ denotes the ability of cardiac cells to conduct electrical impulses from one cell to another.
A. Automaticity
B. Excitability
C. Conductivity
C. Conductivity

Which of the following is the most common type of fetal tachyarrhythmia?
A. Atrial flutter
B. Supraventricular tachycardia (SVT)
C. Premature ventricular contraction (PVC)
B. Supraventricular tachycardia (SVT)

Which of the following is one example of a fetal tachyarrhythmia?
A. Second-degree heart block, Type I
B. Atrial fibrillation
C. Premature atrial contraction (PAC)
B. Atrial fibrillation

(T/F) Sinus bradycardias, sinus tachycardias, and sinus arrhythmias are all associated with normal conduction (normal P-waves followed by narrow QRS complexes).
True

(T/F) An internal scalp electrode will detect the actual fetal ECG.
True

(T/F) An internal scalp electrode can solely diagnose a fetal dysrhythmia.
False

_ are patterns of abnormal FHR associated with variability in R-to-R intervals, but with normal P-waves preceding normal QRS complexes.
A. Arrhythmias
B. Complete heart blocks
C. Dysrhythmias
A. Arrhythmias

_ are abnormal FHR rhythms associated with disordered impulse formation, conduction, or both.
A. Arrhythmias
B. Supraventricular tachycardias
C. Dysrhythmias
C. Dysrhythmias

Which of the following fetal dysrhythmias may be related to maternal hyperthyroidism?
A. Sinus tachycardia
B. Premature atrial contractions (PACs)
C. Third-degree heart block
B. Premature atrial contractions (PACs)

With _ premature ventricular contractions (PVCs), the baseline and variability are obscured.
A. Idioventricular
B. Bigeminal
C. Trigeminal
B. Bigeminal

With _ premature ventricular contractions (PVCs), the upward spikes will be slightly longer than the downward spikes.
A. Idioventricular
B. Bigeminal
C. Trigeminal
C. Trigeminal

Which of the following dysrhythmias may progress to atrial fibrillation or atrial flutter?
A. Premature atrial contractions (PACs)
B. Supraventricular tachycardia (SVT)
C. Sinus tachycardia
B. Supraventricular tachycardia (SVT)

Which of the following is not commonly caused by administration of indomethacin?
A. Decreased fetal urine (decreased amniotic fluid index [AFI])
B. Decreased FHR baseline
C. Increased variable decelerations
B. Decreased FHR baseline

Which of the following is not commonly caused by terbutaline administration?
A. Increased FHR baseline
B. Decreased FHR late decelerations
C. Increased maternal HR
B. Decreased FHR late decelerations

Which of the following is not commonly caused by nifedipine administration?
A. Maternal hypotension
B. Decreased uterine blood flow
C. Increased FHR accelerations
C. Increased FHR accelerations

Which of the following would likely be affected by betamethasone administration?
A. Fetal echocardiogram
B. Biophysical profile (BPP) score
C. Contraction stress test (CST)
B. Biophysical profile (BPP) score

Fetal breathing decreased with betamethasone administration

Which of the following is a common EFM finding when monitoring a preterm fetus?
A. Decreased baseline
B. Variable decelerations
C. Increased FHR variability
B. Variable decelerations

Which of the following is most strongly correlated with placental abruption?
A. Uncontrolled gestational diabetes
B. Multiple gestation
C. Uncontrolled hypertension
C. Uncontrolled hypertension

Which of the following is not typically associated with a postterm pregnancy?
A. Meconium-stained amniotic fluid
B. Presence of late decelerations in the fetal heart rate
C. Polyhydramnios
C. Polyhydramnios

Which of the following is the most appropriate method of monitoring a patient who is a gestational diabetic?
A. Daily NSTs
B. Twice-weekly BPPs
C. Weekly contraction stress tests
B. Twice-weekly BPPs

Which of the following is not commonly caused by magnesium sulfate?
A. Increased FHR baseline
B. Decreased FHR variability
C. Decreased FHR accelerations
A. Increased FHR baseline

Which of the following FHR changes would not typically be seen in a mother with chorioamnionitis?
A. Increased baseline
B. Increased variability
C. Decreased accelerations
B. Increased variability

Which of the following is false regarding electronic fetal monitoring of twins?
A. Both twins must be monitored, as opposed to monitoring one twin at a time
B. Twins rarely have accelerations and decelerations simultaneously, and tracings should appear distinctly different
C. Twins must be identified and monitored as A or B throughout the entire antenatal and intrapartum periods
B. Twins rarely have accelerations and decelerations simultaneously, and tracings should appear distinctly different

pH 6.9, PO2 15, PCO2 55, HCO3 18, BE -22
The nurse reviews the arterial gas results and concludes that the fetus had _ acidosis. With results such as these, you would expect a _ resuscitation.
A. Metabolic; lengthy
B. Metabolic; short
C. Respiratory; lengthy
A. Metabolic; lengthy

Which of the following umbilical artery cord gases would most likely result in a fetus who had a Category I strip, then had a cord prolapse and was delivered within 3 minutes?
A. pH 7.17, PO2 22, PCO2 70, HCO3 24, BE -5
B. pH 7.25, PO2 23, PCO2 46, HCO3 22, BE -8
C. pH 7.02, PO2 18, PCO2 56, HCO3 15, BE -18
A. pH 7.17, PO2 22, PCO2 70, HCO3 24, BE -5

Cord blood gases from one of the umbilical arteries best represent
A. The status of fetal acid-base balance
B. The status of maternal oxygenation
C. The status of placental function
A. The status of fetal acid-base balance

A modified biophysical profile (mBPP) is considered normal if
A. NST is reactive and mother reports at least 10 fetal movements in 2 hours
B. NST is reactive and the amniotic fluid index (AFI) is greater than 5 cm
C. There are no decelerations following contractions
B. NST is reactive and the amniotic fluid index (AFI) is greater than 5 cm

Which of the following is not a component of a BPP?
A. Amniotic fluid index (AFI)
B. Fetal breathing
C. Nonstress test (NST)
A. Amniotic fluid index (AFI)

(Should be volume, not index)

A contraction stress test (CST) is done. During the testing, there were three contractions in 10 minutes, no decels, the baseline was 145, and there was moderate variability. This is interpreted as
A. Negative
B. Positive
C. Suspicious
A. Negative

Which of the following is most likely to result in absent end diastolic flow during umbilical artery velocimetry?
A. Preeclampsia
B. Preterm labor
C. Previous cesarean delivery
A. Preeclampsia

Which of the following best describes a condition in which there is decreased oxygen in the tissues?
A. Acidosis
B. Hypoxemia
C. Hypoxia
C. Hypoxia

What is the maximum amount of time allotted for a nonstress test?
A. 20 minutes
B. 40 minutes
C. 60 minutes
B. 40 minutes

A nonstress test (NST) is nonreactive after 40 minutes. The nurse should
A. Call the physician to plan for a possible induction
B. Call the physician to arrange for a BPP or CST
C. Perform vibroacoustic stimulation and monitor patient for 20 additional minutes
B. Call the physician to arrange for a BPP or CST

(T/F) Contractions during a contraction stress test (CST) may be spontaneous or induced with oxytocin or nipple stimulation.
True

A contraction stress test (CST) is performed. Late decelerations were noted in three out of the five contractions in 10 minutes. This is interpreted as
A. Positive
B. Negative
C. Suspicious
A. Positive

A contraction stress test (CST) is performed. Late decelerations were noted in two out of the five contractions in 10 minutes. This is interpreted as
A. Positive
B. Negative
C. Suspicious
C. Suspicious

A contraction stress test (CST) is performed. Two variable decelerations were seen on the FHR tracing and there were four contractions in 10 minutes. This is interpreted as
A. Positive
B. Negative
C. Suspicious
B. Negative

A contraction stress test (CST) is performed. No decelerations were noted with the two contractions that occurred over 10 minutes. This is interpreted as
A. Positive
B. Negative
C. Unsatisfactory
C. Unsatisfactory

A patient has a reactive NST and negative CST. Select the appropriate followup.
A. Prepare patient for delivery
B. Repeat in 24 hours
C. Repeat in one week
C. Repeat in one week

A patient has a nonreactive NST and negative CST. Select the appropriate followup.
A. Prepare patient for delivery
B. Repeat in 24 hours
C. Repeat in one week
B. Repeat in 24 hours

For a patient with a suspicious or unsatisfactory CST, select the appropriate followup.
A. Prepare patient for delivery
B. Repeat in 24 hours
C. Repeat in one week
B. Repeat in 24 hours

In a patient whose CST reveals late decelerations with three out of the four induced contractions, which of the following would be the least appropriate plan for treatment?
A. Obtain physician order for BPP
B. Prepare for possible induction of labor
C. Repeat CST in 24 hours
C. Repeat CST in 24 hours

For a patient at 38 weeks’ gestation with a BPP score of 6, select the most appropriate course of action.
A. Repeat in one week
B. Consider induction of labor
C. Prepare for cesarean delivery
B. Consider induction of labor

For a patient at 35 weeks’ gestation with a BPP score of 4, select the most appropriate course of action.
A. Repeat in 24 hours
B. Obtain physician order for CST
C. Prepare for probable induction of labor
C. Prepare for probable induction of labor

(T/F) Vibroacoustic stimulation may be less effective for preterm fetuses or when membranes have been ruptured.
True

_ cord blood sampling is predictive of uteroplacental function.
A. Arterial
B. Venous
C. Maternal
B. Venous

(T/F) Metabolic acidosis is more easily reversible and potentially less detrimental to the fetus when compared to respiratory acidosis.
False

(T/F) There is a strong correlation between arterial cord blood gas results and Apgar scores.
False

Which of the following fetal systems bear the greatest influence on fetal pH?
A. Heart and lungs
B. Lungs and kidneys
C. Sympathetic and parasympathetic nervous systems
B. Lungs and kidneys

Your patient is a 41-year-old diabetic primigravida. You have just performed a vaginal exam: 6/90/-1. The fetus has been having late decels with absent to minimal variability and no accels for one hour, despite interventions to improve the tracing. The provider arrives on the unit and states that she will remain on the floor, and writes orders to start oxytocin. You inform the physician that you are not comfortable starting oxytocin based on maternal history, SVE, and FHR tracing. The provider insists that you should start oxytocin. What is your response?
A. Tell the physician that if she will remain on the unit, you will start the oxytocin.
B. Tell the physician that she can start the oxytocin herself.
C. Tell the physician that you decline to start the oxytocin, and notify the physician that you are activating your chain of command.
C. Tell the physician that you decline to start the oxytocin, and notify the physician that you are activating your chain of command.

All of the following are components of liability except
A. Breach of duty
B. Chain of command
C. Damages/loss
B. Chain of command

If the EFM pattern you see does not fit any of the definitions for the NICHD, you should
A. Choose the term closest to the pattern you see
B. Describe the tracing in detail
C. Do not chart the tracing, as there is no correlating terminology to match it
B. Describe the tracing in detail

When using SBAR communication, the “R” involves
A. Making a recommendation
B. Reporting pertinent lab results
C. Reviewing the patient’s medical history
A. Making a recommendation

Which of the following is considered using the chain of command?
A. Asking the patient to talk with the provider directly
B. Notifying the charge nurse
C. Writing a variance report
B. Notifying the charge nurse

Elements of a malpractice claim include all of the following except
A. Breach of duty
B. Deposition
C. Injury or loss
B. Deposition

Ductus venosus connects _ to _?
Umb. vein to IVC

Foramen ovale connects _to?
R. atrium to L. atrium

Ductus arteriosus connects to ?
Pulm. artery to aorta

Normal rate
110 to 160 BPM

Average baseline rate at 15wks gestation
160 BPM

Average rate in fetus with heart block
60 BPM (intrinsic ventricular or nodal rate)

Sympathetic effects
Incr. HR, constricted peripheral and visceral blood vessels, vital organs dilate and receive more blood

Gest. age at which autonomic nervous system is fully developed
~ 32wks

Primary indicator of fetal oxygenation
Presence or absence of variability

Location of chemoreceptors
aortic arch

Chemoreceptors respond to?
Changes in O2 and CO2

Location of baroreceptors
Aortic arch and carotid bodies

Maximal reactivity occurs when?
Late at night

Max duration of fetal nonreactivity?
80min

Sensitivity
Probablity of detecting a true positive

Specificity
Probability that true negative will be detected

False positive
erroneosly positive when reality is negative

False negative
erroneously negative when reality is positive

NST
Two 15×15 accels in 20min (up to 40min)

% of false positive CSTs?
~ 30%

BPP components
1) Fetal breathing movements
2) Gross body movements
3) Fetal tone
4) Reactive fetal heart rate
5) Qualitiative amniotic fluid volume

BPP Score of 10
Normal

BPP Score of 8
Nl infant, low risk of chronic asphyxia

BPP Score of 6
Borderline result – rpt in 24hrs

BPP Score of 4
Suspect chronic asphyxia

BPP Score of 0-2
Strongly suspect chronic asphyxia

Early manifestation of fetal acidosis?
Non-reacive NST and loss of FBM

Change in FBM with maternal glucose changes?
FBM incr. with rising mGlu
FBM decr. with mHYPOglycemia

Maternal smoking effects FBM how?
FBM decr. with msmoking

Corticosteroids effects FBM?
corticosteroids may decr. FBM

Maternal PAO2 at sea level?
~105mmHg

Change in cardiac output and SVR in pre-E?
CO and SVR INCREASED in pre-E

Prolonged decels occured in _% of uterine rupture?
71%

Signif. neonatal morbidity occured when ____min or longer elapsed between onset of HFR decel and delivery.
18min

Normal arterial pH for healthy vaginal deliveries?
7.28+/- 0.05

Normal arterial pO2 for healthy vaginal deliveries?
18.0 +/- 6.2

Normal arterial pCO2 for healthy vaginal deliveries?
49.2 +/- 8.4

Normal base deficit for healthy vaginal deliveries?
Less than 12

pH at which acidosis
7.2 (7.1)

Tachysystole

5 contractions in 10min

Baseline (defn)
mean FHR rounded to increments of 5 BPM during a 10min perioud excl. periodic or episodic changes, contractions, periods of marked variability, and segments of baseline that differ by >25 BPM

Sinusoidal pattern?
Fetal anemia

As many as _% of infants with complete heart block have assoc. congenital cardiac malformations.
50%

Mortality rate for newborns with complete heart block
25%

Newborn with complete heart block in absence of congen. heart disease frequently has neonatal _.
Lupus erthematosus

Baseline variability (defn)
Fluctuations in baseline FHR >2 cycles per minute (peak to trough); irregular in amplitude and frequency

Minimal baseline variability

undetectable but <or= 5bpm

Moderate baseline variability
6-25 bpm

Marked variability

25 bpm

T/F Performing fetal stimulation is appropriate during decels or bradycardia.
FALSE – Fetal stim should be performed when FHR is at the baseline.

Early Decel (defn)
visually apparrent GRADUAL decrease (onset to nadir >=30sec of FHR below baseline), nadir occurs at peak of contraction

Early decel (physiology)
Fetal head copmression (vagal reflex), often sen between 4-6cm dilation

Which of the following factors can have a negative effect on uterine blood flow?
a. Hypertension
b. Epidural
c. Hemorrhage
d. Diabetes
e. All of the above
e. All of the above

Stimulating the vagus nerve typically produces:
a. A decrease in the heart rate
b. An increase in the heart rate
c. An increase in stroke volume
d. No change
a. A decrease in the heart rate

The vagus nerve begins maturation 26 to 28 weeks. Its dominance results in what effect to the FHR baseline?
a. Increases baseline
b. Decreases baseline
b. Decreases baseline

T/F: The most common artifact with the ultrasound transducer system for fetal heart rate is increased variability.
True

T/F: All fetal monitors contain a logic system designed to reject artifact.
True

T/F: Fetal arrhythmias can be seen on both internal and external monitor tracings.
True

T/F: Variability and periodic changes can be detected with both internal and external monitoring.
True

T/F: Variable decelerations are a vagal response.
True

T/F: Variable decelerations are the most frequently seen fetal heart rate deceleration pattern in labor.
True

Etiology of a baseline FHR of 165bpm occurring for the last hour can be:

  1. Maternal supine hypotension
  2. Maternal fever
  3. Maternal dehydration
  4. Unknown
    a. 1 and 2
    b. 1, 2 and 3
    c. 2, 3 and 4
    c. 2, 3 and 4

The most prevalent risk factor associated with fetal death before the onset of labor is:
a. Low socioeconomic status
b. Fetal malpresentation
c. Uteroplacental insufficiency
d. Uterine anomalies
c. Uteroplacental insufficiency

Which of the following conditions is not an indication for antepartum fetal surveillance?
a. Gestational hypertension
b. Diabetes in pregnancy
c. Fetus in breech presentation
d. Decreased fetal movement
c. Fetus in breech presentation

Which of the following does not affect the degree of fetal activity?
a. Vibroacoustic stimulation
b. Smoking
c. Fetal position
d. Gestational age
a. Vibroacoustic stimulation

T/F: Umbilical cord influences that can alter blood flow include true knots, hematomas, and number of umbilical vessels.
True

T/F: Low amplitude contractions are not an early sign of preterm labor.
False

T/F: Corticosteroid administration may cause an increase in FHR accelerations.
False

T/F: Corticosteroid administration may cause an increase in FHR.
True

T/F: Contractions cause an increase in uterine venous pressure and a decrease in uterine artery perfusion.
True

As a result of the intrinsic fetal response to oxygen deprivation, increased catecholamine levels cause the peripheral blood flow to decrease while the blood flow to vital organs increases. These flow changes along with increased catecholamine secretions have what effect on fetal blood pressure and fetal heart rate?
A. Increase BP and increase HR
B. Increase BP and decrease HR
C. Decrease BP and increase HR
D. Decrease BP and decrease HR
B. Increase BP and decrease HR

During a term antepartum NST (non-stress test), you notice several variable decelerations that decrease at least 15 bpm and last at least 15 sec long. Which of the following is the least likely explanation?
A. True knot
B. Gestational diabetes
C. Umbilical cord entanglement
D. Oligohydramnios
B. Gestational diabetes

All of the following are likely causes of prolonged decelerations except:
A. Uterine tachysystole
B. Prolapsed cord
C. Maternal hypotension
D. Maternal fever
D. Maternal fever

All of the following could likely cause minimal variability in FHR except
A. Magnesium sulfate administration
B. Fetal sleep cycle
C. Narcotic administration
D. Ephedrine administration
D. Ephedrine administration

When an IUPC has been placed, Montevideo units must be _ or greater for adequate cervical change to occur.
A. 100
B. 200
C. 300
D. 400
B. 200

What would be a suspected pH in a fetus whose FHTs included recurrent late decelerations during labor?
A. 7.10
B. 7.26
C. 7.32
D. 7.41
A. 7.10

The nurse notes a pattern of decelerations on the fetal monitor that begins shortly after the contraction and returns to baseline just before the contraction is over. The correct nursing response is to:
a. Give the woman oxygen by facemask at 8-10 L/min
b. Position the woman on her opposite side
c. Increase the rate of the woman’s intravenous fluid
d. Continue to observe and record the normal pattern
d. Continue to observe and record the normal pattern

Determining the FHR baseline requires the nurse to approximate the mean FHR rounded to increments of 5 bpm during a _-minute window (excluding accelerations and decelerations).
A. 2
B. 5
C. 10
D. 20
C. 10

Which of the following interventions would best stimulate an acceleration in the FHR?
A. Provide juice to patient
B. Perform vaginal exam
C. Turn patient on left side
D. Vibroacoustic stimulation
B. Perform vaginal exam

Scalp stimulation

The FHR is controlled by the
A. Sympathetic nervous system
B. Sinoatrial node
C. Atrioventricular node
D. Parasympathetic nervous system
B. Sinoatrial node

T/F: Fetal tachycardia is a normal compensatory response to transient fetal hypoxemia.
True

At how many weeks gestation should FHR variability be normal in manner?
A. 24 weeks
B. 28 weeks
C. 32 weeks
D. 36 weeks
B. 28 weeks

Reduced respiratory gas exchange from persistent decelerations may cause a rise in fetal PCO2, which leads first to , then .
A. Respiratory alkalosis; metabolic acidosis
B. Respiratory acidosis; metabolic acidosis
C. Respiratory alkalosis; metabolic alkalosis
D. Respiratory acidosis; metabolic acidosis
B. Respiratory acidosis; metabolic acidosis

Decreased intervillious exchange of oxygenated blood resulting in fetal hypoxia is typically present in _.
A. Variable decelerations
B. Late decelerations
C. Early decelerations
D. Accelerations
B. Late decelerations

Place the following interventions for a sinusoidal FHR in the correct order:

  1. Prepare for cesarean delivery
  2. Place patient in lateral position
  3. Determine if pattern is related to narcotic analgesic administration
  4. Provide oxygen via face mask
    A. 4, 2, 3, 1
    B. 3, 1, 2, 4
    C. 4, 3, 2, 1
    D. 3, 2, 4, 1
    D. 3, 2, 4, 1

FHTs with accelerations, no decelerations, and minimal variability would be categorized as
A. Category I
B. Category II
C. Category III
B. Category II

FHTs with minimal variability, absent accelerations, and a 3-minute prolonged deceleration would be categorized as
A. Category I
B. Category II
C. Category III
B. Category II

Which of the following is not a likely cause of a sinusoidal FHR pattern?
A. Chronic fetal bleeding
B. Fetal hypoxia or anemia
C. Triple screen positive for Trisomy 21
D. Fetal isoimmunization
C. Triple screen positive for Trisomy 21

Which of the following factors is not likely to cause uteroplacental insufficiency?
A. Late-term gestation
B. Preeclampsia
C. Gestational diabetes
D. Polyhydramnios
E. Maternal smoking or drug use
D. Polyhydramnios

The normal FHR baseline
A. Decreases during labor
B. Fluctuates during labor
C. Increases during labor
B. Fluctuates during labor

Bradycardia in the second stage of labor following a previously normal tracing may be caused by fetal
A. Hypoxemia
B. Rotation
C. Vagal stimulation
C. Vagal stimulation

Clinically significant fetal metabolic academia is indicated by an arterial cord gas pH of less than or equal to 7.10 and a base deficit of
A. 3
B. 6
C. 12
C. 12

Fetal bradycardia can result during
A. The sleep state
B. Umbilical vein compression
C. Vagal stimulation
C. Vagal stimulation

While caring for a 235-lb laboring woman who is HIV-seropositive, the external FHR tracing is difficult to obtain. An appropriate nursing action would be to
A. Apply a fetal scalp electrode
B. Auscultate for presence of FHR variability
C. Notify the attending midwife or physician
C. Notify the attending midwife or physician

Which IV fluid is most appropriate for maternal administration for intrauterine resuscitation?
A. Lactated Ringer’s solution
B. D5L/R
C. Normal saline
C. Normal saline

An EFM tracing with absent variability and no decelerations would be classified as
A. Category I
B. Category II (indeterminate)
C. Category III
B. Category II (indeterminate)

An EFM tracing with absent variability and intermittent late decelerations would be classified as
A. Category I
B. Category II
C. Category III
B. Category II

Maternal oxygen administration is appropriate in the context of
A. Recurrent variable decelerations/moderate variability
B. Intermittent late decelerations/minimal variability
C. Prolonged decelerations/moderate variability
B. Intermittent late decelerations/minimal variability

In the context of hypoxemia, fetal blood flow is shifted to the
A. Brain
B. Liver
C. Lungs
A. Brain

Baroreceptor-mediated decelerations are
A. Early
B. Late
C. Variable
C. Variable

An appropriate initial treatment for recurrent late decelerations with moderate variability during first stage labor is
A. Amnioinfusion
B. Maternal repositioning
C. Oxygen at 10L per nonrebreather face mask
B. Maternal repositioning

Most fetal dysrhythmias are not life-threatening, except for _, which may lead to fetal congestive heart failure.
Supraventricular tachycardia

Medications, prematurity, fetal sleep, fetal dysrhythmia, anesthetic agents, or cardiac anomalies may result in _ variability.
Decreased

Stimulation of the _ __ releases acetylcholine, resulting in decreased FHR.
Parasympathetic nervous system

The _ __ maintains transmission of beat-to-beat variability.
Parasympathetic nervous system

Stimulation of the _ __ releases catecholamines, resulting in increased FHR.
Sympathetic nervous system

Stimulation of _ results in abrupt decreases in FHR, CO, and BP.
Baroreceptors

Baroreceptors influence _ decelerations with moderate variability.
Variable

In comparing early and late decelerations, a distinguishing factor between the two is
A. Onset time to the nadir of the deceleration
B. The number of decelerations that occur
C. Timing in relation to contractions
C. Timing in relation to contractions

The underlying cause of early decelerations is decreased
A. Baroceptor response
B. Increased peripheral resistance
C. Vagal reflex
C. Vagal reflex

Glucose is transferred across the placenta via _ _.
Facilitated diffusion

Oxygen, carbon dioxide, water, electrolytes, urea, uric acid, fatty acids, fat-soluble vitamins, narcotics barbiturates, anesthetics, and antibiotics are transferred across the placenta via _ _.
Simple (passive) diffusion

Amino acids, water-soluble vitamins, calcium, phosphorus, iron, and iodine are transferred across the placenta via _ _.
Active transport

Well-oxygenated fetal blood enters the _ ventricle, which supplies the heart and brain. Less-oxygenated blood enters the __ ventricle, which supplies the rest of the body.
Left; right

Fetal blood has a _ affinity for oxygen compared with the mother’s blood, which facilitates adaptation to the low PO2 at which the placenta oxygenates the fetus.
A. Higher
B. Lower
A. Higher

The fetus has a _ cardiac output and heart rate than the adult, resulting in rapid circulation.
A. Higher
B. Lower
A. Higher

Which statement best describes the relationship between maternal and fetal hemoglobin levels?
A. Fetal hemoglobin is higher than maternal hemoglobin
B. Maternal hemoglobin is higher than fetal hemoglobin
C. Maternal and fetal hemoglobin are the same
A. Fetal hemoglobin is higher than maternal hemoglobin

A 36 week gestation patient is brought to triage by squad after an MVA on her back. She is not bleeding and denies pain. She is not short of breath, but c/o dizziness and nausea since they put her on the gurney. The most likely cause is
A. Abruptio placenta
B. Preterm labor
C. Supine hypotension
C. Supine hypotension

When the hydrogen ion content in the blood rises, the pH
A. Lowers
B. Neutralizes
C. Rises
A. Lowers

***A woman receives terbutaline for an external version. You may expect what on the fetal heart tracing?
A. Decrease in variability
B. Increase in baseline
C. No change
B. Increase in baseline

What affect does magnesium sulfate have on the fetal heart rate?
A. Decreases variability
B. Increases variability
C. No change
A. Decreases variability

Sinusoidal pattern can be documented when
A. Cycles are 4-6 beats per minute in frequency
B. The pattern lasts 20 minutes or longer
C. There is moderate or minimal variability
B. The pattern lasts 20 minutes or longer

Vagal stimulation would be manifested as what type of fetal heart rate pattern?
A. Acceleration
B. Early deceleration
C. Tachycardia
B. Early deceleration

Which fetal monitoring pattern is characteristic of cephalopelvic disproportion, especially when seen at the onset of labor?
A. Early deceleration
B. Late deceleration
C. Variable deceleration
A. Early deceleration

A risk of amnioinfusion is
A. Prolonged labor
B. Uterine overdistension
C. Water intoxication
B. Uterine overdistension

A fetal heart rate pattern that can occur when there is a prolapsed cord is
A. Marked variability
B. Prolonged decelerations
C. Tachycardia
B. Prolonged decelerations

The patient is in early labor with pitocin at 8 mu/min, and FHR is Category I. In the next 15 minutes, there are 18 uterine contractions. Recommended management is to
A. Address contraction frequency by reducing pitocin dose
B. Continue to increase pitocin as long as FHR is Category I
C. Turn the patient on her side and initiate an IV fluid bolus
C. Turn the patient on her side and initiate an IV fluid bolus

A woman at 38 weeks gestation is in labor. The labor has been uneventful, and the fetal heart tracings have been normal. Spontaneous rupture of membranes occurs; fetal heart rate drops to 90 beats per minute for four minutes and then resumes a normal pattern. The most likely etiology for this fetal heart rate change is
A. Abnormal fetal presentation
B. Impaired placental circulation
C. Possible cord compression
C. Possible cord compression

A woman has 10 fetal movements in one hour. This is considered what kind of movement?
A. Decreased
B. Excessive
C. Normal
C. Normal

If the pH is low, what other blood gas parameter is used to determine if the acidosis is respiratory or metabolic?
A. HCO3
B. PCO2
C. PO2
B. PCO2

The following cord blood gasses are consistent with: pH 7.10, pCO2 70, pO2 25, base excess -10
A. Metabolic acidosis
B. Mixed acidosis
C. Respiratory acidosis
C. Respiratory acidosis

As a contraction beings, partial umbilical cord compression causes occlusion of the low-pressure vein and decreased return of blood to the fetal heart, resulting in decreased CO, hypotension, and a compensatory FHR _.
A. Acceleration
B. Early deceleration
C. Late deceleration
D. Variable deceleration
A. Acceleration

With complete umbilical cord occlusion, the two umbilical arteries also become occluded, resulting in sudden fetal hypertension, stimulation of the baroreceptors, and a sudden _ in FHR.
A. Increase
B. Decrease
B. Decrease

Central are located in the medulla oblongata; peripheral are found in the carotid sinuses and aortic arch.
A. Baroreceptors
B. Chemoreceptors
B. Chemoreceptors

When a fetus is stressed, catecholamine release (epinephrine, norepinephrine) occurs from the medulla oblongata, shunting blood _ the brain, heart, and adrenal glands.
A. Toward
B. Away from
A. Toward

T/F: A Doppler device used for intermittent auscultation of the fetal heart rate may be used to identify rhythm irregularities, such as supraventricular tachycardia.
False

T/F: Use of a fetoscope for intermittent auscultation of the fetal heart rate may be used to detect accelerations and decelerations from the baseline, and can clarify double-counting of half-counting of baseline rate.
True

T/F: In the context of moderate variability, late decelerations are considered neurogenic in origin and are typically amenable to intrauterine resuscitation techniques directed towards maximizing uterine blood flow.
True

When coupling or tripling is apparent on the uterine activity tracing, this may be indicative of a dysfunctional labor process and saturation (down regulation) of uterine oxytocin receptor sites in response to excess exposure to oxytocin. Which of the following interventions would be most appropriate?
A. Normal response; continue to increase oxytocin titration
B. Turn patient on side
C. Decrease or discontinue oxytocin infusion
C. Decrease or discontinue oxytocin infusion

The most common tachyarrhythmia in fetuses, supraventricular tachycardia, typically occurs at a rate of _ to _ bpm with minimal or absent variability.
A. 160-200
B. 200-240
C. 240-260
C. 240-260

In a patient with oxytocin-induced tachysystole with normal fetal heart tones, which of the following should be the nurse’s initial intervention?
A. Assist the patient to lateral position
B. Discontinue Pitocin
C. Administer IV fluid bolus
A. Assist the patient to lateral position

In a patient with oxytocin-induced tachysystole with indeterminate or abnormal fetal heart tones, which of the following should be the nurse’s initial intervention?
A. Assist the patient to lateral position
B. Discontinue Pitocin
C. Administer IV fluid bolus
B. Discontinue Pitocin

Fetal hypoxia and acidemia are demonstrated by pH < _ and base excess < _.
< 7.15; < -8

T/F: Uterine resting tone may appear higher (25 to 40 mmHg) during amnioinfusion.
True

_ denotes an increase in hydrogen ions in the fetal blood.
A. Acidosis
B. Acidemia
C. Hypercapnia
B. Acidemia

occurs when there is low bicarbonate (base excess) in the presence of normal pressure of carbon dioxide (PCO2) values.
A. Metabolic acidosis
B. Respiratory acidosis
C. Metabolic alkalosis
A. Metabolic acidosis

occurs when there is high PCO2 with normal bicarbonate levels.
A. Metabolic acidosis
B. Respiratory acidosis
C. Metabolic alkalosis
B. Respiratory acidosis

occurs when the HCO3 concentration is lower than normal.
A. Base deficit
B. Base excess
C. Metabolic acidosis
A. Base deficit

occurs when the HCO3 concentration is higher than normal.
A. Base deficit
B. Base excess
C. Metabolic acidosis
B. Base excess

_ is defined as the energy-consuming process of metabolism.
Anabolism

_ is defined as the energy-releasing process of metabolism.
Catabolism

Normal oxygen saturation for the fetus in labor is % to %.
30% to 65%

pH 7.05
PO2 21
PCO2 72
HCO3 24
Base excess -12

A. Metabolic acidosis
B. Respiratory acidosis
C. Mixed acidosis
B. Respiratory acidosis

pH 7.0
PO2 18
PCO2 54
HCO3 20
Base deficit 14

A. Metabolic acidosis
B. Respiratory acidosis
C. Mixed acidosis
A. Metabolic acidosis

pH 7.02
PO2 17
PCO2 72
HCO3 19
Base deficit 16

A. Metabolic acidosis
B. Respiratory acidosis
C. Mixed acidosis
C. Mixed acidosis

With the finding of a single umbilical artery, what would you expect to observe with Doppler flow studies?
A. Decreased blood perfusion from the fetus to the placenta
B. Decreased blood perfusion from the placenta to the fetus
C. Homeostatic dilation of the umbilical artery
A. Decreased blood perfusion from the fetus to the placenta

Two umbilical arteries flow from the fetus to the placenta

A patient presents with a small amount of thick dark blood clots who denies pain and whose abdomen is soft to the touch. Which component of oxygen transport to the fetus could potentially be compromised by this bleeding?
A. Affinity
B. Saturation
C. Delivery
C. Delivery

Which intrinsic homeostatic response is the fetus demonstrating when abrupt variable decelerations are present?
A. Baroreceptor
B. Catecholamine
C. Sympathetic
A. Baroreceptor

An infant was delivered via cesarean. Umbilical cord blood gases were: pH 6.88, PCO2 114, PO2 10, bicarbonate 15, base excess (-) 20. The initial neonatal hemocrit was 20% and the hemoglobin was 8.

Which interpretation of these umbilical cord and initial neonatal blood results is correct?
A. Base buffers have been used to maintain oxygenation
B. The mother was probably hypoglycemic
C. The neonate is anemic
C. The neonate is anemic

An infant was delivered via cesarean. Umbilical cord blood gases were: pH 6.88, PCO2 114, PO2 10, bicarbonate 15, base excess (-) 20. The initial neonatal hemocrit was 20% and the hemoglobin was 8.

These umbilical cord blood gases indicate
A. Asphyxia related to umbilical and placental abnormalities
B. Hypoxia related to neurological damage
C. Mixed acidosis
C. Mixed acidosis

Which FHR tracing features must be assessed to distinguish arrhythmias from artifact?
A. Shape and regularity of the spikes
B. Spikes and variability
C. Spikes and baseline
A. Shape and regularity of the spikes

How might a fetal arrhythmia affect fetal oxygenation?
A. By increasing fetal oxygen affinity
B. By increasing sympathetic response
C. By reducing fetal perfusion
C. By reducing fetal perfusion

Which medication is used to treat fetal arrhythmias?
A. Digoxin
B. Labetolol
C. Nifedipine
A. Digoxin

Inotropic – promotes regular and effective cardiac contraction

Fetal hydrops may present on ultrasound as fetal scalp edema and increased abdominal fluid as a results of
A. An increase in gestational age
B. Congestive heart failure
C. Sustained oligohydramnios
B. Congestive heart failure

What might increase fetal oxygen consumption?
A. Hyperthermia
B. Umbilical cord compression
C. Uterine tachysystole
A. Hyperthermia

Increases metabolism and oxygen consumption

Which assessment or intervention would be least appropriate in a patient whose FHR tracing revealed tachycardia and a prolonged deceleration?
A. Change maternal position to right lateral
B. Further assess fetal oxygenation with scalp stimulation
C. Perform a vaginal exam to assess fetal descent
B. Further assess fetal oxygenation with scalp stimulation

Only used with normal baseline rate and never during decels; not an intervention

Which of the following pieces of information would be of highest priority to relay to the neonatal team as they prepare for an emergency cesarean delivery?
A. FHR arrhythmia, meconium, length of labor
B. Gestational age, meconium, arrhythmia
C. Gravidity & parity, gestational age, maternal temperature
B. Gestational age, meconium, arrhythmia

Which medications used with preterm labor can affect the FHR characteristics?
A. Terbutaline and antibiotics
B. Betamethasone and terbutaline
C. Antibiotics and narcotics
B. Betamethasone and terbutaline

What characterizes a preterm fetal response to stress?
A. More frequently occurring late decelerations
B. More frequently occurring prolonged decelerations
C. More rapid deterioration from Category I to Category II or III
C. More rapid deterioration from Category I to Category II or III

More likely to be subjected to hypoxia

***A woman being monitored externally has a suspected fetal arrhythmia. The most appropriate action is to
A. Insert a spiral electrode and turn off the logic
B. Turn the logic on if an external monitor is in place
C. Use a Doppler to listen to the ventricular rate
A. Insert a spiral electrode and turn off the logic

*** The fetus responds to a significant drop of PO2 by
A. Increasing O2 consumption
B. Reducing lactic acid production
C. Shifting blood to vital organs
C. Shifting blood to vital organs

Which factor influences blood flow to the uterus?
A. Fetal arterial pressure
B. Intervillous space flow
C. Maternal arterial vasoconstriction
C. Maternal arterial vasoconstriction

***Betamethasone given to the mother can transiently affect the FHR by
A. Decreasing variability
B. Increasing variability
C. Lowering the baseline
A. Decreasing variability

In a fetal heart rate tracing with marked variability, which of the following is likely the cause?
A. Recent ephedrine administration
B. Recent epidural placement
C. Fetal acidemia
A. Recent ephedrine administration

A fetal heart rate change that can be seen after administration of butorphanol (Stadol) is
A. Bradycardia
B. Marked variability
C. Sinusoidal-appearing
C. Sinusoidal-appearing

The FHR pattern that is likely to be seen with maternal hypothermia is
A. Bradycardia
B. Marked variability
C. Tachycardia
A. Bradycardia

*** Baseline FHR variability is determined in what amount of time, excluding accelerations and decelerations?
A. 10 min
B. 20 min
C. 30 min
A. 10 min

Which of the following tachyarrhythmias can result in fetal hydrops?
A. Persistent supraventricular tachycardia
B. Premature atrial contractions
C. Sinus tachycardia
A. Persistent supraventricular tachycardia

*** A preterm fetus with persistent supraventricular tachycardia that is not hydropic is best treated with maternal administration of
A. Digoxin
B. Phenobarbital
C. Terbutaline
A. Digoxin

The initial response in treating a primigravida being induced for preeclampsia who has a seizure is
A. Administer terbutaline to slow down uterine activity
B. Initiate magnesium sulfate
C. Perform an immediate cesarean delivery
B. Initiate magnesium sulfate

Which FHR sounds are counted with a stethoscope and a fetoscope?
A. Atrial
B. Atrial and ventricular
C. Ventricular
C. Ventricular

*** When using auscultation to determine FHR baseline, the FHR should be counted after the contractions for
A. 5-10 sec
B. 15-30 sec
C. 30-60 sec
C. 30-60 sec

A woman who is 34 weeks’ gestation is counting fetal movements each day. Today she counted eight fetal movements in a two-hour period. Based on her kick counts, this woman should
A. Continue counting for one more hour
B. Discontinue counting until tomorrow
C. Notify her provider for further evaluation
C. Notify her provider for further evaluation

A BPP score of 6 is considered
A. Abnormal
B. Normal
C. Equivocal
C. Equivocal

*** As fetal hypoxia (asphyxia) worsens, the last component of the BPP to disappear is fetal
A. Breathing
B. Movement
C. Tone
C. Tone

The legal term that describes a failure to meet the required standard of care is
A. Breach of duty
B. Negligence
C. Proximate cause
A. Breach of duty

*** Regarding the reliability of EFM, there is
A. Good interobserver reliability
B. Good intraobserver reliability
C. Poor interobserver and intraobserver reliability
C. Poor interobserver and intraobserver reliability

The objective of intrapartum FHR monitoring is to assess for fetal
A. Acidemia
B. Oxygenation
C. Well-being
B. Oxygenation

Use of the terms “beat-to-beat” variability and “long-term” variability is not recommended by the NICHD because in clinical practice
A. They are visually determined as a unit
B. Both signify an intact cerebral cortex
C. Clinical management is unchanged
A. They are visually determined as a unit

Late decelerations of the FHR are associated most specifically with
A. Transient fetal tissue metabolic acidosis during a contraction
B. Transient fetal hypoxemia during a contraction
C. Transient fetal asphyxia during a contraction
B. Transient fetal hypoxemia during a contraction

Assessment of FHR variability
A. Requires a fetal scalp electrode
B. Includes quantification of beat-to-beat changes
C. Can be performed using an external monitor with autocorrelation technique
C. Can be performed using an external monitor with autocorrelation technique

The “overshoot” FHR pattern is highly predictive of
A. Fetal hypoxia
B. Preexisting fetal neurological injury
C. None of the above
C. None of the above

A Category II tracing
A. Predicts abnormal fetal acid-base status
B. Excludes abnormal fetal acid-base status
C. Is not predictive of abnormal fetal acid-base status
C. Is not predictive of abnormal fetal acid-base status

Plans of the health care team with a patient with a sinusoidal FHR pattern may include
A. Administration of an NST
B. Administration of tocolytics
C. Kleinhauer-Betke lab test
C. Kleinhauer-Betke lab test

Stimulation of the fetal vagus nerve will
A. Increase FHR
B. Decrease FHR
C. Initially increase, then decrease FHR
B. Decrease FHR

Which of the following is not true when assessing preterm fetuses?
A. FHR baseline may be in upper range of normal (150-160 bpm)
B. They may have fewer accels, and if <35 weeks, may be 10×10
C. Variability may be in lower range for moderate (6-10 bpm)
B. They may have fewer accels, and if <35 weeks, may be 10×10

One of the side effects of terbutaline as a tocolytic is
A. Fetal bradycardia
B. Increased oxygen consumption
C. Marked variability
B. Increased oxygen consumption

Common problems seen during monitoring of postterm fetuses include all of the following except
A. Baseline may be 100-110bpm
B. Increased variables
C. Polyhydramnios
C. Polyhydramnios

Which of the following is not commonly affected by corticosteroids?
A. Doppler flow studies
B. FHR baseline
C. Frequency of FHR accelerations
A. Doppler flow studies

A fetus of a diabetic mother may commonly develop
A. Polyhydramnios
B. Supraventricular tachycardia
C. Third-degree heart block
A. Polyhydramnios

The fetus of a mother with preeclampsia is at high risk for developing
A. Intrauterine growth restriction (IUGR)
B. Macrosomia
C. Polyhydramnios
A. Intrauterine growth restriction (IUGR)

High resting tone may occur with an IUPC because of all of the following except
A. Extraovular placement
B. Maternal BMI
C. Multiple gestations
B. Maternal BMI

Which of the following is the primary neurotransmitter of the sympathetic branch of the autonomic nervous system?
A. Acetylcholine
B. Dopamine
C. Norepinephrine
C. Norepinephrine

Which of the following is the primary neurotransmitter of the parasympathetic branch of the autonomic nervous system?
A. Acetylcholine
B. Dopamine
C. Norepinephrine
A. Acetylcholine

Which of the following is responsible for variations in the FHR and fetal behavioral states?
A. Cerebellum
B. Cerebral cortex
C. Medulla oblongata
B. Cerebral cortex

When the umbilical vessels traverse the membranes to the placenta without any cord protection, this is called
A. Placenta previa
B. Succenturiate lobe (SL)
C. Velamentous insertion
C. Velamentous insertion

Which of the following is the primary factor in uteroplacental blood flow?
A.. Fetal heart rate
B. Maternal cardiac output
C. Maternal oxygen consumption
B. Maternal cardiac output

Which of the following occurs when the parasympathetic branch of the autonomic nervous system is stimulated?
A. Norepinephrine release
B. Slowed conduction to sinoatrial node
C. Increase in fetal heart rate
B. Slowed conduction to sinoatrial node

Which of the following is responsible for fetal muscle coordination?
A. Cerebellum
B. Cerebral cortex
C. Medulla oblongata
A. Cerebellum

During periods of fetal tachycardia, FHR variability is usually diminished due to
A. The dominance of the parasympathetic nervous system
B. The dominance of the sympathetic nervous system
C. Stimulation of the fetal vagus nerve
A. The dominance of the parasympathetic nervous system

Periodic accelerations can indicate all of the following except
A. Stimulation of fetal chemoreceptors
B. Tracing is a maternal tracing
C. Umbilical vein compression
A. Stimulation of fetal chemoreceptors

A transient decrease in cerebral blood flow (increased cerebral blood pressure) during a contraction may stimulate _ and may cause a(n) _
A. Baroreceptors; early deceleration
B. Baroreceptors; late deceleration
C. Chemoreceptors; early deceleration
A. Baroreceptors; early deceleration

The primary physiologic goal of interventions for late decelerations is to
A. Decrease maternal oxygen consumption
B. Maximize placental blood flow
C. Maximize umbilical circulation
B. Maximize placental blood flow

Which of the following is most responsible for producing FHR variability as the fetus grows?
A. Maturation of the parasympathetic nervous system
B. Maturation of the sympathetic nervous system
C. Release of maternal prostaglandins
A. Maturation of the parasympathetic nervous system

Which of the following is not a type of supraventricular dysrhythmia?
A. Premature atrial contraction (PAC)
B. Premature ventricular contraction (PVC)
C. Supraventricular tachycardia (SVT)
B. Premature ventricular contraction (PVC)

Which is the most common type of fetal dysrhythmia?
A. Premature atrial contraction (PAC)
B. Premature ventricular contraction (PVC)
C. Third-degree heart block
A. Premature atrial contraction (PAC)

All of the following are traits of fetal supraventricular tachycardia (SVT), but which is most problematic?
A. Decreases diastolic filling time
B. Dramatically increases oxygen consumption
C. Often leads to ventricular tachycardia (VT)
C. Often leads to ventricular tachycardia (VT)

Which abnormal FHR pattern is most likely to lead to hydrops in the fetus?
A. Marked variability
B. A premature ventricular contraction (PVC)
C. Supraventricular tachycardia (SVT)
C. Supraventricular tachycardia (SVT)

Which of the following is an irregular FHR pattern associated with normal conduction and rate?
A. Premature atrial contractions (PACs)
B. Sinus arrhythmias
C. Sinus tachycardias
B. Sinus arrhythmias

Which of the following is one example of a fetal tachyarrhythmia?
A. Second-degree heart block, Type I
B. Atrial fibrillation
C. Premature atrial contraction (PAC)
B. Atrial fibrillation

(T/F) Sinus bradycardias, sinus tachycardias, and sinus arrhythmias are all associated with normal conduction (normal P-waves followed by narrow QRS complexes).
True

(T/F) An internal scalp electrode will detect the actual fetal ECG.
True

(T/F) An internal scalp electrode can solely diagnose a fetal dysrhythmia.
False

_ are patterns of abnormal FHR associated with variability in R-to-R intervals, but with normal P-waves preceding normal QRS complexes.
A. Arrhythmias
B. Complete heart blocks
C. Dysrhythmias
A. Arrhythmias

_ are abnormal FHR rhythms associated with disordered impulse formation, conduction, or both.
A. Arrhythmias
B. Supraventricular tachycardias
C. Dysrhythmias
C. Dysrhythmias

Which of the following fetal dysrhythmias may be related to maternal hyperthyroidism?
A. Sinus tachycardia
B. Premature atrial contractions (PACs)
C. Third-degree heart block
B. Premature atrial contractions (PACs)

With _ premature ventricular contractions (PVCs), the baseline and variability are obscured.
A. Idioventricular
B. Bigeminal
C. Trigeminal
B. Bigeminal

With _ premature ventricular contractions (PVCs), the upward spikes will be slightly longer than the downward spikes.
A. Idioventricular
B. Bigeminal
C. Trigeminal
C. Trigeminal

Which of the following dysrhythmias may progress to atrial fibrillation or atrial flutter?
A. Premature atrial contractions (PACs)
B. Supraventricular tachycardia (SVT)
C. Sinus tachycardia
B. Supraventricular tachycardia (SVT)

Which of the following is not commonly caused by administration of indomethacin?
A. Decreased fetal urine (decreased amniotic fluid index [AFI])
B. Decreased FHR baseline
C. Increased variable decelerations
B. Decreased FHR baseline

Which of the following is not commonly caused by terbutaline administration?
A. Increased FHR baseline
B. Decreased FHR late decelerations
C. Increased maternal HR
B. Decreased FHR late decelerations

Which of the following is not commonly caused by nifedipine administration?
A. Maternal hypotension
B. Decreased uterine blood flow
C. Increased FHR accelerations
C. Increased FHR accelerations

Which of the following would likely be affected by betamethasone administration?
A. Fetal echocardiogram
B. Biophysical profile (BPP) score
C. Contraction stress test (CST)
B. Biophysical profile (BPP) score

Fetal breathing decreased with betamethasone administration

Which of the following is not typically associated with a postterm pregnancy?
A. Meconium-stained amniotic fluid
B. Presence of late decelerations in the fetal heart rate
C. Polyhydramnios
C. Polyhydramnios

Which of the following is the most appropriate method of monitoring a patient who is a gestational diabetic?
A. Daily NSTs
B. Twice-weekly BPPs
C. Weekly contraction stress tests
B. Twice-weekly BPPs

Which of the following is not commonly caused by magnesium sulfate?
A. Increased FHR baseline
B. Decreased FHR variability
C. Decreased FHR accelerations
A. Increased FHR baseline

pH 6.9, PO2 15, PCO2 55, HCO3 18, BE -22
The nurse reviews the arterial gas results and concludes that the fetus had _ acidosis. With results such as these, you would expect a _ resuscitation.
A. Metabolic; lengthy
B. Metabolic; short
C. Respiratory; lengthy
A. Metabolic; lengthy

Which of the following umbilical artery cord gases would most likely result in a fetus who had a Category I strip, then had a cord prolapse and was delivered within 3 minutes?
A. pH 7.17, PO2 22, PCO2 70, HCO3 24, BE -5
B. pH 7.25, PO2 23, PCO2 46, HCO3 22, BE -8
C. pH 7.02, PO2 18, PCO2 56, HCO3 15, BE -18
A. pH 7.17, PO2 22, PCO2 70, HCO3 24, BE -5

Which of the following is most likely to result in absent end diastolic flow during umbilical artery velocimetry?
A. Preeclampsia
B. Preterm labor
C. Previous cesarean delivery
A. Preeclampsia

A contraction stress test (CST) is performed. Late decelerations were noted in two out of the five contractions in 10 minutes. This is interpreted as
A. Positive
B. Negative
C. Suspicious
C. Suspicious

A contraction stress test (CST) is performed. Two variable decelerations were seen on the FHR tracing and there were four contractions in 10 minutes. This is interpreted as
A. Positive
B. Negative
C. Suspicious
B. Negative

A contraction stress test (CST) is performed. No decelerations were noted with the two contractions that occurred over 10 minutes. This is interpreted as
A. Positive
B. Negative
C. Unsatisfactory
C. Unsatisfactory

In a patient whose CST reveals late decelerations with three out of the four induced contractions, which of the following would be the least appropriate plan for treatment?
A. Obtain physician order for BPP
B. Prepare for possible induction of labor
C. Repeat CST in 24 hours
C. Repeat CST in 24 hours

For a patient at 38 weeks’ gestation with a BPP score of 6, select the most appropriate course of action.
A. Repeat in one week
B. Consider induction of labor
C. Prepare for cesarean delivery
B. Consider induction of labor

For a patient at 35 weeks’ gestation with a BPP score of 4, select the most appropriate course of action.
A. Repeat in 24 hours
B. Obtain physician order for CST
C. Prepare for probable induction of labor
C. Prepare for probable induction of labor

(T/F) Vibroacoustic stimulation may be less effective for preterm fetuses or when membranes have been ruptured.
True

_ cord blood sampling is predictive of uteroplacental function.
A. Arterial
B. Venous
C. Maternal
B. Venous

(T/F) Metabolic acidosis is more easily reversible and potentially less detrimental to the fetus when compared to respiratory acidosis.
False

(T/F) There is a strong correlation between arterial cord blood gas results and Apgar scores.
False

Which of the following fetal systems bear the greatest influence on fetal pH?
A. Heart and lungs
B. Lungs and kidneys
C. Sympathetic and parasympathetic nervous systems
B. Lungs and kidneys

All of the following are components of liability except
A. Breach of duty
B. Chain of command
C. Damages/loss
B. Chain of command

Elements of a malpractice claim include all of the following except
A. Breach of duty
B. Deposition
C. Injury or loss
B. Deposition


pH 6.86
pCO2 28
pO2 2.1
HCO3 4.0
B.D. 42
the umbilical arterial cord blood gas values reflect
A. metabolic acidemia
B. mixed acidemia
C. respiratory acidemia
A. Metabolic acidemia

Content on exam
-Pattern recognition & intervention: 70%
-Physiology: 11%
-Fetal assessment methods: 9%
-EFM equipment: 5%
-Professional issues: 5%

Pattern recognition & intervention
-FHR baseline ✓
-FHR variability ✓
-FHR accelerations ✓
-FHR decelerations ✓
-Normal uterine activity ✓
-Abnormal uterine activity ✓
-Fetal dysrhythmias ✓
-Maternal complications ✓
-Uteroplacental complications ✓
-Fetal complications ✓

FHR Descriptors
1) Baseline
2) Variability
3) Presence of accels
4) Presence of decels
5) Changes in trends overtime

FHR Baseline
Average FHR rounded to nearest 5 during a 10 min window
-110 to 160
-excludes accels, decels, & marked variability
-must have 2 mins to identify as a baseline (doesn’t need to be continuous)

Fetal Bradycardia
<110 for ≥10 min
-Causes: hypotension (ex: after epi), cord prolapse, head compression, congenital defect, rapid descent, abruption or rupture, tachysystole, post dates, hypoglycemia, lupus (heart block)
-With ↓ O2, blood will be shunted to brain, heart, & adrenals, eventually ↓ FHR to ↓ O2 demands of heart muscle
-Verify not mom’s HR, vaginal exam (r/o prolapse), resuscitate, evaluate arrhythmia, expedite delivery

Fetal Tachycardia

160 for ≥10 min
-Causes: fetal anemia, maternal fever or infection, fetal immaturity (preterm), SVT, maternal anxiety (catecholamines), dehydration, hyperthyroid, hypoxia
-Med causes: terbutaline, catecholamines (epinephrine, norepi)
-Assess mom’s temp & infection risk (GBS, PROM)

FHR Variability
Irregular in amplitude & frequency, quantified by peak to trough
-Caused by sympathetic vs parasympathetic, r/t neuro maturity
-Less in preterm due to undeveloped CNS
-Absent: undetectable, flat
-Minimal: ≤5 bpm but detectable
-Moderate: 6-25 bpm
-Marked: >25 bpm (indeterminate baseline), significance unknown

Minimal variability
≤5 bpm but detectable
Sleep, sedated, or sick
-Sleep cycle: 20-60 mins
-Sedated: CNS depressant (ex: mag), 1-2 hrs
-Sick (acidemia): unresolved w intervention
-Priority: maximize oxygenation (position, bolus, O2 if needed)

Moderate variability
6 to 25 bpm
-Reliably predicts the absence of metabolic acidosis (even w decels)

FHR Accelerations
Reliably predicts absence of metabolic acidemia (spontaneous or stimulated)
-Onset to peak in <30 sec
-For ≥32 wks: 15×15 (peak ≥15 bpm above baseline lasting ≥15 sec)
-For <32 wks: 10×10
-Prolonged accel: 2-9 mins (at 10 becomes change of baseline)

Early deceleration
Nadir aligns w contraction peak, gradual onset (≥30 secs to nadir), benign vagal response
1) Pressure on fetal head
2) Increased intracranial pressure
3) Alteration in cerebral blood flow
4) Central vagal stimulation
5) FHR deceleration

Periodic vs Episodic
Periodic: caused by contractions
-recurrent: occurs w ≥50% of contractions in 20 min
-intermittent: w <50% of contractions in 20 mins
Episodic: spontaneous

Variable deceleration
Caused by cord compression
-Interventions: position change, amnioinfusion
-Abrupt onset: <30 seconds from onset to nadir dropping ≥15 bpm lasting 15 secs to <2min
-Transient rise in PCO2 & fall in PO2

Mechanisms of variable decelerations
Abruptness r/t pressure changes
1) Vein obstruction → reflex tachy
-↓ venous return & cardiac output → hypotens → baroreceptor reflex ↑ in FHR to maintain BP
2) Arterial obstruction → decreased FHR
-obstructed blood flow back to placenta → HTN → baroreceptor reflex of slowing FHR to maintain BP

Late decelerations
Uteroplacental insufficiency
-Indicative of transient fetal hypoxemia
-Gradual onset: ≥30 secs to nadir w nadir occurring after peak of contraction
-Priority is to maximize uteroplacental blood flow: position lateral (off vena cava & aorta), fluid bolus (perfusion), O2, avoid tachysystole

Mechanisms of a late deceleration
Low O2 → chemoreceptor response peripheral vasoconstriction → blood flow to vital organs → HTN → baroreceptor vagal stimulation → FHR decel
1) Decreased uteroplacental oxygenation (transient hypoxemia)
2) Chemoreceptor stimulation
3) Alpha adrenergic response (catecholamines, peripheral vasoconstriction)
4) Fetal HTN
5) Baroreceptor stimulation
6) Parasympathetic response
7) FHR deceleration
8) ↓ myocardial stress

Prolonged deceleration
Decrease of ≥15 bpm lasting 2 to 9 mins (≥10 = change of baseline)
-Vagal stimulation
-Causes: hypotension, maternal hypoxia, cord prolapse, rapid decent, profound cord compression, uterine rupture

Sinusoidal pattern
Visually apparent, smooth, sine wave-like pattern in FHR lasting ≥20 minutes
-oscillation frequency: 3-5 cycles/min
-no variability classification or reactivity
-r/t severe anemia: previa, hemorrhage, abruption, RH isoimmunization, asphyxia, infection, cardiac anomaly, twin to twin transfusion, gastroschesis
-Transient if <20min, can be r/t thumb sucking or opioids (stadol, fentanyl)

Interventions
-Position change: off of vena cava & aorta, least invasive, 1st line of treatment
-Fluid bolus
-Amnioinfusion (for variables)
-Tocolytics (terb)
-Ephedrine to ↑ BP
-Supplemental O2: not used w O2 >95%, can cause vasoconstriction, free radical formation, ocular toxicity if used limit to 15-30min

Category I tracing
Normal acid base balance
-Baseline between 110 to 160
-Moderate variability
-No late, variable, or prolonged decels
-May have early decels
-May or may not have accels

Category II tracing
Indeterminate acid base balance
-Minimal variability
-Marked variability
-Late or variable decels
-Bradycardia with variability
-Tachycardia
-Prolonged decels
-Absent variability w NO decels
-Absence of induced acccel WITH fetal stimulation

Category III tracing
Predictive of abnormal acid base balance at that moment
-Sinusoidal rhythm: has to last ≥20min, r/t anemia (previa, bleeding, abruption)
-Absent variability WITH one of the following: bradycardia, recurrent late or recurrent variable decels
-Decide for c/s within 30min

Normal uterine activity
≤5 contractions in 10 mins averaging over 30 min window
-adequate contractions: q2-3 lasting 80-90s
-intensity: 25-75 mmhg
-resting tone: 10-15 mmhg
-MVU: 200-220 in 10 mins
-intercontraction interval (relaxation time) should be 45-60sec

Tachysystole

5 contractions in 10 mins averaged over 30 min window
-tetanic contractions: >90 secs
-position change
-500 LR to dilute uterotonic
-↓ pitocin (see protocol)
-tocolytic (terbutaline)
-O2 if decel

Tachysystole & pitocin
With fetal tolerance
-If not resolved in 15min, ↓ pit by 1/2
-If not resolved in another 15min, pause pit
-If pit’s off for <30 min, resume pit at 1/2 of current dose -If off for >30 min, start @ initial order dose
With fetal intolerance: pit off immediately

Hypertonous labor
Frequent & painful but poor quality contractions occurring in latent labor
-despite increased tone, not enough pressure to cause cx change
-indicative of CPD or malpresentation
-tx: comfort care, pitocin, AROM

Hypotonic labor
Weak & insufficient labor occurring during the active phase
-inadequate, infrequent, & less intense contractions don’t dilate or efface cx
-caused by tired uterus or overdistention (poly, multiple gestation, LGA)

Arrest of labor

6cm dilated w/ ROM & one of the following w no cx change:
-4 hrs of adequate contractions (>200 MVUs)
-6 hrs of inadequate contractions

Fetal dysrhythmias
1) SVT: 210-300 bpm
-tx w digoxin (↑ dose to cross placenta)
-concern w hydrops, CHF (heart stress), ↑ O2 demand & use, ↓ stroke volume, demise
2) Congenital heart block (<60 bpm): 3rd degree → concern w hydrops, lupus, CHF
3) Ectopic beats: extra beats heard, benign, may be transient

Lupus (SLE)
Can cause congenital 3rd degree heart block
-bradycardia
-autoimmune → inflammatory response → overgrowth of collagen in heart muscle
-damages fetal conduction system of the heart, attacks fetal tissue
-usually diagnosed in 2nd trimester

Preterm labor
Cx dilation bw 20-36.6 wks
-late preterm: 34-36.6 wks
-common risk factors: multiples, previous PTD (greatest), uterine or cx abnormalities, infection, <18 months bw pregnancies, IUGR
-↓ reserve & ↓ parasympathetic maturity
-↑ baseline, ↓ variability, ↑ variable decels

Treatment for preterm labor
1) Steroids (BMZ, DMZ): allows for synthesis of surfactant & ↓ intracranial hemorrhage risk
-most beneficial after 48 hrs, effects last 1-2 wks, can give 2nd round after 2 wks
-↑ FHR, ↓ variability & accels
-↑ blood sugar & fluid retention
2) Terbutaline (betamimetic): delays PTD by 2-3 days to give steroids
-SE: hyperglycemia, hypotension, tachycardia
-Not given w ↑HR, bleeding, uncontrolled diabetes
3) Nifedipine / procardia (Ca channel blocker): ↓ contractility of smooth muscles, vasodilates
4) Indocin (NSAID, prostaglandin inhibitor): tocolytic
-AE: oligo, GI upset, renal failture, PP hemorrhage
5) Mag: neuroprotection, vasodilates ↑ blood flow to brain, ↓ risk of CP
-can cause fetal bradycardia & ↓ variability

Postdates pregnancy

42 wks
-risk of oligo, ↓ placental perfusion, & meconium aspiration
-newborn appearance: peeling dry wrinkled skin, long nails, thinner (weight loss)
-↑ rate of c/s, recommended to induce at 41 wks
-↑ late decels (old placenta) & variables (oligo)

Hypertension (HTN)
BP > 140/90
-tx w Ca channel blockers, thiazides, BB
-avoid ACE inhibitors & ARBs (congenital defect)
1) Chronic: present prior to pregnancy
-risk of superimposed preeclampsia, abruption, stroke (endothelial damage to vessels), stress on L ventricle (to overcome pressure)
2) Gestational: begins after 20 wks, returns to normal PP (no proteinuria: <300mg/<0.3 pcr)
-↑ BP is sustained 6 hrs apart
-risk of poor perfusion, SGA

Preeclampsia & HELLP
↑BP & proteinuria or a s/s: poor implantation of arterioles & trophoblasts cause ↓ blood flow to placenta, ↑ BP to compensate
-mild: 140/90, protein 300mg/24hr or 1-2g dipstick
-severe: 160/110, protein 5g/24hr or 3g dipstick, N/V, HA, hyperreflexia, clonus
-prevent w baby ASA in 1st trimester (vasodilate)
-damages kidneys, liver, heart, lungs, blood cells & vessels, neuro (cerebral edema), optic nerve
-AE: oligo, IUGR, pulmonary edema, HA, epigastric pain, blurry vision, abruption, IUFD, seizure, liver rupture, renal failure, DIC, CVA
-can cause eclampsia (seizure) from cerebral ischemia & edema, may abrupt or rapidly dilate
-HELLP: hemolysis, ↑ liver enzymes (AST, ALT), ↓ platelets (<100k)

Treatment of preeclampsia & HTN
-Induce by 39 wks
1) Mag: muscle relaxant used to prevent seizure
-loading dose 4-6g/hr, maintenance 2-4g/hr
-therapeutic lvl: 4-8 mEq/L
-↓ FHR variability, hypotonia, resp depression
-antidote: calcium gluconate 1g/10ml D5W
-SE: NV, flushing, HA, ↓ reflexes, pulmonary edema, hypotension, resp depression, oliguria
2) BB: don’t use w asthma or ↓ HR, caution w diabetes (masks hypoglycemia)
3) Hydralazine: relaxes smooth muscles
-can cause rebound tachycardia
4) Ca channel blocker (procardia, nifedipine): relaxes smooth muscles, ↑ renal perfusion & urinary output
5) Valium or keppra w eclampsia

Diabetes
-Excess glucose delays surfactant production (RDS)
-T1 & T2 more likely to cause congenital abnormalities (heart & neural tube defect)
-Hypoglycemia at birth: glucose crosses placenta but not insulin, fetus has ↑ insulin production & bottoms out after delivery
-Risk of LGA, fetal acidosis, impaired perfusion, polycythemia, demise, UTI, polyhydramnios, abruption, PPH, Pre-E (r/t vascular damage), miscarriage
-Induce at 38-39 wks

Types of diabetes
1) T1: insulin dependent (don’t secrete insulin)
2) T2: insulin resistant
3) Gestational: placenta acts as antagonist to insulin, demand for production ↑ 2-3x
-resolves w placental expulsion

Infection
1) GBS: bacteria normally found in the vagina & rectum, typically harmless but can pass to fetus during delivery
-can cause meningitis, pneumonia, or sepsis
-screened from 36-38 wks, tx w antibiotics during labor if positive
2) Chorioamnionitis: infection of membranes, can cause PTD, requires antibiotic tx (risk w PROM)
3) HepB: give infant hepB vax & HBIG
4) HIV: infant will need antivirals, no breastfeed
5) TORCH (toxoplasmosis, other, rubella, cytomegalovirus, herpes): commonly associated w congenital abnormalities
-other: syphillis, varicella zoster, parvovirus B19

Maternal obesity
-Risk of diabetes, preeclampsia, DVT, infection, PPH (↑ estrogen), miscarriage
-Fetus: NTD, heart defect, macrosomia (4000g, 8-13), PTD
-Recommended weight gain: 11-20 lbs (compared to 25-35 in average weight)

Uteroplacental complications
1) Previa: total = covering cx os, marginal = 2cm of os, low lying = 2-3.5 cm from os (usually resolves)
-painless bright red bleeding
-risk: AMA, uterine scarring, fibroids, smoking, multiples
-pelvic rest, bedrest, rhogam (rh-), steroids, PTD
2) Abruption: placental separation from uterus
-painful bleeding (may be concealed), ↑ frequency ↓ amplitude contractions, ↑ fundal height, rigid abd
3) Uterine rupture: risk w overdistended uterus or previous surgery (poly, LGA, TOLAC, pit)
-abrupt pain, fetal intolerance, no contractions, hematuria, head unengaged (loss of station), chest or shoulder pain (blood accumulation in peritoneum)

Fetal injury & hypoxia
1) Cerebral palsy: motor disorder caused by brain damage before or during birth
-Infection → meningitis → brain damage (TORCH, chorioamnionitis, GBS)
-Vacuum or forceps assisted delivery
-Poor perfusion of O2 (abruption, rupture, cord abnormality, HIE, asphyxia)
2) Subgaleal hematoma: brain bleed & IICP caused by external head trauma (vacuum or forceps)
3) HIE: hypoxic ischemic encephalopathy
-causes: cord compression, placental insufficiency, smoking, abruption, rupture, shoulder dystocia, anemia
-can cause IUGR, CP, epilepsy, cognitive impairment, organ damage

Physiology
-Uteroplacental circulation ✓
-Fetal circulation ✓
-Fetal HR regulation ✓
-Factors affecting fetal oxygenation ✓

Uteroplacental circulation
(R arrow indicates blood flow)
-Uterine artery → spiral arterioles → intervillous space → diffusion to chorionic villi → oxygenated blood & nutrients to umbilical vein → fetal circulation
-Opposite occurs to rid deoxygenated blood & waste from uterine artery
-Trophoblasts allow diffusion from chorionic villi & intervillous space without mom & baby RBCs mixing

Fetal circulation
-Umbilical vein → bypass liver through ductus venosus → inferior & superior vena cava → R atrium
-Ductus venosus has most highly oxygenated blood
-R atrium → L atrium via foramen ovale to bypass lungs → L ventricle → aorta
-Ductus arteriosus connects pulmonary artery to aorta to further bypass lungs
-Alveoli are filled with fluid shunting blood away from lungs
-In utero, R side has higher pressure keeping the foramen ovale open, w cord clamp this shifts to L side
-Iliac arteries → umbillical arteries → deoxygenated blood out

Fetal HR regulation
Controlled by the sinoatrial (SA) node
-Rate is affected by maturity of parasympathetic nervous system (cholinergic, beta cell relaxation of smooth muscles)
-Variability: sympathetic (alpha) vs parasympathetic (beta)
-Baroreceptors influence FHR in response to BP changes (↑ BP causes ↓ FHR)
-Chemoreceptors respond to hypoxia & ↑ CO2 w tachycardia & HTN

Factors affecting fetal oxygenation
-Uterine activity: contractions ↓ blood flow, causes stasis in intervillous space
-Maternal factors: AMA, poor nutrition, BMI <18 or >30, CHF, anemia, HTN, diabetes, infection
-Anesthesia: ↓ BP
-Drugs: cocaine & nicotine vasoconstricts, nicotine causes CO to cross placenta (↓ ability of Hgb to carry O2) marijuana ↑ CO2 lvls
-Placental factors: calcifications (post dates, smoking, clotting d/o, diabetes, HTN), previa, abruption
-Umbilical blood flow: cord compression, umbilical abnormalities (single artery), prolapse

Maternal factors affecting fetal oxygenation
1) ↓ maternal O2: respiratory distress, hypoventilation, seizures, trauma, smoking
2) ↓ maternal O2 carrying capacity: anemia, carboxihemoglobin (smoking, poisoning, drugs)
3) ↓ uterine blood flow: hypovolemia, hypotension, anesthesia, dorsal or lithotomy positioning
4) chronic maternal conditions: vasculopathy (lupus, diabetes, HTN), antiphospholipid

Uteroplacental factors affecting fetal oxygenation
1) tachysystole
2) placental abruption
3) uteroplacental dysfunction
4) placental infarction
5) chorioamnionitis
6) abnormalities (fibroids, septum)
7) abnormal implantation of placenta (previa) or cord (valementous, marginal, vasa previa)

Fetal assessment methods
-Auscultation ✓
-Fetal movement & stimulation ✓
-Non stress test (NST) ✓
-Biophysical profile (BPP) ✓
-Cord blood & acid base balance ✓

Auscultation
Intermittently listening to fetal heart sounds w fetoscope or doppler to assess FHR
-Detects baseline, rhythm, increases & decreases from baseline
-Cannot determine variability or classify decels
-Use of fetoscope can verify presence of arrhythmia (most accurate) & clarifies halving or doubling
-Listen to FHR before, during, & for 30 sec after contraction
-Feel for mom’s radial pulse to differentiate

Fetal movement & stimulation
-Kick count: 10 movements in 2 hrs (starting at 28 wks)
-To ↑ movement, have ice chips (auditory stimulation) or juice (sugar)
-Do not do scalp stim when FHR is not at baseline (don’t use during a decel)

Non stress test (NST)
Can be done >26 wks
-Reactive: 2 accels in 20 mins (can prolong to 40min) with moderate variability
-Acoustic stim: ≤3, 1 min apart lasting 3 sec each (not used for oligo or <32 wks), elicits startle reflex
-Valid for 24 hrs

Biophysical Profile (BPP)
Normal = 2 points, abnormal = 0
-Can be done ≥28 wks, valid for 7 days
-BPP done over 30 mins
1) Fetal heart rate
2) Breathing movements
3) Gross body movements
4) Muscle tone: extension/flexion
5) Amniotic fluid volume: (>2cm or AFI >5)

Scoring a BPP
Will get 2 points for each of the following:
-Reactive NST: 2 accles in 20min
-Breathing: 1 lasting 60 secs or 2 lasting 30 secs
-Gross movements: 3 of arms, legs, or body
-Tone: 1 flexion & extension of arms, legs, hands
-AFI: either a 2cm pocket or a total of 5cm (normal 5-25cm)

Interpretation of BPP
-8 to 10 = reassuring (unless oligo)
-6 = equivocal, repeat in 24 hrs (if pt is term then delivery is recommended)
-4 = further testing needed

Normal ABGs
-pH ≥7.1
-CO2 <60 -Bicarb >22
-pO2 >20
-BD <12 -BE: >-12

Cord blood & acid base balance
Direct measurement of oxygenation (at the time of delivery)
-Base deficit or base excess of +/-12 & a pH <7.1 = high probability of neonatal encephalopathy r/t intrapartum -Umbilical artery value is more predictive -Respiratory acidosis: ↑ CO2 >60, normal BD/BE, normal bicarb, can develop quickly & be resolved easily
-Metabolic acidosis: normal CO2, ↓ bicarb <22, abnormal BD/BE, r/t anaerobic metabolism (lactic acid build up), difficult to resolve
-CAN have both metabolic & respiratory (↑ CO2, ↓ bicarb, abnormal BD/BE)

Imbalance definitions
1) Hypoxia: ↓ O2 in tissue
2) Hypoxemia: ↓ O2 in blood
3) Acidosis: ↑ H+ in tissue
4) Acidemia: ↑ H+ in blood
5) Asphyxia: hypoxia w metabolic acidosis (causes organ damage)

EFM Equipment
-External EFM ✓
-Internal EFM (ISE & IUPC) ✓
-Artifact ✓
-Signal ambiguity ✓
-Failure & troubleshooting ✓

Use of EFM
Tells us whether fetus is oxygenated (indirect measurement)
-Does not ↓ rates of infant mortality or CP
-Poor at recognizing deoxygenation but reliable at predicting normal pH
-Low risk: q30 in active stage, q15 in 2nd stage
-High risk: q15 in active stage, q5 in 2nd stage

External FHR monitor
-Non invasive, records baseline, variability, & patterns
-Provides permanent record
-Can be intermittent or continuous
-May restrict maternal movement
-Signal may be affected by maternal position, obesity, fetal position & movement, poly
-Troubleshooting: reposition EFM or mom, apply gel, confirm fetal position
-Most commonly mistaken for mom w breech (near aorta)

External tocodynamometer
Detects changes in shape of the abdomen resulting from uterine tension
-place pressure sensor over the fundus at point of maximum intensity
-only monitors frequency & duration (does not monitor intensity)
-use in conjunction w palpation

Internal fetal scalp electrode (ISE)
-Directly monitors fetal ECG
-Indicated when continuous FHR is unable to be achieved with external EFM
-Requires ROM & dilation (2cm)
-Direct conduit for infection (pierces skin)
-Contraindications: previa, herpes, HIV, hepatitis (may use w GBS but not recommended)
-Avoid sutures, face, fontanelle, genitalia (can trace on buttocks)
-Will pick up maternal HR w demise
-May halve FHR >200

Intrauterine pressure catheter (IUPC)
A catheter that measures uterine pressure during labor
-measures intensity of contractions & resting tone
-requires ROM & dilation
-can be used for amnioinfusion
-insert to 35-45 cm mark
-troubleshooting: verify position (check markings, have pt cough), flush catheter, recalibrate
-risks: placental or uterine perforation, infection

Artifact
Interference in recording or transmission, seen as vertical lines along tracing
-Causes: vaginal exam, fetal movement, connection issue, hair or caput w ISE
-R/o arrhythmia (can see & hear skips, has baseline, isolated spikes), try another method of monitoring, verify w fetoscope

Signal ambiguity
EFM picking up maternal HR (or another fetus)
-Verify whether maternal HR or FHR w use of pulse ox monitor, ISE, or fetoscope
-Can appear as accels with pushing

Professional issues
-Legal ✓
-Ethics ✓
-Patient safety ✓
-Quality improvement ✓

Legal
-Informed consent: procedure risks, benefits, & alternate methods explained
-Consent is implied in emergencies
-Consent may be revoked at any time
-Negligence: breech of duty (standard of care not met) directly causing damages

  • failure to recognize, failure to take action, failure to document, incorrect procedure
    -Malpractice: an act or a failure to act that causes injury or harm (professional negligence, must have a license to commit)

Ethics
Moral principles that govern behavior, ideals, & standards
-Autonomy: allow pt to make decisions
-Veracity: tell the truth
-Fidelity: commitment
-Justice: equal & fair tx
-Beneficence: duty to do good
-Nonmaleficence: duty to do no harm
-Confidentiality & privacy

Patient safety
Sentinel event: patient safety event causing injury or death
-one of the leading causes is poor communication
-SBAR to improve communication

Quality improvement
Actions that lead to measurable improvement outcomes in pt care areas
-Standard of care: how qualified providers should act to prevent injury or harm
-EBP: best available practices based on research, guidelines, & expertise
-Validity: accuracy
-Reliability: consistency

Leave a Comment

Scroll to Top